You are on page 1of 362

CBSE X | Mathematics

Board Paper 2012 - Solution

CBSE Board
Class X Mathematics
Board Paper Solution – 2012
Time: 3 hours Total Marks: 90

Section A

1. Correct answer: B

Let AB be the tower and BC be its shadow. Let  be the angle of elevation of
the sun.

According to the given information,

BC = 3 AB … (1)

In ABC,

AB AB 1
tan     [Using (1)]
BC 3AB 3

1
We know that tan 30 =
3

 = 30

Hence, the angle of elevation of the sun is 30.

www.topperlearning.com 1
CBSE X | Mathematics

Board Paper 2012 - Solution

2. Correct answer: B

Diameters of two circles are given as 10 cm and 24 cm.

Radius of one circle = r1 = 5 cm, Radius of other circle = r2 = 12 cm

According to the given information,

Area of the larger circle = (r1)2  (r2 )2

 (5)2  (12)2
 (25  144)
 169
 (13)2

Radius of larger circle = 13 cm

Hence, the diameter of larger circle = 26 cm

3. Correct answer: C

Let the original radius and the height of the cylinder be r and h respectively.

Volume of the original cylinder = r²h

r
Radius of the new cylinder =
2

Height of the new cylinder = h

2
r  r2h
Volume of the new cylinder =    h =
2 4

r2h
Volume of the new cylinder 1
Required ratio =  42   1 : 4
Volume of the original cylinder r h 4

4. Correct answer: C

When two dice are thrown together, the total number of outcomes is 36.

Favourable outcomes = {(1, 1), (2, 2), (3, 3), (4, 4), (5, 5), (6, 6)}

Number of favourable outcomes 6 1


Required probability =  
Total number of outcomes 36 6

www.topperlearning.com 2
CBSE X | Mathematics

Board Paper 2012 - Solution

5. Correct answer: B

It is given that the point P divides AB in the ratio 2: 1.

Using section formula, the coordinates of the point P are

 1  1  2  4 1  3  2  6   1  8 3  12 
 2 1
,
2 1    3 , 3   3,5
   

Hence the coordinates of the point P are (3, 5).

6. Correct answer: A

Let the coordinates of the other end of the diameter be (x, y).

We know that the centre is the mid-point of the diameter. So, O(-2, 5)
is the mid-point of the diameter AB. The coordinates of the point A and
B are (2, 3) and (x, y) respectively.

Using mid-point formula, we have,

2x
2   4  2  x  x  6
2
3y
5  10  3  y  y  7
2

Hence, the coordinates of the other end of the diameter are (-6, 7).

7. Correct answer: C

The first 20 odd numbers are 1, 3, 5, … … 39

This is an AP with first term 1 and the common difference 2.

Sum of 20 terms = S20

20
S20  2(1)  (20  1)(2)  10 2  38  400
2 

Thus, the sum of first 20 odd natural numbers is 400.

www.topperlearning.com 3
CBSE X | Mathematics

Board Paper 2012 - Solution

8. Correct answer: A

It is given that 1 is a root of the equations ay2 + ay + 3 = 0 and y2 + y + b


= 0.

Therefore, y = 1 will satisfy both the equations.

 a(1)2 + a(1) + 3 = 0

a+a+3=0

 2a + 3 = 0

3
 a
2

Also, (1)2 + (1) + b = 0

 1 + 1 + b = 0  b = 2

3
 ab =  2  3
2

9. Correct answer: B

It is known that the lengths of tangents drawn from a point outside a circle
are equal in length.

Therefore, we have:

AP = AR … (1) (Tangents drawn from point A)

BP = BQ … (2) (Tangents drawn from point B)

CQ = CR … (3) (Tangents drawn from point C)

Using the above equations,

AR = 4 cm (AP = 4 cm, given)

BQ = 3 cm (BP = 3 cm, given)

AC = 11 cm  RC = 11 cm – 4 cm = 7 cm

 CQ = 7 cm

Hence, BC = BQ + CQ = 3 cm + 7 cm = 10 cm

www.topperlearning.com 4
CBSE X | Mathematics

Board Paper 2012 - Solution

10. Correct answer: A

It is known that the tangents from an external point to the circle are equal.

EK = EM, DK = DH and FM = FH … (1)

Perimeter of EDF = ED + DF + FE

= (EK – DK) + (DH + HF) + (EM – FM)

= (EK – DH) + (DH + HF) + (EM – FH) [Using (1)]

= EK + EM

= 2 EK = 2 (9 cm) = 18 cm

Hence, the perimeter of EDF is 18 cm.

SECTION B

11. It is given that the point A (0, 2) is equidistant from the points B(3, p) and
C(p, 5).

So, AB = AC  AB2 = AC2

Using distance formula, we have

 (0  3)2  (2  p)2  (0  p)2  (2  5)2


 9  4  p2  4p  p2  9
 4  4p  0
 4p  4
p 1

Hence, the value of p = 1.

12. Total number of outcomes is 50.

Favourable outcomes = {12, 24, 36, 48}

Number of favourable outcomes 4 2


Required probability =  
Total number of outcomes 50 25

www.topperlearning.com 5
CBSE X | Mathematics

Board Paper 2012 - Solution

13.

1 4851
Given volume of a hemisphere = 2425 cm3  cm3
2 2
Now, let r be the radius of the hemisphere
2
Volume of a hemisphere  r3
3
2 3 4851
 r 
3 2
2 22 3 4851
  r 
3 7 2
3
4851 3 7  21 
 r3      
2 2 22  2 
21
 r  cm
2
So, Curved surface area of the hemisphere  2r2
11 3
22 21 21
 2  x x  693 sq.cm
7 2 2

14. Given: Tangents PA and PB are drawn from an external point P to two
concentric circles with centre O and radii OA = 8 cm, OB = 5 cm
respectively. Also, AP = 15 cm

Construction: We join the points O and P.

Solution: OA ⊥ AP ; OB ⊥ BP

[Using the property that radius is perpendicular to the


tangent at the point of contact of a circle]

In right angled triangle OAP,

OP2 = OA2 + AP2 [Using Pythagoras Theorem]

= (8)2 + (15)2 = 64 + 225 = 289

∴ OP = 17 cm

In right angled triangle OBP,

OP2 = OB2 + BP2

⇒ BP2 = OP2 - OB2 = (17)2 – (5)2 = 289 – 25 = 264

∴ BP = 264  2 66 cm

www.topperlearning.com 6
CBSE X | Mathematics

Board Paper 2012 - Solution

15. Given: ABC is an isosceles triangle, where AB = AC, circumscribing a circle.

To prove: The point of contact P bisects the base BC. i.e. BP = PC

Proof: It can be observed that

BP and BR; CP and CQ; AR and AQ are pairs of tangents drawn to the circle
from the external points B, C and A respectively.

Since the tangents drawn from an external point to a circle, then

BP = BR --- (i)

CP = CQ --- (ii)

AR = AQ --- (iii)

Given that AB = AC

⇒ AR + BR = AQ + CQ

⇒ BR = CQ [from (iii)]

⇒ BP = CP [from (i) and (ii)]

∴ P bisects BC.

OR

Given: The chord AB of the larger of the two concentric circles, with centre O,
touches the smaller circle at C.

To prove: AC = CB

Construction: Let us join OC.

Proof: In the smaller circle, AB is a tangent to the circle at


the point of contact C.

∴ OC ⊥ AB ------ (i)

(Using the property that the radius of a circle is perpendicular to the tangent
at the point of contact)

For the larger circle, AB is a chord and from (i) we have OC ⊥ AB

∴ OC bisects AB

(Using the property that the perpendicular drawn from the centre to a chord
of a circle bisects the chord)

∴ AC = CB

www.topperlearning.com 7
CBSE X | Mathematics

Board Paper 2012 - Solution

16. Given, OABC is a square of side 7 cm

i.e. OA = AB = BC = OC = 7cm

∴ Area of square OABC = (side)2 = 72 = 49 sq.cm

Given, OAPC is a quadrant of a circle with centre O.

∴ Radius of the sector = OA = OC = 7 cm.

Sector angle = 90o

90o
 Area of quadrant OAPC  x r2
o
360
1 22 2 77
 x x 7   sq.cm  38.5 sq.cm
4 7 2
 Area of shaded portion  Area of Square OABC  Area of quadrant OAPC
  49  38.5 sq. cm  10.5 sq.cm

17. First three- digit number that is divisible by 7 = 105

Next number = 105 + 7 = 112

Therefore the series is 105, 112, 119,…

The maximum possible three digit number is 999.

When we divide by 7, the remainder will be 5.

Clearly, 999 – 5 = 994 is the maximum possible three – digit number


divisible by 7.

The series is as follows:

105, 112, 119, …., 994

Here a = 105, d = 7

Let 994 be the nth term of this A.P.

www.topperlearning.com 8
CBSE X | Mathematics

Board Paper 2012 - Solution

an  a  n  1 d
 994  105  n  1 7
 n  1 7  889
 n  1  127
 n  128

So, there are 128 terms in the A.P.

n
 Sum  first term  last term
2
128
 a1  a128 
2
 64 105  994   64  1099   70336

18. Given quadratic equation is 3x2 – 2kx + 12 = 0

Here a = 3, b = -2k and c = 12

The quadratic equation will have equal roots if ∆ = 0

 b2  4ac  0
Putting the values of a,b and c we get

 2k 2  4 3 12   0


 4k2  144  0
 4k2  144
144
 k2   36
4
Considering square root on both sides,
k  36   6

Therefore, the required values of k are 6 and -6.

www.topperlearning.com 9
CBSE X | Mathematics

Board Paper 2012 - Solution

SECTION – C

19.

Let the co-ordinates of point P be (x, y)

Then using the section formula co-ordinates of P are.

4K  3 8K  5
x y
K 1 K 1

Since P lies on x+y=0

4K  3 8K  5
  0
K 1 K 1

2 1
 4K  2  0  k  K
4 2
1
Hence the value of K  .
2

20. The area of a triangle, whose vertices are (x1, y1), (x2, y2)
and (x3, y3) is

1
| x1 (y2 y3 ) x2 (y3 y1) x3 (y1 y2 ) |
2

Substituting the given coordinates

1
Area of |1(p 7) 4(7 3) ( 9)( 3 p) |
2

1
|(p 7) 40 27 9p | 15
2
10p 60 30
10p 30 or 10p 90
p 3. or p 9

www.topperlearning.com 10
CBSE X | Mathematics

Board Paper 2012 - Solution

21. Let ABCD be a parallelogram such that its sides touching a circle with centre O.
We know that the tangents to a circle from an exterior point are equal in
length.

AP = AS [From A] …(i)

BP = BQ [From B] …(ii)

CR = CQ [From C] …(iii)

and, DR = DS [From D] …(iv)

Adding (i), (ii), (iii) and (iv), we get

AP + BP + CR + DR = AS + BQ + CQ + DS

(AP + BP) + (CR + DR) = (AS + DS) + (BQ + CQ)

AB + CD = AD + BC

2AB = 2BC [ ABCD is a parallelogram AB=CD and BC = AD]

AB=BC

Thus, AB = BC = CD = AD

Hence, ABCD is a rhombus.

www.topperlearning.com 11
CBSE X | Mathematics

Board Paper 2012 - Solution

OR

A circle with centre O touches the sides AB, BC, CD, and DA of a
quadrilateral ABCD at the points P, Q, R and S respectively.

TO PROVE AOB COD 180o and, AOD BOC 180o

CONSTRUCTION Join OP, OQ, OR and OS.

PROOF Since the two tangents drawn from a external point to a circle
subtend equal angles at the centre.

1 2, 3 4, 5 6 and 7 8
Now, 1 2 3 4 5 6 7 8 360o

Sum of all the angles


subtended at a point is 360o

2( 2 3 6 7) 360o and 2( 1 8 4 5) 360o


( 2 3) ( 6 7) 180o and ( 1 8) ( 4 5) 180o

2 3 AOB, 6 7 COD
1 8 AOD and 4 5 BOC

AOB COD 180o


and AOD BOC 180o

Hence Proved

www.topperlearning.com 12
CBSE X | Mathematics

Board Paper 2012 - Solution

22. Given: radius of cyl=radius of cone=r=6cm

Height of the cylinder=height of the cone=h=7cm

Slant height of the cone= l

72 62 85 cm

Total surface area of the remaining solid

=curved surface area of the cylinder + area of the base of the cylinder +
curved surface area of the cone

(2 rh r2 rl)
22 22 22
2x 6 7 62 6 85
7 7 7
792 132
264 85
7 7
132
377.1 85 cm2
7

OR

Volume of the conical heap=volume of the sand emptied from the bucket.

Volume of the conical heap=

1 2 1 2
r h r x24 cm2 (height of the coneis 24)----------(1)
3 3
Volume of the sand in the bucket= r 2h
(18)2 32cm2 (2)
Equating 1 and 2
1 2
r x24 (18)2 32
3
(18)2 x32x3
r2
24
r 36cm

www.topperlearning.com 13
CBSE X | Mathematics

Board Paper 2012 - Solution

23. Area of the shaded region= Area of sector POQ - Area of sector AOB

Area of Shaded region ( R2 r2 )


360 360
30 22
x x(72 3.52 )
360 7
77
cm2
8

24.

4x2 4ax (a2 b2 ) 0


(4x2 4ax a2 ) b2 0
(2x)2 2.2x.a a2 b2 0

(2x a)2 b2 0

(2x a) b (2x a) b 0
(2x a) b 0 or (2x a) b 0
a b a b
x ;x
2 2

OR

3x2  2 6 x  2  0
 3x2  6 x  6 x 2 0
 3 x  3 x  2  2  3 x  2  0
   
  3x 2  3x 2 0 
 2
2
 3x  0

 3x 2  0
 3 x 2
2 2  3 6
 x   
 3
3 2 3

www.topperlearning.com 14
CBSE X | Mathematics

Board Paper 2012 - Solution

25. Given: Position of kite is B.

Height of kite above ground= 45 m

Angle of inclination = 60o

Required length of string = AB

In right angled triangle AOB,

OB 45 3 45
sin A   sin 60o   
AB AB 2 AB
45 x 2 90
 AB    30 3 m
3 3
Hence, the length of the string is 30 3 m

26. It is given that A = 105, C = 30.

Using angle sum property of triangle, we get, B = 45

The steps of construction are as follows:

1. Draw a line segment BC = 6 cm.

2. At B, draw a ray making an angle of 45 with BC.

3. At C, draw a ray making an angle of 30 with BC. Let the two rays meet at
point A.

4. Below BC, make an acute ∠CBX. Along BX mark off three points B1, B2, B3,
such that BB1 = B1B2 = B2B3. Join B3C.

7. From B2, draw B2C || B3C.

8. From C, draw CA || CA, meeting BA at the point A.

Then ABC is the required triangle.

www.topperlearning.com 15
CBSE X | Mathematics

Board Paper 2012 - Solution

27. Let a and d respectively be the first term and the common difference of the
AP.

We know that the nth term of an AP is given by an = a + (n – 1)d

According to the given information,

a16 = 1 + 2 a8

 a + (16 – 1)d = 1 + 2[a + (8 – 1)d]

 a + 15d = 1 + 2a + 14d

 a + 15d = 1 + 2a + 14d

 –a + d = 1 … (1)

Also, it is given that, a12 = 47

 a + (12 – 1)d = 47

 a + 11d = 47 … (2)

Adding (1) and (2), we have:

12d = 48

d=4

From (1),

–a + 4 = 1  a = 3

Hence, an = a + (n – 1)d = 3 + (n – 1)(4) = 3 + 4n – 4 = 4n – 1

Hence, the nth term of the AP is 4n – 1.

www.topperlearning.com 16
CBSE X | Mathematics

Board Paper 2012 - Solution

28. Total number of outcomes = 52

(i) Probability of getting a red king

Here the number of favourable outcomes = 2

No. of favourable outcomes 2 1


Probability
Total number of outcomes 52 26

(ii) Probability of getting a face card

Total number of face cards = 12

No. of favourable outcomes 12 3


Probability
Total number of outcomes 52 13

(iii) Probability of queen of diamonds

Number of queens of diamond = 1

No. of favourable outcomes 1


Probability
Total number of outcomes 52

SECTION – D

29. Here, R = 28 cm and r = 21 cm, we need to find h.

Volume of frustum = 28.49 L = 28.49×1000 cm3 = 28490 cm3

Now, Volume of frustum =


3

h 2
R  Rr  r2 

22h
73
 
282  28  21  212 =28490

22
 h  1813  28490
21
28490  21
h  15 cm
22  1813

Hence the height of bucket is 15 cm.

www.topperlearning.com 17
CBSE X | Mathematics

Board Paper 2012 - Solution

30. Let the height of hill is h.

In right triangle ABC,

50 50 1
 tan30    AB  50 3
AB AB 3

In right triangle ABD,

h h
 tan60   3  h  3AB
AB AB

 
 h  3 50 3  150 m

Hence the height of hill is 150 m.

31. Given: AB is a tangent to a circle with centre O.

To prove: OP is perpendicular to AB.

Construction: Take a point Q on AB and join OQ.

Proof: Since Q is a point on the tangent AB,


other than the point of contact P, so Q will
be outside the circle.

Let OQ intersect the circle at R.

Now OQ = OR + RQ

 OQ > OR  OQ > OP [as OR = OP]

 OP < OQ

Thus OP is shorter than any other segment among all and the shortest length
is the perpendicular from O on AB.

 OP  AB. Hence proved.

www.topperlearning.com 18
CBSE X | Mathematics

Board Paper 2012 - Solution

OR

Let ABCD be a quadrilateral, circumscribing a circle.

Since the tangents drawn to the circle from an external


point are equal, we have

AP = AS ... (1)

PB = BQ ... (2)

RC = QC ... (3)

DR = DS ... (4)

Adding, (1), (2), (3) and (4), we get

AP + PB + RC + DR = AS + BQ + QC + DS

(AP + PB) + (DR + RC) = (AS + SD) + (BQ + QC)

AB + CD = AD + BC.

32. Total cost of books = Rs 80

Let the number of books = x

80
So the cost of each book = Rs
x

80
Cost of each book if he buy 4 more book = Rs
x4

As per given in question:

80 80
 1
x x4
80x  320  80x
 1
x(x  4)
320
 2 1
x  4x
 x2  4x  320  0
 (x  20)(x  16)  0
 x  20,16

Since number of books cannot be negative,

So the number of books he brought is 16.


www.topperlearning.com 19
CBSE X | Mathematics

Board Paper 2012 - Solution

OR

Let the first number be x then the second number be 9 – x as the sum of
both numbers is 9.

1
Now the sum of their reciprocal is , therefore
2

1 1 1
 
x 9x 2
9xx 1
 
x(9  x) 2
9 1
 2

9x  x 2
 18  9x  x2
 x2  9x  18  0
 (x  6)(x  3)  0
 x  6,3

If x = 6 then other number is 3.

And, if x = 3 then other number is 6.

Hence numbers are 3 and 6.

33. Given: S20  240 and a  7

Consider, S20  240

20  n 
 2  7  19d  240  Sn  2 2a  (n  1)d
2  
 10(14  19d)  240
 14  19d  24
 19d  38
 d  2
Now, a24  a  23d  7  23  2  39
Hence, a24  39

www.topperlearning.com 20
CBSE X | Mathematics

Board Paper 2012 - Solution

34. Radius of hemi-sphere = 7 cm

Radius of cone = 7 cm

Height of cone = diameter = 14 cm

Volume of solid = Volume of cone + Volume of Hemi-sphere

1 2 2
 r h  r3
3 3
1 2
 r h  2r 
3
1 22
   49 14  14 
3 7
1 22
   49  28
3 7
22  7  28 4312
  cm3
3 3

Radius of cylinder = Radius of cone = r = 6 cm

Height of the cylinder = Height of the cone = h = 8 cm

Slant height of the cone= l 82 62 100 10 cm

Total surface area of the remaining solid

= Curved Surface Area of the Cylinder + Area of the Base of the Cylinder +
Curved Surface Area of the Cone

(2 rh r2 rl)
22 22 2 22
2x x6x7 x6 x6x 85
7 7 7
792 132
264 85
7 7
132
377.1 85cm2
7

www.topperlearning.com 21
CBSE X | Mathematics

Board Paper 2013 - Solution

CBSE Board
Class X Mathematics
Board Paper - 2013
Time: 3 hours Total Marks: 90

Section A

1. Correct answer: C

Let AB be the tower of height 75 m and C be the position of the car.

In ABC,

AC
cot 30o
AB
AC AB cot 30o
AC 75m 3
AC 75 3m

Thus, the distance of the car from the base of the tower is 75 3 m.

2. Correct answer: A

S = {1, 2, 3, 4, 5, 6}

Let event E be defined as 'getting an even number'.

n(E) = {1, 4, 6}

Number of favourable outcomes 3 1


P E
Number of possible outcomes 6 2

www.topperlearning.com 1
CBSE X | Mathematics

Board Paper 2013 - Solution

3. Correct answer: C

S = {1, 2, 3,..90}

n(S) = 90

The prime number less than 23 are 2, 3, 5, 7, 11, 13, 17, and 19.

Let event E be defined as 'getting a prime number less than 23'.

n(E) = 8

Number of favourable outcomes 8 4


P E
Number of possible outcomes 90 45

4. Correct answer: A

Given: AB, BC, CD and AD are tangents to the circle


with centre O at Q, P, S and R respectively.

AB = 29 cm, AD = 23, DS = 5 cm and B = 90o

Construction: Join PQ.

We know that, the lengths of the tangents drawn from an external point to a
circle are equal.

DS = DR = 5 cm

AR = AD - DR = 23 cm - 5 cm = 18 cm

AQ = AR = 18 cm

QB = AB - AQ = 29 cm - 18 cm = 11 cm

QB = BP = 11 cm

In PQB,

PQ2 = QB2 + BP2 = (11 cm)2 + (11 cm)2 = 2 (11 cm)2

PQ = 11 2 cm ... (1)

In OPQ,

PQ2 = OQ2 + OP2 = r2 + r2 = 2r2

(11 2 )2 = 2r2

121 = r2

www.topperlearning.com 2
CBSE X | Mathematics

Board Paper 2013 - Solution

r = 11

Thus, the radius of the circle is 11 cm.

5. Correct answer: B

AP PB (Given)

CA AP, CB BP (Since radius is perpendicular to tangent)

AC = CB = radius of the circle

Therefore, APBC is a square having side equal to 4 cm.

Therefore, length of each tangent is 4 cm.

6. Correct answer: C

From the figure, the coordinates of A, B, and C are (1, 3), (-1, 0) and (4, 0)

respectively.

Area of  ABC

1
 | 1(0  0)  (1)(0  3)  4(3  0) |
2
1
 | 0  3  12 |
2
1
 | 15 |
2
 7.5 sq units

www.topperlearning.com 3
CBSE X | Mathematics

Board Paper 2013 - Solution

7. Correct answer: B

Let r be the radius of the circle.

From the given information, we have

2 r - r = 37 cm

r 2 1 37 cm
22
r 2 1 37 cm
7
37
r 37 cm
7
r 7 cm

22
Circumference of the circle = 2 r 2 7 cm 44 cm
7

8. Correct answer: C

1 6q 1 1 6q 1 6q
Common difference = 2
3q 3q 3q 3q

9. Given: ABCD be a parallelogram circumscribing a circle with centre O.

To prove: ABCD is a rhombus.

We know that the tangents drawn to a circle from an


exterior point are equal in length.

Therefore, AP = AS, BP = BQ, CR = CQ and DR = DS.

Adding the above equations,

AP + BP + CR + DR = AS + BQ + CQ + DS

(AP + BP) + (CR + DR) = (AS + DS) + (BQ + CQ)

AB + CD = AD + BC

2AB = 2BC

(Since, ABCD is a parallelogram so AB = DC and AD = BC)

AB = BC

Therefore, AB = BC = DC = AD.

Hence, ABCD is a rhombus.

www.topperlearning.com 4
CBSE X | Mathematics

Board Paper 2013 - Solution

10. Dimension of the rectangular card board = 14 cm 7 cm

Since, two circular pieces of equal radii and maximum area touching each
other are cut from the rectangular card board, therefore, the diameter of each
14
of each circular piece is = 7 cm.
2

7
Radius of each circular piece = cm.
2

2
7 22 49
Sum of area of two circular pieces = 2 2 77cm2
2 7 4

Area of the remaining card board

= Area of the card board - Area of two circular pieces

= 14 cm 7 cm - 77 cm2

= 98 cm2 - 77 cm2

= 21 cm2

11. Given: AB = 12 cm, BC = 8 cm and AC = 10 cm.

Let, AD = AF = x cm, BD = BE = y cm and CE = CF = z cm

(Tangents drawn from an external point to the circle are equal in length)

2(x + y + z) = AB + BC + AC = AD + DB + BE + EC + AF + FC = 30 cm

x + y + z = 15 cm

AB = AD + DB = x + y = 12 cm

 z = CF = 15 - 12 = 3 cm

AC = AF + FC = x + z = 10 cm

 y = BE = 15 - 10 = 5 cm

 x = AD = x + y + z - z - y = 15 - 3 - 5 = 7 cm

www.topperlearning.com 5
CBSE X | Mathematics

Board Paper 2013 - Solution

12. Three digit numbers divisible by 7 are

105, 112, 119, … 994

This is an AP with first term (a) = 105 and common difference (d) = 7

Let an be the last term.

an = a + (n - 1)d

994 = 105 + (n - 1)(7)

7(n - 1) = 889

n - 1 = 127

n = 128

Thus, there are 128 three-digit natural numbers that are divisible by 7.

13.

4 3x2 5x 2 3 0
2
4 3x 8x 3x 2 3 0
4x 3x 2 3 3x 2 0

4x 3 3x 2 0

3 2
x or x
4 3

14. Let E be the event that the drawn card is neither a king nor a queen.

Total number of possible outcomes = 52

Total number of kings and queens = 4 + 4 = 8

Therefore, there are 52 - 8 = 44 cards that are neither king nor queen.

Total number of favourable outcomes = 44

Favourable outcomes 44 11
Required probability = P(E) =
Total number of outcomes 52 13

www.topperlearning.com 6
CBSE X | Mathematics

Board Paper 2013 - Solution

15. Let the radius and height of cylinder be r cm and h cm respectively.

Diameter of the hemispherical bowl = 14 cm

Radius of the hemispherical bowl = Radius of the cylinder

14
=r cm 7cm
2

Total height of the vessel = 13 cm

Height of the cylinder, h = 13 cm - 7 cm = 6 cm

Total surface area of the vessel = 2(curved surface area of the cylinder +
curved surface area of the hemisphere)

(Since, the vessel is hollow)

22
2 2 rh 2 r2 4 r h r 4 7 6 7 cm2
7

= 1144 cm2

16. Height of the cylinder, h = 10 cm

Radius of the cylinder = Radius of each hemisphere = r = 3.5 cm

Volume of wood in the toy = Volume of the cylinder - 2


Volume of each hemisphere

2 3
r2h 2 r
3
4
r2 h r
3
22 4
(3.5)2 10 3.5
7 3
38.5 10 4.67
38.5 5.33
205.205 cm3

Radius = 21 cm

www.topperlearning.com 7
CBSE X | Mathematics

Board Paper 2013 - Solution

17. The arc subtends an angle of 60o at the centre.

60o 22
(i) l = o
2 r o
2 21 cm 22 cm
360 360 7

60o 22
(ii) Area of the sector o
r2 o
21 21 cm2 = 231 cm2
360 360 7

18. AB and CD are the diameters of a circle with centre O.

OA = OB = OC = OD = 7 cm (Radius of the circle)

Area of the shaded region

= Area of the circle with diameter OB + (Area of the semi-circle ACDA - Area
of ACD)

2
7 1 2 1
7 CD OA
2 2 2
22 49 1 22 1
49 14 7
7 4 2 7 2

77
77 49
2
66.5 cm2

19. Let the y-axis divide the line segment joining the points (-4, -6) and (10, 12)
in the ratio k: 1 and the point of the intersection be (0, y).

Using section formula, we have

10k 4 12k 6
, 0, y
k 1 k 1
10k 4 4 2
0 10k 4 0 k
k 1 10 5
Thus, the y-axis divides the line segment joining the
given points in the ratio 2:5.
2 24 30
12k 6 12 6
5 5 6
y
k 1 2 2 5 7
1
5 5

6
Thus, the coordinates of the point of division are 0, .
7

www.topperlearning.com 8
CBSE X | Mathematics

Board Paper 2013 - Solution

20.

Let AB and CD be the two poles, where CD (the second pole) = 24 m.

BD = 15 m

Let the height of pole AB be h m.

AL = BD = 15 m and AB = LD = h

So, CL = CD - LD = 24 – h

In ACL,

CL
tan30o
AL
24 h
tan30o
15
1 24 h
3 15
15
24 h 5 3
3
h 24 5 3
h 24 5 1.732 Taking 3 1.732
h 15.34

Thus, height of the first pole is 15.34 m.

www.topperlearning.com 9
CBSE X | Mathematics

Board Paper 2013 - Solution

21. k + 4)x2 + (k + 1)x + 1 = 0

a = k + 4, b = k + 1, c = 1

For equal roots, discriminant, D = 0

b2 - 4ac = 0

(k + 1)2 - 4(k + 4) 1=0

k2 + 2k + 1 - 4k - 16 = 0

k2 - 2k - 15 = 0

k2 - 5k + 3k - 15 = 0

k(k - 5) + 3(k - 5) = 0

(k - 5) (k + 3) = 0

k = 5 or k = -3

Thus, for k = 5 or k = -3, the given quadratic equation has equal roots.

22. Sn = 3n2 + 4n

First term (a1) = S1 = 3(1)2 + 4(1) = 7

S2 = a1 + a2 = 3(2)2 + 4(2) = 20

a2 = 20 - a1 = 20 - 7 = 13

So, common difference (d) = a2 - a1 = 13 - 7 = 6

Now, an = a + (n - 1)d

 a25 = 7 + (25 - 1) 6 = 7 + 24 6 = 7 + 144 = 151

www.topperlearning.com 10
CBSE X | Mathematics

Board Paper 2013 - Solution

23.

Steps of construction:

1. Draw two concentric circle with centre O and radii 4 cm and 6 cm. Take a
point P on the outer circle and then join OP.

2. Draw the perpendicular bisector of OP. Let the bisector intersects OP at M.

3. With M as the centre and OM as the radius, draw a circle. Let it intersect the
inner circle at A and B.

4. Join PA and PB.

Therefore, PA and PB are the required tangents.

www.topperlearning.com 11
CBSE X | Mathematics

Board Paper 2013 - Solution

24. The given points are A(-2,3) B(8,3) and C(6,7).

Using distance formula, we have:

2 2
AB2 = 8 2 3 3

AB2 = 102 + 0

AB2 = 100

2 2
BC2 = 6 8 7 3

BC2 = (-2)2 + 42

BC2 = 4 + 16

BC2 = 20

2 2
CA2 2 6 3 7

CA = (-8)2 + (-4)2

CA2 = 64 + 16

CA2 = 80

It can be observed that:

BC2 + CA2 = 20 + 80 = 100 = AB2

So, by the converse of Pythagoras Theorem,

ABC is a right triangle right angled at C.

25. Diameter of circular end of pipe = 2 cm

2
Radius r1 of circular end of pipe = m 0.01 m
200

2
Area of cross-section = r12 0.01 0.0001 m2

Speed of water = 0.4 m/s = 0.4 60 = 24 metre/min

Volume of water that flows in 1 minute from pipe =


24 0.0001 m3 0.0024 m3

Volume of water that flows in 30 minutes from pipe =

www.topperlearning.com 12
CBSE X | Mathematics

Board Paper 2013 - Solution

30 0.0024 m3 0.072 m3

Radius (r2) of base of cylindrical tank = 40 cm = 0.4 m

Let the cylindrical tank be filled up to h m in 30 minutes.

Volume of water filled in tank in 30 minutes is equal to the volume of water


flowed out in 30 minutes from the pipe.

r22 h 0.072
2
0.4 h 0.072
0.16 h 0.072
0.072
h
0.16
h 0.45 m 45 cm

Therefore, the rise in level of water in the tank in half an hour is 45 cm.

26. The group consists of 12 persons.

Total number of possible outcomes = 12

Let A denote event of selecting persons who are extremely patient

Number of outcomes favourable to A is 3.

Let B denote event of selecting persons who are extremely kind or honest.

Number of persons who are extremely honest is 6.

Number of persons who are extremely kind is 12 - (6 + 3) = 3

Number of outcomes favourable to B = 6 + 3 = 9.

Number of outcomes favrouableto A 3 1


(i) P A
Total number of possible outcomes 12 4

Number of outcomes favorableto B 9 3


(ii) P B
Total number of possible outcomes 12 4

Each of the three values, patience, honesty and kindness is important in


one's life.

www.topperlearning.com 13
CBSE X | Mathematics

Board Paper 2013 - Solution

27. Diameter of upper end of bucket = 30 cm

Radius (r1) of upper end of bucket = 15 cm

Diameter of lower end of bucket = 10 cm

Radius (r2) of lower end of bucket = 5 cm

Slant height (l) of frustum

2
= r1 r2 h2

2 2 2 2
15 5 24 10 24 100 576
676 26cm

Area of metal sheet used to make the bucket

r1 r2 l r22
2
15 5 26 5

520 25 545 cm2

Cost of 100 cm2 metal sheet = Rs 10

Cost of 545 cm2 metal sheet

545 3.14 10
= Rs. Rs.171.13
100

Therefore, cost of metal sheet used to make the bucket is Rs 171.13.

www.topperlearning.com 14
CBSE X | Mathematics

Board Paper 2013 - Solution

28. Given: l and m are two parallel tangents to the circle with centre O touching
the circle at A and B respectively. DE is a tangent at the point C, which
intersects l at D and m at E.

To prove: DOE 90o

Construction: Join OC.

Proof:

In ODA and ODC,

OA = OC (Radii of the same circle)

AD = DC (Length of tangents drawn from an external point to a circle are


equal)

DO = OD (Common side)

ODA ODC (SSS congruence criterion)

DOA COD ... (1)

Similarly, OEB OEC

EOB COE ... (2)

Now, AOB is a diameter of the circle. Hence, it is a straight line.

DOA COD COE EOB 180o

From (1) and (2), we have:

2 COD + 2 COE = 180o

COD COE 90o


DOE 90o

Hence, proved.

www.topperlearning.com 15
CBSE X | Mathematics

Board Paper 2013 - Solution

29. Let the sides of the two squares be x cm and y cm where x > y.

Then, their areas are x2 and y2 and their perimeters are 4x and 4y.

By the given condition:

x2 + y2 = 400 … (1)

and 4x - 4y = 16

4(x - y) = 16 x-y=4

x=y+4 ... (2)

Substituting the value of x from (2) in (1), we get:

(y + 4)2 + y2 = 400

y2 + 16 + 8y + y2 = 400

2y2 + 16 + 8y = 400

y2 + 4y - 192 = 0

y2 + 16y - 12y - 192 = 0

y(y + 16) - 12 (y + 16) = 0

(y + 16) (y - 12) = 0

y = -16 or y = 12

Since, y cannot be negative, y = 12.

So, x = y + 4 = 12 + 4 = 16

Thus, the sides of the two squares are 16 cm and 12 cm.

www.topperlearning.com 16
CBSE X | Mathematics

Board Paper 2013 - Solution

30.

1 1 1 1
2a b
2x 2a b 2x
1 1 1 1
2a b 2x 2x 2a b
2x 2a b 2x b 2a
2x 2a b 2x 2ab
2a b b 2a
2x 2a b 2x 2ab

1 1
x 2a b 2x ab
2x2 2ax bx ab 0
2x x a b x a 0
x a 2x b 0
x a 0 or 2x b 0
b
x a, or x
2

31. Given: A circle with centre O and a tangent XY to the circle at a point P

To Prove: OP is perpendicular to XY.

Construction:
Take a point Q on XY other than P and join
OQ.

Proof:
Here the point Q must lie outside the circle as
if it lies inside the tangent XY will become secant to the circle.

Therefore, OQ is longer than the radius OP of the circle, That is, OQ > OP.
This happens for every point on the line XY except the point P.

So OP is the shortest of all the distances of the point O to the points on XY.

And hence OP is perpendicular to XY.

Hence, proved.

www.topperlearning.com 17
CBSE X | Mathematics

Board Paper 2013 - Solution

32. Given A.P. is -12, -9, -6, ..., 21

First term, a = -12

Common difference, d = 3

Let 21 be the nth term of the A.P.

21 = a + (n - 1)d

21 = -12 + (n - 1) 3

33 = (n - 1) 3

n = 12

Sum of the terms of the A.P. = S12

n 12
= 2a n 1 d 24 11 3 54
2 2

If 1 is added to each term of the AP, the sum of all the terms of the new AP
will increase by n, i.e., 12.

 Sum of all the terms of the new AP = 54 + 12 = 66

33. Let AC and BD be the two poles of the same height h m.

Given AB = 80 m

Let AP = x m, therefore, PB = (80 - x) m

In  APC,

AC
tan30o
AP
1 h
... (1)
3 x

In  BPD,

BD
tan60o
AB
h
3 ... (2)
80 x

www.topperlearning.com 18
CBSE X | Mathematics

Board Paper 2013 - Solution

Dividing (1) by (2),


1 h
3 x
3 h
80 x
1 80 x
3 x
x 240 3x
4x 240
x 60
From (1),
1 h
3 x
60
h 20 3m
3

Thus, the height of both the poles is 20 3m and the distances of the point
from the poles are 60 m and 20 m.

34. The given vertices are A(x, y), B(1, 2) and C(2, 1).

It is know that the area of a triangle whose vertices are (x1, y1), (x2, y2) and
(x3, y3) is given by

1
| x1 y2 y3 x2 y3 y1 x3 y1 y2 |
2

 Area of ABC

1
|x 2 1 1 1 y 2 y 2 |
2
1
| x 1 y 2y 4 |
2
1
| x y 3|
2

The area of ABC is given as 6 sq units.

1
x y 3 6 x y 3 12
2
x y 15

www.topperlearning.com 19
CBSE X | MATHEMATICS

Board Paper ˗ 2014

CBSE Board
Class X Summative Assessment – II
Mathematics
Board Question Paper 2014
Time: 3 hrs Max. Marks: 90

Solution
Section A

1. Correct answer: C

The multiples of 4 between 1 and 15 are 4, 8 and 12. The probability of getting a
3 1
multiple of 4 = 
15 5

2. Correct answer: A

Let AB be the tower and BC be distance between tower and car. Let  be the angle of
depression of the car.

According to the given information,

In ABC,
BC 1
tan   [Using (1)] and tan 30 =
AB 3

150 150 3
 BC =   50 3
3 3

Hence, distance between the tower and car is 50 3 .

www.topperlearning.com 1
CBSE X | MATHEMATICS

Board Paper ˗ 2014

3. Correct answer: D

TA  TP  TAP  TPA
TB  TP  TBP  TPB
TAP  TBP=TPA +TPB=APB
TAP  TBP  APB  180  sum of.....180
APB  APB  180
2APB  180
APB  90

4. Correct answer: B

k, 2k - 1, 2k + 1 are in Arithmetic Progression

2  (2k - 1) = k + 2k +1

4k - 2 = 3k +1

k=3

www.topperlearning.com 2
CBSE X | MATHEMATICS

Board Paper ˗ 2014

5. Correct answer: B

Given  AOB is given as 90 

ΔAOB is an isosceles triangle since OA = OB

Therefore  OAB =  OBA = 45 

Thus  AOP = 45  and  BOP = 45 

Hence ∆AOP and ∆BOP also are isosceles triangles

Thus let AP = PB = OP = x

Using Pythagoras theorem

x 2 + x 2 = 102

Thus 2 x 2 = 100

x =5 2

Hence length of chord AB = 2 x = 10 2

www.topperlearning.com 3
CBSE X | MATHEMATICS

Board Paper ˗ 2014

6. Correct answer: A

We see that AB = 4 units and BC = 3 units

Using Pythagoras theorem

AC2  AB2  BC2

= 42  32

AC2  25

Thus AC = 5 units
Hence length of the diagonal of the rectangle is 5 units

www.topperlearning.com 4
CBSE X | MATHEMATICS

Board Paper ˗ 2014

7. Correct answer: C

It is given that AB = 5 and BC = 12

Using Pythagoras theorem

AC2  AB2  BC2

= 52  122

= 169

Thus AC = 13

We know that two tangents drawn to a circle from the same point that is exterior to
the circle are of equal lengths.
Thus AM = AQ = a

Similarly MB = BP = b and PC = CQ = c

We know AB = a + b = 5

BC = b + c = 12 and AC = a + c = 13

Solving simultaneously we get a=3, b=2 and c=10

We also know that the tangent is perpendicular to the radius

Thus OMBP is a square with side b

Hence the length of the radius of the circle inscribed in the right angled triangle is 2
cm.

www.topperlearning.com 5
CBSE X | MATHEMATICS

Board Paper ˗ 2014

8. Correct answer: A

There are in all 23 = 8 combinations or outcomes for the gender of the 3 children

The eight combinations are as follows

BBB, BBG, BGB, BGG, GBB, GBG, GGB, GGG

7
Thus the probability of having at least one boy in a family is
8

SECTION B

9. Solution:

Given: AB and CD are common tangents to both the circles.

To prove: AB = CD

Proof:

We know that two tangents drawn to a circle for the same exterior point are equal.

Thus we get AE = EC

Similarly ED = EB

AB = AE + EB and CD = ED + EC

Since AE = EC we can write AB = EC + EB

And since ED = EB we get CD = EB + EC

Therefore AB = CD

Hence proved.

www.topperlearning.com 6
CBSE X | MATHEMATICS

Board Paper ˗ 2014

10.

Given:  ABC is an isosceles triangle with a circle inscribed in the triangle.

To prove: BD = DC

Proof:

AF and AE are tangents drawn to the circle from point A.

Since two tangents drawn to a circle from the same exterior point are equal,
AF = AE = a

Similarly BF = BD = b and CD = CE = c

We also know that  ABC is an isosceles triangle

Thus AB = AC,

a+b=a+c

Thus b = c

Therefore, BD = DC

Hence proved.

www.topperlearning.com 7
CBSE X | MATHEMATICS

Board Paper ˗ 2014

11. Solution:

The total number of outcomes when two dice are tossed together is 36.

The sample space is as follows

i. Favourable outcomes = { (2,2) (2,4) (2,6) (4,2) (4,4) (4,6) (6,2) (6,4) (6,6) }

Probability that the number on each dice is even

Number of favourable outcomes 9 1


=  
Total number of outcomes 36 4
ii. Favouable outcomes = { (1,4) (2,3) (3,2) (4,1) }

Probability that the sum of the numbers appearing on the two dice is 5

Number of favourable outcomes 4 1


=  
Total number of outcomes 36 9

12. Solution:

Given total surface area of hemisphere = 462 cm2


2r2  462
r  8.574cm
2
Volume of a hemisphere  r3
3
2 3 2 22
 r    8.5743  1320.54
3 3 7
3
4851 3 7  21 
 r3    
2 2 22  2 

www.topperlearning.com 8
CBSE X | MATHEMATICS

Board Paper ˗ 2014

13. Solution:

Numbers which are divisible by both 2 and 5 are the numbers which are divisible by
10.

Thus we need to find the number of natural numbers between 101 and 999 which
are divisible by 10.

The first number between 101 and 999 which is divisible by 10 is 110

And the last number between 101 and 999 which is divisible by 10 is 990

Using the formula for arithmetic progression where first term ( a ) = 110, last term
( Tn ) = 990 and difference (d) =10

Tn  a  (n  1)d
990  110  (n  1)10
880  (n  1)10
88  n  1
n  89

Hence there are 89 natural numbers between 101 and 999 which are divisible by
both 2 and 5.

14. Solution:

Given: Quadratic equation 9x2  3kx  k  0 has equal roots

Let β be the equal roots of the equation

3k k
Thus 2β =  (Sum of the roots is equal to –b/a)
9 3
k
We get  
6

k
And we also know that  2 = (Product of the roots is equal to c/a)
9

k2 k

36 9
k 1
For k  0, 
36 9

Thus k = 4

www.topperlearning.com 9
CBSE X | MATHEMATICS

Board Paper ˗ 2014

SECTION – C

15. Solution:

Let P and Q be the two positions of the plane and A be the point of observation. Let
ABC be the horizontal line through A.

It is given that angles of elevation of the plane in two positions P and Q from a point
A are 60° and 30° respectively.

∴ ∠PAB = 60°, ∠QAB = 30°. It is also given that PB = 3000√3 m meters

In ΔABP, we have

Tan 60 = BP/AB

Root 3 = 3000√3/ AB

AB = 3000 m

In ΔACQ, we have

tan30 = CQ/AC

1/√3 = 3000√3/ AC

AC = 9000 m

∴ Distance = BC = AC – AB = 9000m – 3000m = 6000m

Thus, the plane travels 6km in 30 seconds

Hence speed of plane = 6000/30 = 200 m/sec = 720km/h

www.topperlearning.com 10
CBSE X | MATHEMATICS

Board Paper ˗ 2014

16. Solution:

Diameter of sphere curved out = side of cube = 7cm ∴ r = 3.5 cm

Volume of cube = a3

= 73

= 343 cm3

Volume of sphere curved out = 4/3  r3

= 4/3 × 22/7 × 7/2 × 7/2 × 7/2

= 179.66 cm3

Volume of of wood left = 343 - 179.66 = 163.34 cm3

17. Solution:

Speed = 4km/h = 200/3 m/min

Volume of water irrigate in 10 min = 10 × 6 × 1.5 × 200/3 = 6000m3

Volume of water irrigated = base area (of irrigated land) x height = base area x 8cm
= base area x 0.08m

6000 = base area x 0.08

Base area = 6000/0.08 =75000 m2 = 7.5 hectare

www.topperlearning.com 11
CBSE X | MATHEMATICS

Board Paper ˗ 2014

18.

Solution:

Area of trapezium = 24.5 cm2

½ [AD + BC] × AB = 24.5 cm2

½ [10+4] × AB = 24.5

AB = 3.5 cm

r = 3.5 cm

Area of quadrant = ¼ × pi × r2 = 0.25 × 22/7 × 3.5 × 3.5 = 9.625 cm2

The area of shaded region = 24.5 - 9.625 = 14.875 cm2

www.topperlearning.com 12
CBSE X | MATHEMATICS

Board Paper ˗ 2014

19. Solution:

Point p lies on x axis so it’s ordinate is 0 (Using section formula)

Let the ratio be k: 1 Let the coordinate of the point be P(x , 0)

As given A(3,-3) and B(-2,7)

Py = (my2 + ny1) /(m + n)

0 = (k × 7 +1 × -3) / (k+1)

0(k + 1) = 7k -3

0 = 7k - 3

3 = 7k

k = 3 /7

k:1=3:7

x = (mx2 + nx1) /(m + n) = [(3/7 × -2 )+(1 × 3)]/(3/7 + 1) = 1.5

www.topperlearning.com 13
CBSE X | MATHEMATICS

Board Paper ˗ 2014

20. Solution:

The area of shaded region = Area of ring – Area of ABCD

= [(R2 – r2) ] - [ (R2 – r2)]×(θ/360)

= [ (R2 – r2) ] [1 –(θ/360)]

= [22/7 (422 – 212) ] [1 –(60/360)] = 3465 cm2

21. Solution:
(16/x)-1 = 15/(x+1)

(16 - x )/x = 15/(x+1)

15 x = 16x + 16 – x2 – x

16 = x2

x=4

22. Solution:
The sum of 2nd and the 7th terms of an AP is 30

a + d + a + 6 d = 30

2a + 7d = 30

15th term is 1 less than twice the 8th term

a + 14d = 2(a + 7d) - 1

a + 14d = 2a + 14d - 1

a=1

Now, 2 × 1 + 7d = 30
d=4

AP : 1,5,9 ………

www.topperlearning.com 14
CBSE X | MATHEMATICS

Board Paper ˗ 2014

23. Solution:

Find midpoint of AB draw the circle

24. Solution:

AC2 = (5-2)2 + (6 +1)2 = 9 + 49 =58 sq. unit

BD2 = (5-2)2 + (-1 - 6)2 =9 + 49 =58 sq. unit

Diagonals of parallelogram are equal so rectangle

SECTION D

25. Given : A circle C (0, r) and a tangent l at point A.

To prove : OA ⊥ l

www.topperlearning.com 15
CBSE X | MATHEMATICS

Board Paper ˗ 2014

Construction: Take a point B, other than A, on the tangent l. Join OB. Suppose OB
meets the circle in C.

Proof: We know that, among all line segment joining the point O to a point on l, the
perpendicular is shortest to l.

OA = OC (Radius of the same circle)

Now, OB = OC + BC.

∴ OB > OC

⇒ OB > OA

⇒ OA < OB

26. Solution:

Volume of 150 spherical marbles, each of diameters 1.4 cm = volume of cylindrical


vessel of diameter 7 cm

150 × 4/3 ×  × 1.4/2 ×1.4/2 × 1.4/2 =  × 7/2 × 7/2 × h

h = 5.6 cm

27. Solution:

Volume of container = 1/3  × h (R2 + r2 + Rr)

= 1/3 × 22/7 × 24[20 × 20 + 8 × 8 + 20 × 8 ]

= 15689.14 cm3

= 15.69 litre
The cost of milk which can completely fill the container at the rate of Rs.21 per liter
= Rs. (21 × 15.69) = Rs. 329.49

www.topperlearning.com 16
CBSE X | MATHEMATICS

Board Paper ˗ 2014

28. Solution:

Let AB is the tower of height h meter and AC is flagstaff of height x meter.

APB = 45˚ and BPC = 60˚

Tan 60 = (x + h)/120

√3 = (x + h)/120

(x + h) = 120√3

x = 120√3 –h

Tan 45˚ = h/120

1 = h/120

h = 120

Therefore height of the flagstaff = 120√3 – 120

= 120(√3 - 1) m

= 87.6 m

www.topperlearning.com 17
CBSE X | MATHEMATICS

Board Paper ˗ 2014

29. Solution:

Let speed of stream = x km/h

Speed f boat in steel water = 18 km/h

Speed f boat in upstream = (18 – x )km/h

Speed f boat in downstream = (18 + x ) km/h

Distance = 24 km

As per question,

24 km /(18 – x ) = 24 km/(18 + x ) + 1

x2 + 48x – 324 = 0
x = 6 or – 54

Hence, the speed of stream = 6 km/h

30. Solution:

Class 1 plant trees = 2 × class 1 x 2 section = 2 × 1 × 2 = 4 × class

= 4 × 1 = 4 trees

Class 2 plant trees =4 × class = 4 × 2 = 8 trees

a=4

d=8

n = 12
S12 = 12/2[2 × 4 + 11 × 4] = 312 trees

31. Solution:

(x-3) / (x-4) +(x -5) /(x-6) = 10/3

[(x -3 )(x - 6) + (x- 4) (x-5) ] /[(x- 4)x (x- 6) ] = 10/3

2[x2 - 9x + 19]/[ x2 - 10x + 24] = 10/3

2x2 - 23x + 63 = 0
x = 7 and 9/2

www.topperlearning.com 18
CBSE X | MATHEMATICS

Board Paper ˗ 2014

32. Solution:
(i) face card are removed from a pack of 52 playing card = 6

Total favorable outcomes = 52 – 6 = 46

Number of all possible outcomes = 26 - 6= 20

P[E] = 20/46 = 0.43

(ii) Number of all possible outcomes a queen = 2

P[E] = 2/46= 1/23

(iii) Number of all possible outcomes an ace = 2

P[E] = 2/46= 1/23

(iv) Number of all possible outcomes = 6

P[E] = 6/46= 3/23

33. Solution:
Let co – ordinate of D (x, y) and D is midpoint of BC

x = (3 + 5)/2 = 4; y = (2 - 2)/2 = 0

Now Area of triangle ABD = ½ {4(-2 – 0) + 3[(0 – (-6)] + 4 [(-6) – (-2)]}

= 0.5 × [ -8 + 18 - 16] = 3 sq unit

and Area of triangle ACD = ½ [5(-6 - 0) + 4(0 - 2) + 4(2 + 6)] = 3 sq unit

Hence, the median AD divides triangle ABC into two triangle of equal area.

www.topperlearning.com 19
CBSE X | MATHEMATICS

Board Paper ˗ 2014

34. Solution:

Let ABCD be a quadrilateral circumscribing a circle centered at O such that it


touches the circle at point P, Q, R, S. Let us

Join the vertices of the quadrilateral ABCD to the center of the circle.

Consider ΔOAP and ΔOAS,

AP = AS (Tangents from the same point)

OP = OS (Radii of the same circle)

OA = OA (Common side)

ΔOAP ≅ ΔOAS (SSS congruence criterion)

Therefore, A ↔ A, P ↔ S, O ↔ O

And thus, ∠POA = ∠AOS

∠1 = ∠8

Similarly,

∠2 = ∠3

∠4 = ∠5

∠6 = ∠7

∠1 + ∠2 + ∠3 + ∠4 + ∠5 + ∠6 + ∠7 + ∠8 = 360º

(∠1 + ∠8) + (∠2 + ∠3) + (∠4 + ∠5) + (∠6 + ∠7) = 360º


2∠1 + 2∠2 + 2∠5 + 2∠6 = 360º

2(∠1 + ∠2) + 2(∠5 + ∠6) = 360º

(∠1 + ∠2) + (∠5 + ∠6) = 180º

www.topperlearning.com 20
CBSE X | MATHEMATICS

Board Paper ˗ 2014

∠AOB + ∠COD = 180º

Similarly, we can prove that ∠BOC + ∠DOA = 180º

Hence, opposite sides of a quadrilateral circumscribing a circle subtend


supplementary angles at the centre of the circle.

www.topperlearning.com 21
CBSE X | Mathematics

Board Paper – 2015 Solution All India Set – 1

CBSE
Class X Mathematics
Board Paper – 2015 Solution
All India Set – 1

Time : 3 hours Total Marks : 90

SECTION A

1.
Given quadratic equation is,
px 2  2 5px  15  0
Here,a  p, b  2 5p, c  15
For real equal roots, discriminant  0
 b2  4ac  0

 
2
 2 5p  4p 15  0
20p2  60p  0
20p  p  3  0
 p  3 or p  0
But, p  0 is not possible.
p  3

2.
Let AB be the tower and BC be its shadow.
AB  20, BC  20 3
In ABC,
AB
tan  
BC
20
tan  
20 3
1
tan  
3
1
but, tan 30 
3
  30
 The Sun is at an altitude of 30 .

www.topperlearning.com 1
CBSE X | Mathematics

Board Paper – 2015 Solution All India Set – 1

3.
Two dice are tossed
S = {(1,1),(1,2),(1,3),(1,4),(1,5),(1,6),
(2,1),(2,2),(2,3),(2,4),(2,5),(2,6),
(3,1),(3,2),(3,3),(3,4),(3,5),(3,6),
(4,1),(4,2),(4,3),(4,4),(4,5),(4,6),
(5,1),(5,2),(5,3),(5,4),(5,5),(5,6),
(6,1),(6,2),(6,3),(6,4),(6,5),(6,6)}
Total number of outcomes when two dice are tossed = 6 x 6= 36
Favourable events of getting product as 6 are:
(1  6  6), (6  1  6),(2  3  6),(3  2  6)
i.e.(1,6), (6,1), (2,3), (3,2)
Favourable events of getting product as 6 = 4
4 1
P(getting product as 6) = 
36 9

4.

mOPT  90  radius is perpendicular to the tangent 


So, OPQ = OPT  QPT
= 90  60
= 30
mPOQ = 2QPT  2  60  120
reflex mPOQ = 360  120  240
1
mPRQ = reflex POQ
2
1
= 240
2
 mPRQ  120

www.topperlearning.com 2
CBSE X | Mathematics

Board Paper – 2015 Solution All India Set – 1

SECTION B

5.

Given that mPRQ = 120


We know that the line joining the centre and
the external point is the angle bisector between
the tangents.
120
Thus, mPRO = mQRO =  60
2
Also we know that lengths of tangents from an external point
are equal.
Thus, PR = RQ.
Join OP and OQ.
Since OP and OQ are the radii from the centre O,
OP  PR and OQ  RQ.
Thus, OPR and OQR are right angled congruent triangles.
Hence, POR=90  PRO=90  60  30
QOR=90  QRO = 90  60  30
1
sin QRO = sin30 
2
PR 1
sin30  
OR 2
 OR  2PR
 OR  PR  PR
 OR  PR  QR

www.topperlearning.com 3
CBSE X | Mathematics

Board Paper – 2015 Solution All India Set – 1

6.

Let the given circle touch the sides AB and AC of the triangle at points F and E
respectively and let the length of line segment AF be x.
Now, it can be observed that:
BF = BD = 6 cm (tangents from point B)
CE = CD = 9 cm (tangents from point C)
AE = AF = x (tangents from point A)

AB = AF + FB = x + 6
BC = BD + DC = 6 + 9 = 15
CA = CE + EA = 9 + x
2s = AB + BC + CA = x + 6 + 15 + 9 + x = 30 + 2x
s = 15 + x
s – a = 15 + x – 15 = x
s – b = 15 + x – (x + 9) = 6
s – c = 15 + x – (6 + x) = 9
Area of ABC = s  s  a  s  b  s  c 

54  15  x  x 6  9

54  3 6 15x  x 2 

18  6 15x  x 2 
324  6 15x  x  2

54  15x  x2
x2  15x  54  0
x2  18x  3x  54  0
x(x  18)  3(x  18)
 x  18  x  3  0
x  18 and x  3
As distance cannot be negative, x = 3
AC = 3 + 9 = 12
AB = AF + FB = 6 + x = 6 + 3 = 9

www.topperlearning.com 4
CBSE X | Mathematics

Board Paper – 2015 Solution All India Set – 1

7.
4x 2  4bx  (a 2  b2 )  0
 a2  b2 
 x 2  bx   0
 4 
b a2  b2
 x  2  x 
2

2 4
2 2
 b  b a b  b
2 2
 x  2  x    
2
 
2 2 4 2
2
 b  a2
x   
 2 4
b a
x 
2 2
b a
x 
2 2
b  a b  a
x ,
2 2
ab ab
Hence, the roots are    and  .
 2   2 

8.
S5  S7  167 and S10  235
n
Now, Sn 
2
2a   n  1 d
S5  S7  167
5 7
 2a  4d  2a  6d  167
2 2
 5a  10d  7a  21d  167
 12a  31d  167 ....(1)
Also, S10  235
10
 2a  9d  235
2
 10a  45d  235
 2a  9d  47 ....(2)

www.topperlearning.com 5
CBSE X | Mathematics

Board Paper – 2015 Solution All India Set – 1

Multiplying equation (2) by 6, we get


12a  54d  282 .....(3)
Subtracting (1) from (3), we get
12a  54d  282
   12a  31d  167
  
23d  115
d  5
Substituting value of d in (2), we have
2a  9(5)  47
 2a  45  47
 2a  2
a 1
Thus, the given A.P. is 1, 6, 11, 16,..........

9.
ABC is right angled at B.
 AC2  AB2  BC2 ....(1)
Also, A   4,7  , B   p,3 and C  7,3

Now, AC2  7  4   3  7   3   4   9  16  25


2 2 2 2

AB2   p  4   3  7   p2  8p  16   4 
2 2 2

 p2  8p  16  16
 p2  8p  32
BC2  7  p   3  3  49  14p  p2  0
2 2

 p2  14p  49
From (1), we have
  
25  p2  8p  32  p2  14p  49 
 25  2p2  22p  81
 2p2  22p  56  0
 p2  11p  28  0
 p2  7p  4p  28  0
 p p  7  4p  7  0
  p  7  p  4   0
 p  7 and p  4

www.topperlearning.com 6
CBSE X | Mathematics

Board Paper – 2015 Solution All India Set – 1

10.
Given, the point s A(x,y), B( 5,7) and C( 4,5) are collinear.
So, the area formed by these vertices is 0.
1
  x 7  5   55  y    4  y  7    0
2
1
 2x  25  5y  4y  28  0
2
1
 2x  y  3  0
2
 2x  y  3  0
 y  2x  3

SECTION C

11.
Here it is given that,
T14 = 2(T8)
⇒ a + (14 – 1)d = 2[a + (8 – 1)d]
⇒ a + 13d = 2[a + 7d]
⇒ a + 13d = 2a + 14d
⇒ 13d – 14d = 2a – a
⇒ – d = a …. (1)

Now, it is given that its 6th term is –8.


T6 = –8
⇒ a + (6 – 1)d = – 8
⇒ a + 5d = –8
⇒ –d + 5d = –8 [∵ Using (1)]
⇒ 4d = –8

⇒ d = –2
Subs. this in eq. (1), we get a = 2
Now, the sum of 20 terms,
n
Sn  2a  (n  1)d 
2
20
S20  2a  (20  1)d 
2
= 102(2)  19( 2)
= 10[4  38]
=  340

www.topperlearning.com 7
CBSE X | Mathematics

Board Paper – 2015 Solution All India Set – 1

12.
For the given equation, 3x 2  2 2x  2 3  0
Comparing thisequation withax 2  bx  c  0, we obtain
a  3,b  2 2,c  2 3
Now, D  b2  4ac
 ( 2 2)2  4( 3)( 2 3)
 8  24  32  4 2
Using quadraticformula, we obtain
b  b2  4ac
x
2a
( 2 2)  4 2
x
2 3
2 24 2 2 2 4 2
x or x 
2 3 2 3
2 2 2 2 2 2
x or x =
3 3
3 2  2
x or x 
3 3
 2
 x  3 2 or x 
3
 2
 x  6 or x 
3

13.
Let BC be the height at which the aeroplane is observed from point A.
Then, BC = 1500 3
In 15 seconds, the aeroplane moves from point A to D.
A and D are the points where the angles of elevation 60 and 30
are formed respectively.
Let BA = x metres and AD  y metres
BC = x + y

www.topperlearning.com 8
CBSE X | Mathematics

Board Paper – 2015 Solution All India Set – 1

In CBA,
BC
tan60°=
BA
1500 3
3
x
 x  1500 m ....(1)

In CBD,
BC
tan30°=
BD
1 1500 3

3 xy
 x  y  1500(3)  4500
1500  y  4500
 y  3000 m ....(2)
We know that, the aeroplane moves from point A to D in 15 seconds and the
distance covered is 3000 metres. (by 2)
dis tance
Speed 
time
3000
Speed 
15
Speed  200m / s

18
Converting it to km/hr = 200   720km / hr
5

www.topperlearning.com 9
CBSE X | Mathematics

Board Paper – 2015 Solution All India Set – 1

14.
Here, P(x,y) divides line segment AB, such that
3
AP  AB
7
AP 3

AB 7
AB 7

AP 3
AB 7
1  1
AP 3
AB  AP 7 3

AP 3
BP 4

AP 3
AP 3

BP 4
P divides AB in the ratio 3: 4
3  2  4  2 3   4   4  2 
x ; y
3 4 3 4
68 12  8
x ; y
7 7
2 20
x ; y
7 7
 2 20 
 The coordinates of P are  , 
 7 7 

15.
Here the jar contains red, blue and orange balls.
Let the number of red balls be x.
Let the number of blue balls be y.
Number of orange balls = 10
 Total number of balls = x + y + 10
Now, let P be the probability of drawing a ball from the jar
x
P(a red ball) =
x + y + 10
1 x
 
4 x + y + 10
 4x  x + y + 10
 3x  y  10 ----(i)

www.topperlearning.com 10
CBSE X | Mathematics

Board Paper – 2015 Solution All India Set – 1

Next,
y
P(a blue ball) =
x + y + 10
1 y
 
3 x + y + 10
 3y  x + y + 10
 2y  x  10 -----(ii)
Multiplying eq. (i) by 2 and adding to eq. (ii), we get
6x  2y  20
x + 2y  10
5x = 30
 x=6
Subs. x = 6 in eq. (i), we get y = 8
 Total number of balls = x + y + 10 = 6 + 8 + 10 = 24
Hence, total number of balls in the jar is 24.

16.
Radius of the circle =14 cm

Central Angle, 𝜽 = 60,


Area of the minor segment
 1
  r2  r2 sin 
360 2
60 1
   142   142  sin60
360 2
1 22 1 3
   14  14   14  14 
6 7 2 2
22  14
  49 3
3
22  14 147 3
 
3 3
308  147 3 2
 cm
3
308  147 3 2
Area of the minor segment  cm
3

www.topperlearning.com 11
CBSE X | Mathematics

Board Paper – 2015 Solution All India Set – 1

308  147 3 2
Area of major segment  r2  cm
3
22 308  147 3 2
  14  14  cm
7 3
308  147 3 2
 616  cm
3

17.
Diameter of the tent = 4.2 m
Radius of the tent, r = 2.1 m
Height of the cylindrical part of tent, hcylinder = 4 m
Height of the conical part, hcone = 2.8 m

 hcone2  r2

 2.82  2.12
 2.82  2.12
 12.25  3.5 m
Curved surface area of the cylinder = 2𝜋r hcylinder
22
=2× × 2.1 × 4
7
= 22 × 0.3 × 8 = 52.8 m2
22
Curved surface area of the conical tent = 𝜋rl = × 2.1 × 3.5 = 23.1 m2
7
Total area of cloth required for building one tent
= Curved surface area of the cylinder + Curved surface area of the conical tent
= 52.8 + 23.1
= 75.9 m2
Cost of building one tent = 75.9 × 100 = Rs 7590
Total cost of 100 tents = 7590 × 100 = Rs 7,59,000
759000
Cost to be borne by the associations = = Rs 3,79,500
2
It shows the helping nature, unity and cooperativeness of the associations.

www.topperlearning.com 12
CBSE X | Mathematics

Board Paper – 2015 Solution All India Set – 1

18.
Internal diameter of the bowl = 36 cm
Internal radius of the bowl, r = 18 cm
2 2
Volume of the liquid, V = 𝜋r3 = × 𝜋 × 183
3 3
Let the height of the small bottle be ‘h’.
Diameter of a small cylindrical bottle = 6 cm
Radius of a small bottle, R = 3 cm
Volume of a single bottle = 𝜋R2h = 𝜋 × 32 × h
No. of small bottles, n = 72
10 2
Volume wasted in the transfer = × × 𝜋 × 183
100 3
Volume of liquid to be transferred in the bottles
2 10 2
= × 𝜋 × 183 − × × 𝜋 × 183
3 100 3
2  10 
= × 𝜋 × 183  1 
3  100 
2 90
= × 𝜋 × 183 ×
3 100
Volume of the liquid to be transferred
Number of small cylindrical bottles =
Volume of a single bottle
2 90
  183 
 72  3 2
100
 3  h
2 9
 183 
 72  3 2 10
3 h
2 9
 18  18  18 
h 3 10
32  72
 h  5.4 cm

Height of the small cylindrical bottle = 5.4 cm

www.topperlearning.com 13
CBSE X | Mathematics

Board Paper – 2015 Solution All India Set – 1

19. Side of the cubical block, a = 10 cm


Longest diagonal of the cubical block = a√3 = 10√3 cm
Since the cube is surmounted by a hemisphere, therefore the side of the cube should
be equal to the diameter of the hemisphere.
Diameter of the sphere = 10 cm
Radius of the sphere, r = 5 cm
Total surface area of the solid = Total surface area of the cube – Inner cross-section
area of the hemisphere + Curved surface area of the hemisphere
= 6a2 – 𝜋r2 + 2𝜋r2
= 6a2 + 𝜋r2
= 6 × (10)2 + 3.14 52
 600  78.5  678.5 cm2
Total surface area of the solid = 678.5 cm2
Cost of painting per sq. cm = Rs. 5
Cost of painting the total surface area of the solid = 678.5 × 5 = Rs. 3392.50

20. No. of cones = 504


Diameter of a cone = 3.5 cm
Radius of the cone, r = 1.75 cm
Height of the cone, h = 3 cm
Volume of a cone
1
 r2h
3
2
1  3.5 
     3
3  2 
1 3.5 3.5
     3cm3
3 2 2
Volume of 504 cones
1 3.5 3.5
 504      3cm3
3 2 2
Let the radius of the new sphere be ‘R’.
4
Volume of the sphere  R3
3
Volume of 504 cones = Volume of the sphere
1 3.5 3.5 4
504      3  R3
3 2 2 3
504  1   3.5  3.5  3  3
  R3
3 2 2 4  
504  3  49
 R3 
64

www.topperlearning.com 14
CBSE X | Mathematics

Board Paper – 2015 Solution All India Set – 1

7  8  9  3  72
3
R 
64
8  27  73
 R3 
64
2 3 7
R 
4
21
R   10.5 cm
2
Radius of the new sphere = 10.5 cm
Surface area of the new sphere = 4R 2
22 21 21
 4  
7 2 2
 2772 cm2

SECTION D

21. Let l be the length of the longer side and b be the length of the shorter side.
Given that the length of the diagonal of the rectangular field is 16 metres more than
the shorter side.
Thus, diagonal = 16 + b
Since longer side is 14 metres more than shorter side, we have,
 l = 14 + b
Diagonal is the hypotenuse of the triangle.
Consider the following figure of the rectangular field.

www.topperlearning.com 15
CBSE X | Mathematics

Board Paper – 2015 Solution All India Set – 1

By applying Pythagoras Theorem in ABD, we have,


Diagonal2  Length2  Breadth2
 (16  b)2  b2
 256  b2  32b  196  b2  28b  b2
 256  32b  196  28b  b2
 60  32b  28b  b2
 b2  4b  60  0
 b2  10b  6b  60  0
 b(b  10)  6(b  10)  0
 (b  6)(b  10)  0
 (b  6)  0 or (b  10)  0
 b  6 or b=10
As breadth cannot be negative, breadth = 10 m
Thus, length of the rectangular field = 14+10 = 24m

22. Consider the given A.P. 8, 10, 12, …


Here the initial term is 8 and the common difference is 10 - 8 = 2 and 12 - 10 = 2
General term of an A.P. is tn and formula to find out tn is
t n  a  (n  1)d
 t 60  8  (60  1)  2
 t 60  8  59  2
 t 60  8  118
 t 60  126
We need to find the sum of the last 10 terms.
Thus,
Sum of last 10 terms = Sum of first 60 terms - Sum of first 50 terms
n
Sn  2a  (n  1)d 
2
60
 S60  2  8  (60  1)  2
2
 S60  3016  59  2
 S60  30[134]
 S60  4020

www.topperlearning.com 16
CBSE X | Mathematics

Board Paper – 2015 Solution All India Set – 1

Similarly,
50
 S50  2  8  (50  1)  2
2
 S50  2516  49  2
 S50  25114
 S50  2850
Thus the sum of last 10 terms = S60  S50  4020  2850  1170
Therefore,
Sum of last 10 terms = Sum of first 60 terms - Sum of first 50 terms

23.

Let x be the first speed of the train


Distance
We know that  time
Speed
Thus, we have,
54 63
  3 hours
x x 6
54(x  6)63x
 3
x(x  6)
 54(x+6)+63x=3x(x+6)
 54x+324+63x=3x 2  18x
 117x  324  3x 2  18x
 3x 2  117x  324  18x  0
 3x 2  99x  324  0
 x 2  33x  108  0
 x 2  36x  3x  108  0
 x(x  36)  3(x  36)  0
 (x  3)(x  36)  0
 (x  3)  0 or (x-36)=0
 x=-3 or x =36
Speed cannot be negative and hence intial speed of the train is 36km/hour

www.topperlearning.com 17
CBSE X | Mathematics

Board Paper – 2015 Solution All India Set – 1

24. Consider the following diagram.

Let P be an external point and PA and PB be tangents to the circle.


We need to prove that PA = PB
Now consider the triangles OAP and OBP
mA = mB = 90
OP = OP [common]
OA = OB = radii of the circle
Thus, by Right Angle-Hypotenuse-Side criterion of congruence we have,
OAP  OBP
The corresponding parts of the congruent triangles are congruent.
Thus,
PA = PB

25.
In the figure, C is the midpoint of the minor arc PQ, O is the centre of the circle and
AB is tangent to the circle through point C.
We have to show the tangent drawn at the midpoint of the arc PQ of a circle is parallel
to the chord joining the end points of the arc PQ.
We will show PQ AB.
It is given that C is the midpoint point of the arc PQ.
So, arc PC = arc CQ.
PC = CQ

www.topperlearning.com 18
CBSE X | Mathematics

Board Paper – 2015 Solution All India Set – 1

This shows that PQC is an isosceles triangle.


Thus, the perpendicular bisector of the side PQ of PQC passes through vertex C.
The perpendicular bisector of a chord passes through the centre of the circle.
So the perpendicular bisector of PQ passes through the centre O of the circle.
Thus perpendicular bisector of PQ passes through the points O and C.
PQ  OC
AB is the tangent to the circle through the point C on the circle.
AB  OC
The chord PQ and the tangent PQ of the circle are perpendicular to the same line OC.
PQ  AB.

26.

1)Construct the ABC as per given measurements.


2)In the half plane of AB which does not contain C, draw AX
such that BAX is an acute angle.
3) With some approprriate radius and centre A, Draw an arc to
intersect AX at B1 . Similarly, with center B1 and the same radius,
draw an arc to intersect BX at B2 such that B1B2 = B3B4 = B4B5 = B5B6
 B6B7 =B7B8
4) Draw B6B.
5) Through B8 draw a ray parallel to B6B. to intersect AY at B'.
6) Through B' draw a ray parallel to BC to intersect AZ at C'.
Thus, AB'C' is the required triangle.

www.topperlearning.com 19
CBSE X | Mathematics

Board Paper – 2015 Solution All India Set – 1

27.

Let PB be the surface of the lake and A be the point of observation such that
AP = 20 metres. Let C be the position of the cloud and C’ be its reflection in the lake.
Then CB = C’B. Let AM be perpendicular from A on CB.
Then mCAM  30 and mC'AM  60
Let CM = h. Then CB = h + 20 and C’B = h + 20. (CB=CB’ since refection about PB)

www.topperlearning.com 20
CBSE X | Mathematics

Board Paper – 2015 Solution All India Set – 1

In CMA we have,
CM
tan30 
AM
1 h
 
3 AM
 AM  3h...................(i)
In AMC' we have,
C'M
tan60 
AM
C'B  BM
 3
AM
h  20  20
 3
AM
h  20  20
 AM  ...................(ii)
3
From equation (i) and (ii), we get
h  20  20
3h 
3
 3h  h  40
 2h  40
 h  20 m
 AM  20 3
Now, to find AC using pythagoras theorem
AC2  AM2  MC2
 1600
AC  40
Hence, the height of the cloud from the
surface of the lake is 40 metres.

28.
Let S be the sample space of drawing a card from a well-shuffled deck.
n  S   52 C1  52

(i)There are 13 spade cards and 4 ace's in a deck


As ace of spade is included in 13 spade cards,
so there are 13 spade cards and 3 ace's

a card of spade or an ace can be drawn in 13


C1  3 C1  13  3  16
16 4
Probability of drawing a card of spade or an ace = 
52 13
www.topperlearning.com 21
CBSE X | Mathematics

Board Paper – 2015 Solution All India Set – 1

(ii)There are 2 black King cards in a deck


a card of black King can be drawn in 2 C1  2
2 1
Probability of drawing a black king = 
52 26

(iii)There are 4 Jack and 4 King cards in a deck.


So there are 52  8 = 44 cards which are neither Jacks nor Kings.
a card which is neither a Jack nor a King can be drawn in 44
C1  44
44 11
Probability of drawing a card which is neither a Jack nor a King = 
52 13

(iv)There are 4 King and 4 Queen cards in a deck.


So there are 4  4 = 8 cards which are either King or Queen.
a card which is either a King or a Queen can be drawn in 8 C1  8
8 2
Probability of drawing a card which is either a King or a Queen = 
52 13

29.
Take  x1 ,y 1   1, 1  ,  4,2k  and  k, 5
It is given that the area of the triangle is 24 sq. unit

Area of the triangle having vertices  x 1 ,y 1  ,  x 2 ,y 2  and  x3 ,y 3 


is given by
1
=  x1  y 2  y 3   x 2  y 3  y 1   x3  y 1  y 2  
2
1
24  1  2k   5    4    5   1     k    1   2k  
2

48   2k  5  16  k  2k 2  
2k 2  3k  27  0
 2k  9  k  3  0
9
 k   or k  3
2

9
 The values of k are  and3.
2

www.topperlearning.com 22
CBSE X | Mathematics

Board Paper – 2015 Solution All India Set – 1

30.
PQRS is a square.
So each side is equal and angle between the adjacent sides is a right angle.
Also the diagonals perpendicularly bisect each other.
In PQR using pythagoras theorem,
PR 2 =PQ2 +QR 2
2 2
PR 2   42   42
PR= 2  42
1 42
OR  PR=  OQ
2 2
From the figure we can see that the radius of flower bed ORQ is OR.
1
Area of sector ORQ  r2
4
2
1  42 
 
4  2 
1
Area of the ROQ =  RO  OQ
2
1 42 42
  
2 2 2
2
 42 
 
 2 
Area of the flower bed ORQ
=Area of sector ORQ  Area of the ROQ
2 2
1  42   42 
=  
4  2   2 
2
 42    
     1
 2  2 
  441  0.57
 251.37cm2
Area of the flower bed ORQ = Area of the flower bed OPS
= 251.37cm2
Total area of the two flower beds
= Area of the flower bed ORQ + Area of the flower bed OPS
=251.37  251.37
 502.74 cm2

www.topperlearning.com 23
CBSE X | Mathematics

Board Paper – 2015 Solution All India Set – 1

31.
Height of the cylinder (h) = 10 cm
Radius of the base of the cylinder = 4.2 cm
Volume of original cylinder = r2h
22 2
   4.2  10
7
 554.4cm3
2 3
Volume of hemisphere = r
3
2 22 3
    4.2
3 7
 155.232 cm3

Volume of the remaining cylinder after scooping out hemisphere from each end
 Volume of original cylinder  2  Volume of hemisphere
 554.4  2  155.232
 243.936 cm3

The remaining cylinder is melted and converted to


a new cylindrical wire of 1.4 cm thickness.
So they have same volume and radius of new cylindrical wire is 0.7 cm.
Volume of the remaining cylinder = Volume of the new cylindrical wire
243.936  r2h
22 2
243.936   0.7  h
7
h  158.4 cm

 The length of the new cylindrical wire of 1.4 cm thickness is 158.4 cm.

www.topperlearning.com 24
CBSE X | Mathematics

Board Paper – 2015 Solution All India Set – 2

CBSE
Class X Mathematics
Board Paper – 2015 Solution
All India Set – 2

Time: 3 hours Total Marks: 90

1.

mOPT  90  radius is perpendicular to the tangent 


So, OPQ = OPT  QPT
= 90  60
= 30
mPOQ = 2QPT  2  60  120
reflex mPOQ = 360  120  240
1
PRQ = reflex POQ
2
1
=  240
2
 120
 mPRQ  120

www.topperlearning.com 1
CBSE X | Mathematics

Board Paper – 2015 Solution All India Set – 2

2.
The given quadratic equation is,
px 2  2 5px  15  0
Here,a  p, b  2 5p, c  15
For real equal roots, discriminant  0
 b2  4ac  0

 
2
 2 5p  4p 15  0

20p2  60p  0
20p  p  3  0
 p  3 or p  0
But, p  0 is not possible.
p  3

3.
Let AB be the tower and BC be its shadow.
AB  20, BC  20 3
In ABC,
AB
tan  
BC
20
tan  
20 3
1
tan  
3
1
but, tan 30 
3
  30
 The Sun is at an altitude of 30 .

www.topperlearning.com 2
CBSE X | Mathematics

Board Paper – 2015 Solution All India Set – 2

4.
Two dice are tossed
S = [(1,1),(1,2),(1,3),(1,4),(1,5),(1,6),
(2,1),(2,2),(2,3),(2,4),(2,5),(2,6),
(3,1),(3,2),(3,3),(3,4),(3,5),(3,6),
(4,1),(4,2),(4,3),(4,4),(4,5),(4,6),
(5,1),(5,2),(5,3),(5,4),(5,5),(5,6),
(6,1),(6,2),(6,3),(6,4),(6,5),(6,6)]
Total number of outcomes when two dice are tossed = 6 x 6= 36
Favourable events of getting the product as 6 are:
(1  6  6), (6  1  6),(2  3  6),(3  2  6)
i.e.(1,6), (6,1), (2,3), (3,2)
Favourable events of getting product as 6 = 4
4 1
P(getting product as 6) = 
36 9

SECTION B

5.
Given that the point s A(x,y), B( 5,7) and C( 4,5) are collinear.
So, the area formed by these vertices is 0.
1
  x 7  5   55  y    4  y  7    0
2
1
 2x  25  5y  4y  28  0
2
1
 2x  y  3  0
2
 2x  y  3  0
 y  2x  3

www.topperlearning.com 3
CBSE X | Mathematics

Board Paper – 2015 Solution All India Set – 2

6.
S5  S7  167 and S10  235
n
Now, Sn 
2
2a   n  1 d
S5  S7  167
5 7
 2a  4d  2a  6d  167
2 2
 5a  10d  7a  21d  167
 12a  31d  167 ....(1)
Also, S10  235
10
 2a  9d  235
2
 10a  45d  235
 2a  9d  47 ....(2)
Multiplying equation (2) by 6, we get
12a  54d  282 .....(3)
Subtracting (1) from (3), we get
12a  54d  282
   12a  31d  167
  
23d  115
d  5
Substituting value of d in (2), we have
2a  9(5)  47
 2a  45  47
 2a  2
a 1
Thus, the given A.P. is 1, 6, 11, 16,..........

www.topperlearning.com 4
CBSE X | Mathematics

Board Paper – 2015 Solution All India Set – 2

7.

Given that m PRQ = 120


We know that the line joining the centre and
the external point is the angle bisector between
the tangents.
120
Thus, m PRO = m QRO = 60
2
Also we know that lengths of tangents from an external point
are equal.
Thus, PR = RQ.
Join OP and OQ.
Since OP and OQ are the radii from the centre O,
OP PR and OQ RQ.
Thus, OPR and OQR are right angled congruent triangles.
Hence, m POR = 90 m PRO = 90 60 30
m QOR = 90 m QRO = 90 60 30
1
sin QRO = sin30
2
PR
But sin30
OR
PR 1
Thus,
OR 2
OR 2PR
OR PR PR
OR PR QR

www.topperlearning.com 5
CBSE X | Mathematics

Board Paper – 2015 Solution All India Set – 2

8.

Let the given circle touch the sides AB and AC of the triangle at points F and E
respectively and let the length of the line segment AF be x.
Now, it can be observed that:
BF = BD = 6 cm (tangents from point B)
CE = CD = 9 cm (tangents from point C)
AE = AF = x (tangents from point A)

AB = AF + FB = x + 6
BC = BD + DC = 6 + 9 = 15
CA = CE + EA = 9 + x
2s = AB + BC + CA = x + 6 + 15 + 9 + x = 30 + 2x
s = 15 + x
s – a = 15 + x – 15 = x
s – b = 15 + x – (x + 9) = 6
s – c = 15 + x – (6 + x) = 9
Area of ABC = s  s  a  s  b  s  c 

54  15  x  x 6  9

54  3 6 15x  x 2 

18  6 15x  x2 
324  6 15x  x  2

54  15x  x2
x2  15x  54  0
x2  18x  3x  54  0
x(x  18)  3(x  18)
 x  18  x  3  0
x  18 and x  3
As distance cannot be negative, x = 3
AC = 3 + 9 = 12
AB = AF + FB = 6 + x = 6 + 3 = 9

www.topperlearning.com 6
CBSE X | Mathematics

Board Paper – 2015 Solution All India Set – 2

9.
4x 2  4bx  (a 2  b2 )  0
 a2  b2 
 x 2  bx   0
 4 
b a2  b2
 x  2  x 
2

2 4
2 2
 b  b a b  b
2 2
 x  2  x    
2
 
2 2 4 2
2
 b  a2
x   
 2 4
b a
x 
2 2
b a
x 
2 2
b  a b  a
x ,
2 2
ab ab
Hence, the roots are    and  .
 2   2 

www.topperlearning.com 7
CBSE X | Mathematics

Board Paper – 2015 Solution All India Set – 2

10. Given that A(4, 3), B(-1, y) and C(3, 4) are the vertices of the ABC.
∆ABC is a right triangle at A.
Hence by applying the Pythagoras Theorem, we have,
AB2  AC2  BC2 ....(1)
Let us find the distances, AB, BC and CA using the
distance formula.

 1  4    y  3
2 2
AB =

3  1    4  y 
2 2
BC 

3  4    4  3
2 2
CA   2
Squaring both the sides, we have
AB2  25  y 2  9  6y
BC2  4  16  y 2  8y
AC2  2
Therefore, from equation (1), we have,
25  y 2  9  6y  2  4  16  y 2  8y
 36  y 2  6y  20  y 2  8y
 16  6y  8y
 16  8y  6y
 2y  16
16
 y 
2
 y  8

www.topperlearning.com 8
CBSE X | Mathematics

Board Paper – 2015 Solution All India Set – 2

SECTION C

11. Diameter of the tent = 4.2 m


Radius of the tent, r = 2.1 m
Height of the cylindrical part of tent, hcylinder = 4 m
Height of the conical part, hcone = 2.8 m
Slant height of the conical part, 

hcone2 r2

2.82 2.12
2.82 2.12
12.25 3.5 m
Curved surface area of the cylinder = 2𝜋r hcylinder
22
=2× × 2.1 × 4
7
= 22 × 0.3 × 8 = 52.8 m2
22
Curved surface area of the conical tent = 𝜋rl = × 2.1 × 3.5 = 23.1 m2
7
Total area of cloth required for building one tent
= Curved surface area of the cylinder + Curved surface area of the conical tent
= 52.8 + 23.1
= 75.9 m2
Cost of building one tent = 75.9 × 100 = Rs. 7590
Total cost of 100 tents = 7590 × 100 = Rs. 7,59,000
759000
Cost to be borne by the associations = = Rs. 3,79,500
2
It shows the helping nature, unity and cooperativeness of the associations.

www.topperlearning.com 9
CBSE X | Mathematics

Board Paper – 2015 Solution All India Set – 2

12.
Let BC be the height at which the aeroplane is observed from point A.
Then, BC = 1500 3
In 15 seconds, the aeroplane moves from point A to D.
A and D are the points where the angles of elevation 60 and 30
are formed respectively.
Let BA = x metres and AD  y metres
BC = x + y

In CBA,
BC
tan60°=
BA
1500 3
3
x
 x  1500 m ....(1)

In CBD,
BC
tan30°=
BD
1 1500 3

3 xy
 x  y  1500(3)  4500
1500  y  4500
 y  3000 m ....(2)
We know that the aeroplane moves from point A to D in 15 seconds and the distance
covered is 3000 metres. (by 2)

www.topperlearning.com 10
CBSE X | Mathematics

Board Paper – 2015 Solution All India Set – 2

dis tance
Speed 
time
3000
Speed 
15
Speed  200m/s

18
Converting it to km/hr = 200   720 km/hr
5

13. Internal diameter of the bowl = 36 cm


Internal radius of the bowl, r = 18 cm
2 2
Volume of the liquid, V = 𝜋r3 = × 𝜋 × 183
3 3
Let the height of the small bottle be ‘h’.
Diameter of a small cylindrical bottle = 6 cm
Radius of the small bottle, R = 3 cm
Volume of a single bottle = 𝜋R2h = 𝜋 × 32 × h
No. of small bottles, n = 72
10 2
Volume wasted in the transfer = × × 𝜋 × 183
100 3
Volume of liquid to be transferred in the bottles
2 10 2
= × 𝜋 × 183 − × × 𝜋 × 183
3 100 3
2 10
= × 𝜋 × 183 1
3 100
2 90
= × 𝜋 × 183 ×
3 100
Volume of the liquid to be transferred
Number of the small cylindrical bottles =
Volume of a single bottle
2 90
183
72 3 100
32 h
2 9
183
72 3 2 10
3 h
2 9
18 18 18
h 3 2
10
3 72
h 5.4 cm

Height of the small cylindrical bottle = 5.4 cm

www.topperlearning.com 11
CBSE X | Mathematics

Board Paper – 2015 Solution All India Set – 2

14.
Here the jar contains red, blue and orange balls.
Let the number of red balls be x.
Let the number of blue balls be y.
Number of orange balls = 10
 Total number of balls = x + y + 10
Now, let P be the probability of drawing a ball from the jar
x
P(a red ball) =
x + y + 10
1 x
 
4 x + y + 10
 4x  x + y + 10
 3x  y  10 ----(i)
Next,
y
P(a blue ball) =
x + y + 10
1 y
 
3 x + y + 10
 3y  x + y + 10
 2y  x  10 -----(ii)
Multiplying eq. (i) by 2 and adding to eq. (ii), we get
6x  2y  20
x + 2y  10
5x = 30
 x=6
Subs. x = 6 in eq. (i), we get y = 8
 Total number of balls = x + y + 10 = 6 + 8 + 10 = 24
Hence, total number of balls in the jar is 24.

www.topperlearning.com 12
CBSE X | Mathematics

Board Paper – 2015 Solution All India Set – 2

15. Side of the cubical block, a = 10 cm


Longest diagonal of the cubical block = a√3 = 10√3 cm
Since the cube is surmounted by a hemisphere, therefore the side of the cube should
be equal to the diameter of the hemisphere.
Diameter of the sphere = 10 cm
Radius of the sphere, r = 5 cm
Total surface area of the solid = Total surface area of the cube – Inner cross-section
area of the hemisphere + Curved surface area of the hemisphere
= 6a2 – 𝜋r2 + 2𝜋r2
= 6a2 + 𝜋r2
= 6 × (10)2 + 3.14 52
600 78.5 678.5 cm2
Total surface area of the solid = 678.5 cm2
Cost of painting per sq. cm = Rs. 5
Cost of painting the total surface area of the solid = 678.5 × 5 = Rs. 3392.50

16.
Here, P(x,y) divides line segment AB, such that
3
AP  AB
7
AP 3

AB 7
AB 7

AP 3
AB 7
1  1
AP 3
AB  AP 7 3

AP 3
BP 4

AP 3
AP 3

BP 4
P divides AB in the ratio 3: 4
3  2  4  2 3   4   4  2 
x ; y
3 4 3 4
68 12  8
x ; y
7 7
2 20
x ; y
7 7
 2 20 
 The co  ordinates of P are  , 
 7 7 

www.topperlearning.com 13
CBSE X | Mathematics

Board Paper – 2015 Solution All India Set – 2

17. No. of cones = 504


Diameter of a cone = 3.5 cm
Radius of the cone, r = 1.75 cm
Height of the cone, h = 3 cm
Volume of a cone
1 2
rh
3
2
1 3.5
3
3 2
1 3.5 3.5
3 cm3
3 2 2
Volume of 504 cones
1 3.5 3.5
504 3 cm3
3 2 2
Let the radius of the new sphere be ‘R’.
4 3
Volume of the sphere R
3
Volume of 504 cones = Volume of the sphere
1 3.5 3.5 4 3
504 3 R
3 2 2 3
504 1 3.5 3.5 3 3
R3
3 2 2 4
504 3 49
R3
64
7 8 9 3 72
R3
64
8 27 73
R3
64
2 3 7
R
4
21
R 10.5 cm
2
Radius of the new sphere = 10.5 cm

Surface area of the new sphere=4R 2


22 21 21
 4  
7 2 2
 2772 cm2

www.topperlearning.com 14
CBSE X | Mathematics

Board Paper – 2015 Solution All India Set – 2

18. Given that the area of the circle is 1256 cm2.


r2  1256
 3.14  r2  1256
1256
 r2 
3.14
 r  400
2

 r  20 cm
If all the vertices of a rhombus lie on a circle, then
the rhombus is square.
Consider the following figure.

Here A, B, C and D are four points on the circle.


Thus, OA = OB = OC = OD = radius of the circle.
AC and BD are the diameters of the circle.
Consider the  ADC.
By Pythagoras theorem, we have,
AD2  CD2  AC2
 2AD2   2  20  ....[AD  CD, side of the square]
2

 2AD2   40
2

 2AD2  1600
1600
 AD2 
2
 AD  800cm2
2

If AD is the side of the square, then AD2 is the area of the square.
Thus area of the square is 800cm2

www.topperlearning.com 15
CBSE X | Mathematics

Board Paper – 2015 Solution All India Set – 2

19. Consider the given equation:


2x2  6 3  60  0
 x2  3 3  30  0
Let us the quadratic formula to find x.
b  b2  4ac
x=
2a
Here, a = 1, b = 3 3 and c =  30
Thus,

3 3 
2
3 3   4  1   30 
x=
2 1
3 3  27  120
 x=
2
3 3  147
 x=
2

20. Given that 16th term of an A.P. is five times its third term.
We know that
t n  a   n  1 d
Thus,
t 16  a  16  1  d
t 3  a  3  1  d
Since t 16  5t 3 , we have,
a  16  1  d  5 a  3  1  d 
 a  15d  5a  2d 
 a  15d  5a  10d
 5d  4a
 4a  5d  0...(1)
Also given that t 10  41
 t 10  a  10  1  d
 41  a  9d
 a  9d  41...(2)
Multiplying equation (2) by 4, we have,
4a + 36d = 164...(3)

www.topperlearning.com 16
CBSE X | Mathematics

Board Paper – 2015 Solution All India Set – 2

Subtracting equation (1) from equation (3), we have,


36   5  d  164
 41d  164
164
d
41
d4
Substituting d = 4 in equation (1) 4a  5d  0,we have,
4a  5  4  0
 4a  20  0
 4a  20
20
a
4
a 5
We need to find S15
We know that
n
Sn  2a   n  1  d 
2
15
 S15  2  5  15  1   4 
2
 a  5,n  15,d  4 

15
 S15  10  14  4
2
15
 S15   66
2
 S15  495

www.topperlearning.com 17
CBSE X | Mathematics

Board Paper – 2015 Solution All India Set – 2

SECTION D

21. In the figure, C is the midpoint of the minor arc PQ, O is the centre of the circle and
AB is tangent to the circle through point C.
We have to show the tangent drawn at the mid-point of the arc PQ of a circle is
parallel to the chord joining the end points of the arc PQ.
We will show PQ AB.
It is given that C is the midpoint point of the arc PQ.
So, arc PC = arc CQ.
PC = CQ

This shows that PQC is an isosceles triangle.


Thus, the perpendicular bisector of the side PQ of PQC passes through vertex C.
The perpendicular bisector of a chord passes through the centre of the circle.
So the perpendicular bisector of PQ passes through the centre O of the circle.
Thus the perpendicular bisector of PQ passes through the points O and C.
PQ  OC
AB is the tangent to the circle through the point C on the circle.
AB  OC
The chord PQ and the tangent PQ of the circle are perpendicular to the same line OC.
PQ  AB.

www.topperlearning.com 18
CBSE X | Mathematics

Board Paper – 2015 Solution All India Set – 2

22.

Let PB be the surface of the lake and A be the point of observation such that
AP = 20 metres. Let C be the position of the cloud and C’ be its reflection in the lake.
Then CB = C’B. Let AM be perpendicular from A on CB.
Then mCAM  30 and mC'AM  60
Let CM = h. Then CB = h + 20 and C’B = h + 20. (CB=CB’ since refection about PB)
In CMA we have,
CM
tan30 
AM
1 h
 
3 AM
 AM  3h...................(i)

In AMC' we have,
C'M
tan 60 
AM
C'B  BM
 3
AM
h  20  20
 3
AM
h  20  20
 AM  ...................(ii)
3
From equation (i) and (ii), we get
h  20  20
3h 
3
 3h  h  40
 2h  40
 h  20 m
 AM  20 3
Now , to find AC u sin g pythagoras theorem
AC2  AM2  MC2
 1600
AC  40
Hence, the height of the cloud from the
surface of the lake is 40 metres.

www.topperlearning.com 19
CBSE X | Mathematics

Board Paper – 2015 Solution All India Set – 2

23.
Let S be the sample space of drawing a card from a well-shuffled deck.
n  S   52 C1  52

(i)There are 13 spade cards and 4 ace's in a deck


As ace of spade is included in 13 spade cards,
so there are 13 spade cards and 3 ace's

A card of spade or an ace can be drawn in 13


C1  3 C1  13  3  16
16 4
Probability of drawing a card of spade or an ace = 
52 13

(ii)There are 2 black king cards in a deck


A card of black king can be drawn in 2 C1  2
2 1
Probability of drawing a black king = 
52 26

(iii)There are 4 jack and 4 king cards in a deck.


So there are 52  8 = 44 cards which are neither jack nor king.
a card which is neither a jack nor a king can be drawn in 44
C1  44
44 11
Probability of drawing a card which is neither a jack nor a king = 
52 13

(iv)There are 4 king and 4 queen cards in a deck.


So there are 4  4 =8 cards which are either king or queen.
a card which is either a king or a queen can be drawn in 8 C1  8
8 2
Probability of drawing a card which is either a king or a queen = 
52 13

www.topperlearning.com 20
CBSE X | Mathematics

Board Paper – 2015 Solution All India Set – 2

24. PQRS is a square.

So each side is equal and angle between the adjacent sides is a right angle.
Also the diagonals perpendicularly bisect each other.
In PQR using pythagoras theorem,
PR 2 =PQ2 +QR 2
2 2
PR 2   42   42
PR= 2  42
1 42
OR  PR=  OQ
2 2
From the figure we can see that the radius of the flower bed ORQ is OR.
1
Area of sector ORQ  r 2
4
2
1  42 
 
4  2 
1
Area of the ROQ =  RO  OQ
2
1 42 42
  
2 2 2
2
 42 
 
 2 

Area of the flower bed ORQ


= Area of sector ORQ  Area of the ROQ
2 2
1  42   42 
=  
4  2   2 
2
 42    
     1
 2  2 
  441 0.57
 251.37cm2

Area of the flower bed ORQ =Area of the flower bed OPS
=251.37cm2

Total area of the two flower beds


= Area of the flower bed ORQ+Area of the flower bed OPS
=251.37  251.37
 502.74cm2

www.topperlearning.com 21
CBSE X | Mathematics

Board Paper – 2015 Solution All India Set – 2

25. Height of the cylinder (h) = 10 cm


Radius of the base of the cylinder = 4.2 cm
Volume of original cylinder =r2h
22 2
   4.2  10
7
 554.4cm3
2
Volume of hemisphere = r3
3
2 22 3
    4.2
3 7
 155.232cm3

Volume of the remaining cylinder after scooping out the hemisphere from each end
 Volume of original cylinder  2  Volume of hemisphere
 554.4  2  155.232
 243.936cm3

The remaining cylinder is melted and converted to


a new cylindrical wire of 1.4 cm thickness.
So they have the same volume and radius of the new cylindrical wire, i.e. 0.7 cm.
Volume of the remaining cylinder = Volume of the new cylindrical wire
243.936  r2h
22 2
243.936   0.7  h
7
h  158.4cm

 The length of the new cylindrical wire of 1.4 cm thickness is 158.4 cm.

www.topperlearning.com 22
CBSE X | Mathematics

Board Paper – 2015 Solution All India Set – 2

26. Let  be the length of the longer side and b be the length of the shorter side.
Given that the length of the diagonal of the rectangular field is 16 metres more than
the shorter side.
Thus, diagonal = 16 + b
Since longer side is 14 metres more than shorter side, we have,
=14 + b
Diagonal is the hypotenuse of the triangle.
Consider the following figure of the rectangular field.

By applying Pythagoras Theorem in ABD, we have,


Diagonal2 Length2 Breadth2
(16 b)2 (14 b)2 b2
256 b2 32b 196 b2 28b b2
256 32b 196 28b b2
60 32b 28b b2
b2 4b 60 0
b2 10b 6b 60 0
b(b 10) 6(b 10) 0
(b 6)(b 10) 0
(b 6) 0 or (b 10) 0
b 6 or b 10
As breadth cannot be negative, breadth =10 m
Thus, length of the rectangular field=14+10=24 m

www.topperlearning.com 23
CBSE X | Mathematics

Board Paper – 2015 Solution All India Set – 2

27. Consider the given A.P. 8, 10, 12, …


Here the initial term is 8 and the common difference is 10 - 8 = 2 and 12 - 10 = 2
General term of an A.P. is tn and formula to tn is
tn a n 1 d
t 60 8 60 1 2
t 60 8 59 2
t 60 8 118
t 60 126
We need to find the sum of last 10 terms.
Thus,
Sum of last 10 terms = Sum of first 60 terms - Sum of first 50 terms
n
Sn 2a n 1 d
2
60
S60 2 8 60 1 2
2
S60 30 16 59 2
S60 30 134
S60 4020

Similarly,
50
S50 2 8 50 1 2
2
S50 25 16 49 2
S50 25 114
S50 2850
Therefore,
Sum of last 10 terms = Sum of first 60 terms Sum of first 50 terms
Thus the sum of last 10 terms =S60 S50 4020 2850 1170

www.topperlearning.com 24
CBSE X | Mathematics

Board Paper – 2015 Solution All India Set – 2

28. Let x be the initial speed of the bus.


Distance
We know that time
Speed
Thus,we have,
75 90
3 hours
x x 10
75 x+10 90x
3
x x+10
75 x+10 90x 3x x+10
75x 750 90x 3x 2 30x
2
165x 750 3x 30x
2
3x 165x 750 30x 0
2
3x 135x 750 0
x2 45x 250 0
x 2 50x 5x 250 0
x(x 50) 5(x 50) 0
(x 50) 0 or (x 5) 0
x 5 or x 50
Speed cannot be negative and hence first speed of the train is 50 km/hour.

www.topperlearning.com 25
CBSE X | Mathematics

Board Paper – 2015 Solution All India Set – 2

29.

Given: Line l is tangent to the circle  O, r  at point A.


To prove: OA  l
Proof: Let P  l , P  A.
If P is in the interior of circle  O, r  , then the line l will be a secant of the circle and not a tangent.
But l is a tangent of the circle, so P is not in the interior of the circle.
Also P  A.
P is the point in the exterior of the circle.

OP > OA. OA is the radius of the circle 
Therefore each point P l except A satisfies the inequality OP > OA.
Therefore OA is the shortest distance of line l from O.
OA  l .

www.topperlearning.com 26
CBSE X | Mathematics

Board Paper – 2015 Solution All India Set – 2

30. Steps of construction:


1) Construct the triangle as per given measurements.
2) Take any arbitrary radius and draw two arcs of circle from point B on AC,
intersecting AC at X and Y.
3) Taking X and Y as centres, draw two arcs of circles to intersect each other at point
E. Join B and E. BE is the perpendicular from B on AC.
4) BDC is a right angled. Hence, BC the hypotenuse will form the diameter of the
circle passing through the vertices of BDC.
5) BC = 8 cm OC = 4 cm. draw a circle of radius equal 4 cm, passing through B, D
and C.
6) Join O and A. Obtain the mid-point P of segment OA by drawing perpendicular
bisector to OA.
7) Draw a circle with centre P and radius AP.
8) Let B and F be the points of intersection of these two circles.
Hence, AB and AF are the required tangents.

www.topperlearning.com 27
CBSE X | Mathematics

Board Paper – 2015 Solution All India Set – 2

31. Let A(k + 1, 1), B(4, -3) and C(7, - k) are the vertices of the triangle.
Given that the area of the triangle is 6 sq. units.
Area of the triangle is given by
1
 
A=  x1  y 2  y 3   x 2  y 3  y 1   x3 y 1  y 2 
2 
1
 
 6   x1  y 2  y 3   x 2  y 3  y 1   x3 y 1  y 2 
2 
 12   k  1 3  k   4  k  1   7 1  3
 12  3k  k 2  3  k  4k  4  28
 12  k 2  6k  21
 k 2  6k  21  12  0
 k 2  6k  9  0
  k  3  0
2

 k  3,3

www.topperlearning.com 28
CBSE X | Mathematics

Board Paper – 2015 Solution All India Set – 3

CBSE
Class X Mathematics
Board Paper – 2015 Solution
All India Set – 3
Time: 3 hours Total Marks: 90

1.
Let AB be the tower and BC be its shadow.
AB  20, BC  20 3
In ABC,
AB
tan  
BC
20
tan  
20 3
1
tan  
3
1
but, tan30 
3
  30
 The Sun is at an altitude of 30 .

2.
Two dice are tossed
S = [(1,1),(1,2),(1,3),(1,4),(1,5),(1,6),
(2,1),(2,2),(2,3),(2,4),(2,5),(2,6),
(3,1),(3,2),(3,3),(3,4),(3,5),(3,6),
(4,1),(4,2),(4,3),(4,4),(4,5),(4,6),
(5,1),(5,2),(5,3),(5,4),(5,5),(5,6),
(6,1),(6,2),(6,3),(6,4),(6,5),(6,6)]
Total number of outcomes when two dice are tossed = 6 x 6= 36
Favourable events of getting the product as 6 are:
(1  6  6), (6  1  6),(2  3  6),(3  2  6)
i.e.(1,6), (6,1), (2,3), (3,2)
Favourable events of getting product as 6 = 4
4 1
P(getting product as 6) = 
36 9

www.topperlearning.com 1
CBSE X | Mathematics

Board Paper – 2015 Solution All India Set – 3

3.
Given quadratic equation is,
px 2  2 5px  15  0
Here,a  p, b  2 5p, c  15
For real equal roots, discriminant  0
 b2  4ac  0

 
2
 2 5p  4p 15  0
20p2  60p  0
20p  p  3  0
 p  3 or p  0
But, p  0 is not possible.
p  3

www.topperlearning.com 2
CBSE X | Mathematics

Board Paper – 2015 Solution All India Set – 3

4.

mOPT  90  radius is perpendicular to the tangent 


So, OPQ = OPT  QPT
= 90  60
= 30
mPOQ = 2mQPT  2  60  120
reflex mPOQ = 360  120  240
1
PRQ = reflex POQ
2
1
=  240
2
 120
 mPRQ  120

www.topperlearning.com 3
CBSE X | Mathematics

Board Paper – 2015 Solution All India Set – 3

SECTION B

5.
S5  S7  167 and S10  235
n
Now, Sn 
2
2a   n  1 d
S5  S7  167
5 7
 2a  4d  2a  6d  167
2 2
 5a  10d  7a  21d  167
 12a  31d  167 ....(1)
Also, S10  235
10
 2a  9d  235
2
 10a  45d  235
 2a  9d  47 ....(2)
Multiplying equation (2) by 6, we get
12a  54d  282 .....(3)
Subtracting (1) from (3), we get
12a  54d  282
   12a  31d  167
  
23d  115
d  5
Substituting value of d in (2), we have
2a  9(5)  47
 2a  45  47
 2a  2
a 1
Thus, the given A.P. is 1, 6, 11, 16,..........

www.topperlearning.com 4
CBSE X | Mathematics

Board Paper – 2015 Solution All India Set – 3

6.
ABC is right triangle at B.
 AC2  AB2  BC2 ....(1)
Also, A   4,7  , B   p,3 and C  7,3

Now, AC2  7  4   3  7   3   4   9  16  25


2 2 2 2

AB2   p  4   3  7   p2  8p  16   4 
2 2 2

 p2  8p  16  16
 p2  8p  32
BC2  7  p   3  3  49  14p  p2  0
2 2

 p2  14p  49
From (1), we have
  
25  p2  8p  32  p2  14p  49 
 25  2p2  22p  81
 2p2  22p  56  0
 p2  11p  28  0
 p2  7p  4p  28  0
 p p  7  4p  7  0
  p  7  p  4   0
 p  7 and p  4

www.topperlearning.com 5
CBSE X | Mathematics

Board Paper – 2015 Solution All India Set – 3

7.
Given that m PRQ = 120
We know that the line joining the centre and
the external point is the angle bisector between
the tangents.
120
Thus, m PRO = m QRO = 60
2
Also we know that lengths of tangents from an external point
are equal.
Thus, PR = RQ.
Join OP and OQ.
Since OP and OQ are the radii from the centre O,
OP PR and OQ RQ.
Thus, OPR and OQR are right angled congruent triangles.
Hence, POR=90 PRO=90 60 30
QOR=90 QRO=90 60 30
1
sin QRO = sin30
2
PR
But sin30
OR
PR 1
Thus,
OR 2
OR 2PR
OR PR PR
OR PR QR

www.topperlearning.com 6
CBSE X | Mathematics

Board Paper – 2015 Solution All India Set – 3

8. Let the given circle touch the sides AB and AC of the triangle at points F and E
respectively and let the length of line segment AF be x.
Now, it can be observed that:
BF = BD = 6 cm (tangents from point B)
CE = CD = 9 cm (tangents from point C)
AE = AF = x (tangents from point A)

AB = AF + FB = x + 6
BC = BD + DC = 6 + 9 = 15
CA = CE + EA = 9 + x
2s = AB + BC + CA = x + 6 + 15 + 9 + x = 30 + 2x
s = 15 + x
s – a = 15 + x – 15 = x
s – b = 15 + x – (x + 9) = 6
s – c = 15 + x – (6 + x) = 9
Area of ABC = s  s  a  s  b  s  c 

54  15  x  x 6  9

54  3 6 15x  x 2 

18  6 15x  x 2 
324  6 15x  x  2

54  15x  x2
x2  15x  54  0
x2  18x  3x  54  0
x(x  18)  3(x  18)
 x  18  x  3  0
x  18 and x  3
As distance cannot be negative, x = 3
AC = 3 + 9 = 12
AB = AF + FB = 6 + x = 6 + 3 = 9

www.topperlearning.com 7
CBSE X | Mathematics

Board Paper – 2015 Solution All India Set – 3

9.
Given, the point s A(x,y), B( 5,7) and C( 4,5) are collinear.
So, the area formed by these vertices is 0.
1
  x 7  5   55  y    4  y  7    0
2
1
 2x  25  5y  4y  28  0
2
1
 2x  y  3  0
2
 2x  y  3  0
 y  2x  3

10. x2 – 2ax – (4b2 – a2) = 0


⇒ x2 + (2b – a)x – (2b + a)x – (4b2 – a2) = 0
⇒ x(x + 2b – a) – (2b + a)(x + 2b – a) = 0
⇒ (x + 2b – a)(x – 2b – a) = 0
⇒ (x + 2b – a) = 0, (x – 2b – a) = 0
 x = a − 2b, a + 2b

www.topperlearning.com 8
CBSE X | Mathematics

Board Paper – 2015 Solution All India Set – 3

SECTION C

11. Diameter of the tent = 4.2 m


Radius of the tent, r = 2.1 m
Height of the cylindrical part of tent, hcylinder = 4 m
Height of the conical part, hcone = 2.8 m
Slant height of the conical part, 

hcone2 r2

2.82 2.12
2.82 2.12
12.25 3.5 m
Curved surface area of the cylinder = 2𝜋r hcylinder
22
=2× × 2.1 × 4
7
= 22 × 0.3 × 8 = 52.8 m2
22
Curved surface area of the conical tent = 𝜋rl = × 2.1 × 3.5 = 23.1 m2
7
Total area of cloth required for building one tent
= Curved surface area of the cylinder + Curved surface area of the conical tent
= 52.8 + 23.1
= 75.9 m2
Cost of building one tent = 75.9 × 100 = Rs. 7590
Total cost of 100 tents = 7590 × 100 = Rs. 7,59,000
759000
Cost to be borne by the associations = = Rs. 3,79,500
2
It shows the helping nature, unity and cooperativeness of the associations.

www.topperlearning.com 9
CBSE X | Mathematics

Board Paper – 2015 Solution All India Set – 3

12. Internal diameter of the bowl = 36 cm


Internal radius of the bowl, r = 18 cm
2 2
Volume of the liquid, V = 𝜋r3 = × 𝜋 × 183
3 3
Let the height of the small bottle be ‘h’.
Diameter of a small cylindrical bottle = 6 cm
Radius of a small bottle, R = 3 cm
Volume of a single bottle = 𝜋R2h = 𝜋 × 32 × h
No. of small bottles, n = 72
10 2
Volume wasted in the transfer = × × 𝜋 × 183
100 3
Volume of liquid to be transferred in the bottles
2 10 2
= × 𝜋 × 183 − × × 𝜋 × 183
3 100 3
2 10
= × 𝜋 × 183 1
3 100
2 90
= × 𝜋 × 183 ×
3 100
Volume of the liquid to be transferred
Number of the small cylindrical bottles =
Volume of a single bottle
2 90
183
72 3 100
32 h
2 9
183
72 3 2 10
3 h
2 9
18 18 18
h 3 10
32 72
h 5.4 cm

Height of the small cylindrical bottle = 5.4 cm

www.topperlearning.com 10
CBSE X | Mathematics

Board Paper – 2015 Solution All India Set – 3

13. Side of the cubical block, a = 10 cm


Longest diagonal of the cubical block = a√3 = 10√3 cm
Since the cube is surmounted by a hemisphere, therefore the side of the cube should
be equal to the diameter of the hemisphere.
Diameter of the sphere = 10 cm
Radius of the sphere, r = 5 cm
Total surface area of the solid = Total surface area of the cube – Inner cross-section
area of the hemisphere + Curved surface area of the hemisphere
= 6a2 – 𝜋r2 + 2𝜋r2
= 6a2 + 𝜋r2
= 6 × (10)2 + 3.14 52
600 78.5 678.5 cm2
Total surface area of the solid = 678.5 cm2
Cost of painting 100 cm2 = Rs. 5
5
Cost of painting 1 cm2 = Rs.
100
5
Cost of painting the total surface area of the solid = × 678.5 = Rs. 33.925  Rs. 34.
100

www.topperlearning.com 11
CBSE X | Mathematics

Board Paper – 2015 Solution All India Set – 3

14. No. of cones = 504


Diameter of a cone = 3.5 cm
Radius of the cone, r = 1.75 cm
Height of the cone, h = 3 cm
Volume of a cone
1 2
rh
3
2
1 3.5
3
3 2
1 3.5 3.5
3 cm3
3 2 2
Volume of 504 cones
1 3.5 3.5
504 3cm3
3 2 2
Let the radius of the new sphere be ‘R’.
4 3
Volume of the sphere R
3
Volume of 504 cones = Volume of the sphere
1 3.5 3.5 4 3
504 3 R
3 2 2 3
504 1 3.5 3.5 3 3
R3
3 2 2 4
504 3 49
R3
64
7 8 9 3 72
R3
64
8 27 73
R3
64
2 3 7
R
4
21
R 10.5 cm
2
Radius of the new sphere = 10.5 cm
Surface area of the new sphere=4R 2
22 21 21
 4  
7 2 2
 2772 cm2

www.topperlearning.com 12
CBSE X | Mathematics

Board Paper – 2015 Solution All India Set – 3

15. √3 x2 – 2√2x – 2√3 = 0


Here a = √3, b = − 2√2, c = − 2√3
( 2 2)  ( 2 2)2  4  3  ( 2 3)
x 
2 3
2 2  8  24

2 3
2 2  32

2 3
2 24 2

2 3
2 2 4 2 2 2 4 2
 ,
2 3 2 3
3 2 2 2
 ,
3 2 3
2
x  6 , 
3

www.topperlearning.com 13
CBSE X | Mathematics

Board Paper – 2015 Solution All India Set – 3

16.
Let BC be the height at which the aeroplane is observed from point A.
Then, BC = 1500 3
In 15 seconds, the aeroplane moves from point A to D.
A and D are the points where the angles of elevation 60 and 30
are formed respectively.
Let BA = x metres and AD  y metres
BC = x + y

In CBA,
BC
tan60°=
BA
1500 3
3
x
 x  1500 m ....(1)

In CBD,
BC
tan30°=
BD
1 1500 3

3 xy
 x  y  1500(3)  4500
1500  y  4500
 y  3000 m ....(2)
We know that the aeroplane moves from point A to D in 15 seconds and the distance
covered is 3000 metres. (by 2)

www.topperlearning.com 14
CBSE X | Mathematics

Board Paper – 2015 Solution All India Set – 3

dis tance
Speed 
time
3000
Speed 
15
Speed  200m/s

18
Converting it to km/hr = 200   720 km/hr
5

17. Radius of the circle = 14 cm


Central Angle, 𝜽 = 60,
Area of the minor segment
1 2
r2 r sin
360 2
60 1
142 142 sin60
360 2
1 22 1 3
14 14 14 14
6 7 2 2
22 14
49 3
3
22 14 147 3
3 3
308 147 3 2
cm
3
308 147 3 2
Area of the minor segment cm
3

www.topperlearning.com 15
CBSE X | Mathematics

Board Paper – 2015 Solution All India Set – 3

18. Let the first term be ‘a’ and the common difference be ‘d’ of the A.P.
t13 = 4t3
⇒ a + 12d = 4(a + 2d)
⇒ a + 12d = 4a + 8d
⇒ 4d = 3a
4d
⇒a=
3
t5 = 16
⇒ a + 4d = 16
4d
⇒ + 4d = 16
3
4d  12d
⇒ = 16
3
16d
⇒ = 16
3
d=3
n
Sn = [2a + (n – 1)d]
2
10
S10 = [2 × 10 + (10 – 1) × 3]
2
= 5 [20 + 27]
= 5 × 47
= 235
Sum of the first 10 terms = 235.

19.
2
AP  AB
5
AP 2

AB 5
AP 2

PB 3
P divides AB in the ratio of 2 : 3 internally
 6  2  13 7  2  23 
 23 , 23 
 
 15 20 
 , 
 5 5 
 3,4 
Co-ordinates of P(3, 4)

www.topperlearning.com 16
CBSE X | Mathematics

Board Paper – 2015 Solution All India Set – 3

20. n(total balls in the bag) = x


2 20 number of black balls
P(getting a black ball) =  
5 50 total number of balls
Number of black balls in the bag =20
Total number of balls = 50
3 15 number of white balls
P(getting a white ball) =  
10 50 total number of balls
Number of white balls in the bag = 15
Number of red ball
= Total number of ball – (Number of black ball + Number of white ball)
= 50 – (20 + 15)
= 50 – 35
= 15
Total number of balls in the bag = 15.

www.topperlearning.com 17
CBSE X | Mathematics

Board Paper – 2015 Solution All India Set – 3

SECTION D
21.

Let PB be the surface of the lake and A be the point of observation such that
AP = 20 metres. Let C be the position of the cloud and C’ be its reflection in the lake.
Then CB = C’B. Let AM be perpendicular from A on CB.
Then mCAM  30 and mC'AM  60
Let CM = h. Then CB = h + 20 and C’B = h + 20. (CB=CB’ since refection about PB)
In CMA we have,
CM
tan30 
AM
1 h
 
3 AM
 AM  3h...................(i)

In AMC' we have,
C'M
tan 60 
AM
C'B  BM
 3
AM
h  20  20
 3
AM
h  20  20
 AM  ...................(ii)
3
From equation (i) and (ii), we get
h  20  20
3h 
3
 3h  h  40
 2h  40
 h  20 m
 AM  20 3
Now , to find AC u sin g pythagoras theorem
AC2  AM2  MC2
 1600
AC  40
Hence, the height of the cloud from the
surface of the lake is 40 metres.

www.topperlearning.com 18
CBSE X | Mathematics

Board Paper – 2015 Solution All India Set – 3

22.
Let S be the sample space of drawing a card from a well-shuffled deck.
n  S   52 C1  52

(i)There are 13 spade cards and 4 ace's in a deck


As ace of spade is included in 13 spade cards,
so there are 13 spade cards and 3 ace's

A card of spade or an ace can be drawn in 13


C1  3 C1  13  3  16
16 4
Probability of drawing a card of spade or an ace = 
52 13

(ii)There are 2 black king cards in a deck


A card of black king can be drawn in 2 C1  2
2 1
Probability of drawing a black king = 
52 26

(iii)There are 4 jack and 4 king cards in a deck.


So there are 52  8 = 44 cards which are neither jacks nor kings.
A card which is neither a jack nor a king can be drawn in 44
C1  44
44 11
Probability of drawing a card which is neither a jack nor a king = 
52 13

(iv)There are 4 king and 4 queen cards in a deck.


So there are 4  4 =8 cards which are either king or queen.
a card which is either a king or a queen can be drawn in 8 C1  8
8 2
Probability of drawing a card which is either a king or a queen = 
52 13

www.topperlearning.com 19
CBSE X | Mathematics

Board Paper – 2015 Solution All India Set – 3

23. PQRS is a square.


So each side is equal and angle between the adjacent sides is a right angle.
Also the diagonals perpendicularly bisect each other.
In PQR using pythagoras theorem,
PR 2 = PQ2 + QR 2
2 2
PR 2   42   42
PR = 2  42
1 42
OR  PR=  OQ
2 2
From the figure we can see that the radius of the flower bed ORQ is OR.
1
Area of sector ORQ  r 2
4
2
1  42 
 
4  2 
1
Area of the ROQ =  RO  OQ
2
1 42 42
  
2 2 2
2
 42 
 
 2 
Area of the flower bed ORQ
=Area of sector ORQ  Area of the ROQ
2 2
1  42   42 
=  
4  2   2 
2
 42    
     1
 2  2 
  441  0.57
 251.37cm2

Area of the flower bed ORQ = Area of the flower bed OPS
= 251.37cm2

Total area of the two flower beds


= Area of the flower bed ORQ + Area of the flower bed OPS
= 251.37  251.37
 502.74cm2

www.topperlearning.com 20
CBSE X | Mathematics

Board Paper – 2015 Solution All India Set – 3

24. Height of the cylinder (h) = 10 cm


Radius of the base of the cylinder = 4.2 cm
Volume of original cylinder = r2h
22 2
   4.2  10
7
 554.4 cm3
2
Volume of hemisphere = r3
3
2 22 3
    4.2
3 7
 155.232 cm3

Volume of the remaining cylinder after scooping out the hemisphere from each end
 Volume of original cylinder  2  Volume of hemisphere
 554.4  2  155.232
 243.936cm3

The remaining cylinder is melted and converted to


a new cylindrical wire of 1.4 cm thickness.
So they have same volume and radius of new cylindrical wire is 0.7 cm.
Volume of the remaining cylinder = Volume of the new cylindrical wire
243.936  r2h
22 2
243.936   0.7  h
7
h  158.4cm

 The length of the new cylindrical wire of 1.4 cm thickness is 158.4 cm.

www.topperlearning.com 21
CBSE X | Mathematics

Board Paper – 2015 Solution All India Set – 3

25. Let  be the length of the longer side and b be the length of the shorter side.
Given that the length of the diagonal of the rectangular field is 16 metres more than
the shorter side.
Thus, diagonal = 16 + b
Since longer side is 14 metres more than shorter side, we have,
= 14 + b
Diagonal is the hypotenuse of the triangle.
Consider the following figure of the rectangular field.

By applying Pythagoras Theorem in ABD, we have,


Diagonal2 Length2 Breadth2
(16 b)2 (14 b)2 b2
256 b2 32b 196 b2 28b b2
256 32b 196 28b b2
60 32b 28b b2
b2 4b 60 0
2
b 10b 6b 60 0
b(b 10) 6(b 10) 0
(b 6)(b 10) 0
(b 6) 0 or (b 10) 0
b 6 or b 10
As breadth cannot be negative, breadth = 10 m
Thus, length of the rectangular field = 14 + 10 = 24 m

www.topperlearning.com 22
CBSE X | Mathematics

Board Paper – 2015 Solution All India Set – 3

26. Consider the following diagram.

Let P be an external point and PA and PB are tangents to the circle.


We need to prove that PA = PB
Now consider the triangles OAP and OBP
mA = mB = 90
OP = OP [common]
OA = OB = radii of the circle
Thus, by Right Angle-Hypotenuse-Side criterion of congruence we have,
OAP  OBP
The corresponding parts of the congruent triangles are congruent.
Thus,
PA = PB

www.topperlearning.com 23
CBSE X | Mathematics

Board Paper – 2015 Solution All India Set – 3

27. In the figure, C is the midpoint of the minor arc PQ, O is the centre of the circle and
AB is tangent to the circle through point C.
We have to show that the tangent drawn at the midpoint of the arc PQ of a circle is
parallel to the chord joining the end points of the arc PQ.
We will show that PQ AB.
It is given that C is the midpoint point of the arc PQ.
So, arc PC = arc CQ.
PC = CQ

This shows that PQC is an isosceles triangle.


Thus, the perpendicular bisector of the side PQ of PQC passes through vertex C.
The perpendicular bisector of a chord passes through the centre of the circle.
So the perpendicular bisector of PQ passes through the centre O of the circle.
Thus perpendicular bisector of PQ passes through the points O and C.
PQ  OC
AB is the tangent to the circle through the point C on the circle.
AB  OC
The chord PQ and the tangent PQ of the circle are perpendicular to the same line OC.
PQ  AB.

www.topperlearning.com 24
CBSE X | Mathematics

Board Paper – 2015 Solution All India Set – 3

28. Let the original speed of the truck = s km/hr


New speed of the truck = (s + 20) km/hr
Time taken for 150 km + Time taken for 200 km = 5
150 200
 5
s (s  20)
150s  3000  200s
 5
s(s  20)
350s  3000
 5
s(s  20)
50(7s  60)
 5
s(s  20)
 10(7s  60)  s(s  20)
 70s  600  s2  20s
 s2  50s  600  0
 s2  60s  10s  600  0
 s(s  60)  10(s  60)  0
 (s  10)(s  60)  0
 s  10,60
s  10[Not possible]
 First speed of the truck = 60 km/hr

29. 5, 12, 19, …………50 terms


Common difference, d = 7
First term, a = 5
Last term, t50 = a + (50 – 1)d = 5 + (50 – 1) × 7 = 5 + 49 × 7 = 5 + 343 = 348
Sum of last 15 terms = S50 – S35
50 35
[2  5  (50  1)  7]  [2  5  (35  1)  7]
2 2
35
 25[10  343]  [10  34  7]
2
35
 25  353   248
2
 8825  4340
 4485
The sum of last 15 terms = 4485.

www.topperlearning.com 25
CBSE X | Mathematics

Board Paper – 2015 Solution All India Set – 3

30.
Steps for construction:
1. Draw a line segment AB = 5 cm.
2. At B construct mABC = 60
3. Take a measure of 6 cm, and draw an arc from B on BC.
4. Join AC to obtain ABC.
5. Below AB, make an acute angle BAX.
6. Since 7 > 5, mark off 7 points A1, A2, A3, A4, A5, A6 and A7 such that
AA1 = A1A2 = A2A3 = A3A4 = A4A5 = A5A6 = A6A7.
7. Join A7B.
5
8. Since we have to construct a triangle each of whose sides is of the
7
corresponding sides of ABC. So take five parts out of seven equal parts on AX.
i.e. from point A5, draw A5B' || A7B, meeting AB at B'.
9. From B', draw B'C' || BC, meeting AC at C'
10. AB'C' is the required, each of the sides is five-seventh of the corresponding
sides of ABC.

www.topperlearning.com 26
CBSE X | Mathematics

Board Paper – 2015 Solution All India Set – 3

31. A(k + 1, 2k) , B(3k, 2k + 3) and (5k – 1, 5k)


If 3 points are collinear then area of triangle formed by them = 0
1
(k  1)(2k  3  5k)  2k(3k  5k  1)  1(15k 2  10k 2  2k  15k  3)  0
2
1
 3k 2  3k  3k  3  4k 2  2k  15k 2  10k 2  2k  15k  3  0
2
1
6k 2  11k   0
2
6k 2  11k  0
11
k  0, 
6

www.topperlearning.com 27
CBSE X | Mathematics

Board Paper – 2016 Solution All India Set – 1

CBSE
Class X Mathematics
Board Paper – 2016 Solution
All India Set – 1
Time: 3 hours Total Marks: 90

SECTION A

1. In the given figure,


In ΔACO,
OA = OC …(Radii of the same circle)
 ΔACO is an isosceles triangle.
∠CAB = 30° …(Given)
 ∠CAO = ∠ACO = 30°
…(angles opposite to equal sides of an isosceles
triangle are equal)
∠PCO = 90° …(radius drawn at the point of contact is
perpendicular to the tangent)
Now ∠PCA = ∠PCO – ∠CAO
 ∠PCA = 90° – 30° = 60°

2. If k + 9, 2k – 1 and 2k + 7 are the consecutive terms of A.P., then the


common difference will be the same.
 (2k – 1) – (k + 9) = (2k + 7) – (2k – 1)
 k – 10 = 8
 k = 18

3. Let AB be the ladder and CA be the wall.


The ladder makes an angle of 60° with the horizontal.
 ΔABC is a 30°-60°-90°, right triangle.
Given: BC = 2.5 m, ∠ABC = 60°
 ∠BAC = 30°
 AB = 5 cm

4. There are 26 red cards including 2 red queens.


Two more queens along with 26 red cards will be 26 + 2 = 28
28
 P(getting a red card or a queen) 
52
28 24 6
 P(getting neither a red card nor a queen)  1   
52 52 13

www.topperlearning.com 1
CBSE X | Mathematics

Board Paper – 2016 Solution All India Set – 1

SECTION B

5. Given –5 is a root of the quadratic equation 2x2 + px – 15 = 0.


 5 satisfies the given equation.
 2  5  p  5   15  0
2

 50  5p  15  0
 35  5p  0
 5p  35  p  7
 
Substituting p  7 in p x2  x  k  0, we get

 2
7 x x k  0 
 7x2  7x  k  0
The roots of the equation are equal.
 Discriminant  b2  4ac  0
Here, a  7,b  7, c  k
b2  4ac  0
  7   4  7  k   0
2

 49  28k  0
 28k  49
49 7
k  
28 4

6. Since P and Q are the points of trisection of AB, AP = PQ = QB


Thus, P divides AB internally in the ratio 1 : 2
and Q divides AB internally in the ratio 2 : 1.
 By section formula,
 1  7   2 2  1  4   2  2    7  4 4  4   3 
P
 1  2
,
1  2
  
3
,
3    3 ,0    1,0 
     
 2  7   1 2  2  4   1  2    14  2 8  2   12 6 
Q ,    ,  ,   4,2 

 2 1 2 1   3 3   3 3 

7. Since tangents drawn from an exterior point to a circle are equal in length,
AP = AS ….(1)
BP = BQ ….(2)
CR = CQ ….(3)
DR = DS ….(4)
Adding equations (1), (2), (3) and (4), we get
AP + BP + CR + DS = AS + BQ + CQ + DS
 (AP + BP) + (CR + DR) = (AS + DS) + (BQ + CQ)
 AB + CD = AD + BC
 AB + CD = BC + DA …..(proved)

www.topperlearning.com 2
CBSE X | Mathematics

Board Paper – 2016 Solution All India Set – 1

8. Let A(3, 0), B(6, 4) and C(–1, 3) be the given points.


Now,

6  3   4  0   32  42  9  16  25
2 2
AB 

 1  6   3  4    7    1 
2 2 2 2
BC  49  1  50

 1  3  3  0    4 
2 2 2
AC   32  16  9  25
 AB  AC

   25
2
AB2  25

BC2  50   50

AC2  25   25

 AB2  AC2  BC2


Thus, ABC is a right-angled isosceles triangle.

9. 4th term of an A.P.= a4 = 0


 a + (4 – 1)d = 0
 a + 3d = 0
 a = –3d ….(1)
25 term of an A.P. = a25
th

= a + (25 – 1)d
= –3d + 24d ….[From (1)]
= 21d
3 times 11 term of an A.P. = 3a11
th

= 3[a + (11 – 1)d]


= 3[a + 10d]
= 3[–3d + 10d]
= 3 × 7d
= 21d
 a25 = 3a11
i.e., the 25th term of the A.P. is three times its 11th term.

www.topperlearning.com 3
CBSE X | Mathematics

Board Paper – 2016 Solution All India Set – 1

10. In the given figure,


OP = 2r … (Given)
∠OTP = 90° … (radius drawn at the point of contact is perpendicular to the
tangent)
In OTP,
OT 1
sinOPT =   sin30
OP 2
 OPT = 30
 ∠TOP = 60°
 ΔOTP is a 30°-60°-90°, right triangle.
In ΔOTS,
OT = OS … (Radii of the same circle)
 ΔOTS is an isosceles triangle.
 ∠OTS = ∠OST … (Angles opposite to equal sides of an isosceles triangle are
equal)

In ΔOTQ and ΔOSQ


OS = OT … (Radii of the same circle)
OQ = OQ ...(side common to both triangles)
∠OTQ = ∠OSQ … (angles opposite to equal sides of an isosceles triangle are
equal)
 ΔOTQ ≅ ΔOSQ … (By S.A.S)

 ∠TOQ = ∠SOQ = 60° … (C.A.C.T)

 ∠TOS = 120° … (∠TOS = ∠TOQ + ∠SOQ = 60° + 60° = 120°)


 ∠OTS + ∠OST = 180° – 120° = 60°
 ∠OTS = ∠OST = 60° ÷ 2 = 30°

www.topperlearning.com 4
CBSE X | Mathematics

Board Paper – 2016 Solution All India Set – 1

SECTION C

11. Diameter, AB = 13 cm
13
 Radius of the circle, r   6.5 cm
2
ACB is the angle in the semi-circle.
 ACB  90
Now, in ACB, using Pythagoras theorem, we have
AB2  AC2  BC2
 13  12   BC 
2 2 2

 BC   13  12   169  144  25


2 2 2

 BC  25  5 cm
Now, area of shaded region  Area of semi-circle  Area of ACB
1 1
 r2   BC  AC
2 2
1 1
  3.14   6.5   5  12
2

2 2
 66.33  30
 36.33 cm2
Thus, the area of the shaded region is 36.33 cm2.

12. For conical portion, we have


r = 1.5 m and l = 2.8 m
 S1 = Curved surface area of conical portion
 S1 = πrl
= π × 1.5 × 2.8
= 4.2π m2

For cylindrical portion, we have


r = 1.5 m and h = 2.1 m
 S2 = Curved surface area of cylindrical portion
 S2 = 2πrh
= 2 × π × 1.5 × 2.1
= 6.3π m2

Area of canvas used for making the tent  S1  S2


 4.2  6.3
 10.5
22
 10.5 
7
 33 m2
Total cost of the canvas at the rate of Rs. 500 per m2  Rs. 500  33  Rs. 16500

www.topperlearning.com 5
CBSE X | Mathematics

Board Paper – 2016 Solution All India Set – 1

13. P(x, y) is equidistant from the points A(a + b, b – a) and B(a – b, a + b).
 AP = BP
 x   a  b    y  b  a   x   a  b    y   a  b  
2 2 2 2

 x   a  b    y  b  a   x   a  b    y   a  b  


2 2 2 2

 x2  2x  a  b    a  b   y2  2y b  a  b  a
2 2

 x2  2x  a  b    a  b   y2  2y  a  b    a  b 
2 2

 2x  a  b   2y b  a  2x  a  b   2y  a  b 
 ax  bx  by  ay  ax  bx  ay  by
 2bx  2ay
 bx  ay ....(proved)

14. Area of the region ABDC = Area of sector AOC – Area of sector BOD
40 22 40 22
   14  14   77
360 7 360 7
1 1
  22  14  2   22  7  1
9 9
22
  28  7 
9
22
  21
9
154

3
 51.33 cm2
22 22
Area of circular ring   14  14  77
7 7
 22  14  2  22  7  1
 22  28  7 
 22  21
 462 cm2
 Re quired shaded region  Area of circular ring  Area of region ABDC
 462  51.33
 410.67 cm2
Thus, the area of shaded region is 410.67 cm2.

www.topperlearning.com 6
CBSE X | Mathematics

Board Paper – 2016 Solution All India Set – 1

15. Let a1, a2 be the first terms and d1, d2 the common differences of the two
given A.P’s.
n n
Then, we have Sn  2a1  n  1 d1  and Sn'  2a2  n  1 d2 
2 2
n
2a1  n  1 d1  2a  n  1 d
Sn
 '  2  1   1
2a2  n  1 d2  2a2  n  1 d2
Sn n
2
S 7n  1
It is given that n' 
Sn 4n  27
2a1  n  1 d1 7n  1
  ....(1)
2a2  n  1 d2 4n  27
To find the ratio of the mth terms of the two given A.P's,
replace n by (2m  1) in equation (1).
2a1  2m  1  1 d1 7 2m  1  1
 
2a2  2m  1  1 d2 4 2m  1  27
2a1  2m  2  d1 14m  7  1
 
2a2  2m  2  d2 8m  4  27
a1  m  1 d1 14m  6
 
a2  m  1 d2 8m  23
Hence, the ratio of the mth terms of the two A.P's is 14m  6 : 8m  23.

www.topperlearning.com 7
CBSE X | Mathematics

Board Paper – 2016 Solution All India Set – 1

1 1 2
16.  
 x  1 x  2  x  2 x  3 3


 x  3    x  1  2
 x  1 x  2  x  3 3
x 3  x 1 2
 
x 2

 2x  x  2  x  3  3
2x  4 2
 
x 2

 3x  2  x  3  3
2x  4 2
 3 2 2

x  3x  3x  9x  2x  6 3
2x  4 2
 3 2

x  6x  11x  6 3
 6x  12  2x3  12x2  22x  12
 2x3  12x2  16x  0

 2x x2  6x  8  0 

 x2  6x  8  0 
2
 x  4x  2x  8  0
 x  x  4  2  x  4  0
  x  4  x  2   0
 x  4  0 or x  2  0
x  4 or x  2

17. Let the radius of the conical vessel = r1 = 5 cm


Height of the conical vessel = h1 = 24 cm
Radius of the cylindrical vessel = r2
Let the water rise upto the height of h2 cm in the cylindrical vessel.
Now, volume of water in conical vessel = volume of water in cylindrical vessel
1
 r12h1  r22h2
3
 r12h1  3r22h2
 5  5  24  3  10  10  h2
5  5  24
 h2   2 cm
3  10  10
Thus, the water will rise upto the height of 2 cm in the cylindrical vessel.

www.topperlearning.com 8
CBSE X | Mathematics

Board Paper – 2016 Solution All India Set – 1

18. Radius of sphere = r = 6 cm


4 4
Volume of sphere  r 3    6   288 cm3
3

3 3
Let R be the radius of cylindrical vessel.
5 32
Reise in the water level of cylindrical vessel  h  3 cm  cm
9 9
32 32
Increase in volume of cylindrical vessel  R 2h  R 2   R 2
9 9
Now, volume of water displaced by the sphere is equal to volume of sphere.
32
 R 2  288
9
288  9
 R2   81
32
 R  9 cm
 Diameter of the cylindrical vessel  2  R  2  9  18 cm

19. Let CD be the hill and suppose the man is standing on the
deck of a ship at point A.
The angle of depression of the base C of the hill CD
observed from A is 30° and the angle of elevation of the top
D of the hill CD observed from A is 60°.
 ∠EAD = 60° and ∠BCA = 30°
In ΔAED,
DE
tan60 
EA
h
 3 
x
 h  3x ....(1)
In ABC,
AB
tan30 
BC
1 10
 
3 x
 x  10 3 ....(2)
Substituting x  10 3 in equation (1), we get
h  3  10 3  10  3  30
 DE  30 m
 CD  CE  ED  10  30  40 m
Thus, the distance of the hill from the ship is 10 3 m and
the height of the hill is 40 m.

www.topperlearning.com 9
CBSE X | Mathematics

Board Paper – 2016 Solution All India Set – 1

20. When three coins are tossed together, the possible outcomes are
HHH, HTH, HHT, THH, THT, TTH, HTT, TTT
 Total number of possible outcomes = 8
(i) Favourable outcomes of exactly two heads are HTH, HHT, THH
 Total number of favourable outcomes  3
3
 P(exactly two heads) 
8
(ii) Favourable outcomes of at least two heads are HHH, HTH, HHT, THH
 Total number of favourable outcomes  4
4 1
 P(at least two heads)  
8 2
(iii)Favourable outcomes of at least two tails are THT, TTH, HTT, TTT
 Total number of favourable outcomes  4
4 1
 P(at least two tails)  
8 2

SECTION D

21. Height of conical upper part = 3.5 m, and radius = 2.8 m


(Slant height of cone)2 2.12 2.82 4.41 7.84
Slant height of cone 12.25 3.5 m
The canvas used for each tent
curved surface area of cylindrical base curved surface area of conical upper part
2 rh rl
r(2h l)
22
2.8(7 3.5)
7
22
2.8 10.5
7
92.4 m2
So, the canvas used for one tent is 92.4 m2.
Thus, the canvas used for 1500 tents (92.4 1500) m2.
Canvas used to make the tents cos t Rs. 120 per sq. m
So, canvas used to make 1500 tents will
cos t Rs. 92.4 1500 120.
The amount shared by each school to set up the tents
92.4 1500 120
Rs.332640
50
The amount shared by each school to set up the tents is Rs.332640.
The value to help others in times of troubles is generated from the problem.

www.topperlearning.com 10
CBSE X | Mathematics

Board Paper – 2016 Solution All India Set – 1

22. Consider a circle centered at O.


Let PR and QR are tangents drawn from an external point R to the circle
touching at points P and Q respectively.
Join OR.

Proof:
In OPR and OQR,
OP = OQ ... (Radii of the same circle)
∠OPR = ∠OQR …. (Since PR and QR are
tangents to the circle)
OR = OR ... (Common side)
 OPR ≅ OQ R ….(By R.H.S)
 PR = QR ….(c.p.c.t)
Thus, tangents drawn from an external point to a circle are equal.

23. Steps of construction:


(i) Take a point O on the plane of the paper and draw a circle of radius
OA = 4 cm.
(ii) Produce OA to B such that OA = AB = 4 cm.
(iii) Draw a circle with centre at A and radius AB.
(iv) Suppose it cuts the circle drawn in step (i) at P and Q.
(v) Join BP and BQ to get the desired tangents.

Justification:
In OAP, OA = OP = 4 cm ...(radii of the same circle)
Also, AP = 4 cm ….(Radius of the circle with centre A)
 OAP is equilateral.
 ∠PAO = 60
 ∠BAP = 120
In BAP, we have BA = AP and ∠ BAP = 120
 ∠ABP = ∠APB = 30
Similarly we can get ∠ABQ = 30
 ∠PBQ = 60

www.topperlearning.com 11
CBSE X | Mathematics

Board Paper – 2016 Solution All India Set – 1

24. AO’ = O’X = XO = OC …..(Since the two circles are equal.)


So, OA = AO’ + O’X + XO …..(A-O’-X-O)
 OA = 3O’A
In AO'D and AOC,
DAO' = CAO ....(Common angle)
ADO' = ACO ....(both measure 90)
 ADO' ~ ACO ....(By AA test of similarity)
DO' O'A O'A 1
   
CO OA 3O'A 3

1 2 4
25.
x 1 x 2 x 4
L.C.M. of all the deno min ators is (x  1)(x  2)(x  4)
Multiply throughout by the L.C.M., we get
(x  2)(x  4)  2(x  1)(x  4)  4(x  1)(x  2)
 (x  4)(x  2  2x  2)  4(x2  3x  2)
 (x  4)(3x  4)  4x2  12x  8
 3x2  16x  16  4x2  12x  8
 x2  4x  8  0
Now, a  1,b  4, c  8
b  b2  4ac 4  16  32 4  48 4  4 3
x   
2a 2 2 2
x  2 2 3

26. MP = YX = 40 m
 QM  h  40
In right angled QMY,
QM h - 40
tan 45  1 ....(MY = PX)
MY PX
 PX  h  40 ....(1)
In right angled QPX,
QP QP
tan60   3 
PX PX
h
 PX  ....(2)
3
h
From (1) and (2), h  40 
3
 3h  40 3  h
 3h  h  40 3
 1.73h  h  40(1.73)  h  94.79 m
Thus, PQ is 94.79 m.

www.topperlearning.com 12
CBSE X | Mathematics

Board Paper – 2016 Solution All India Set – 1

27. Let there be a value of x such that the sum of the numbers of the houses
preceding the house numbered x is equal to the sum of the numbers of the
houses following it.
That is, 1  2  3  . . . .  (x - 1)  (x  1)  (x  2)  . . . . .  49
 1  2  3  . . . .  (x  1)
 [1  2  ......  x  (x  1)  ....  49]  (1  2  3  . . . .  x)
x 1 49 x
 [1  x  1]  [1  49]  [1  x]
2 2 2
 x(x  1)  49  50  x(1  x)
 x(x  1)  x(1  x)  49  50
 x2  x  x  x2  49  50
 x2  49  25
 x  7  5  35
Since x is not a fraction, the value of x satisfying the given condition exists
and is equal to 35.

AD AE 1
28.
AB AC 3
AB AC
  3
AD AE
AD  DB AE  EC
  3
AD AE
DB EC
1  1 3
AD AE
DB EC
  2
AD AE
AD AE 1
  
DB EC 2
 AD : DB  AE : EC  1 : 2
So, D and E divide AB and AC respectively in the ratio 1 : 2.
So the coordinates of D and E are
 1  8 5  12   17   7  8 2  12   14 
 1  2 , 1  2    3, 3  and  1  2 , 1  2    5, 3  respectively.
       

www.topperlearning.com 13
CBSE X | Mathematics

Board Paper – 2016 Solution All India Set – 1

Area of ADE
1  17 14   17 14 
=  4  3  3  3  5  6   3  6  5  3  4  3 
2    
1  68   85 56 
=   14  30   18  
2 3   3 3 
1  68  42  90   54  85  56 
= 
2  3   3 

1  200   195 
= 
2  3   3 
1 5
= 
2 3
5
 sq. units
6

4 1 7 4

6 5 2 6

Area of ABC
1
=  4  5  1  2  7  6   1  6  7  5  4  2 
2
1
= 20  2  42    6  35  8 
2
1
= 64  49
2
15
= sq. units
2
5
Area of ADE 1
  6 
Area of ABC 15 9
2

29. x is selected from 1,2,3 and 4


1,2,3, 4
y is selected from 1, 4,9 and 16
Let A  {1, 4,9,16,2,8,18,32,3,12,27, 48,36, 64} which consists
of elements that are product of x and y
Number of outcomes less than 16
P(product of x and y is less than 16) 
Total number of outcomes
7

14
1

2

www.topperlearning.com 14
CBSE X | Mathematics

Board Paper – 2016 Solution All India Set – 1

30. Perimeter of shaded region  AB  PB  arc length AP...(1)


 r
Arc length AP   2r  ....(2)
360 180
In right angled OAB,
AB
tan =  AB  r tan ....(3)
r
OB
sec  =  OB  r sec 
r
OB  OP  PB
 r sec   r  PB
 PB  r sec   r.....(4)
Substitute (2), (3) and (4) in (1), we get
Perimeter of shaded region  AB  PB  arc length AP
r
 r tan   r sec   r 
180
  
 r tan   sec   1
 180 

31. Let the speed of the stream be s km/h.

Speed of the motor boat  24 km / h


Speed of the motor boat upstream  24  s
Speed of the motor boat downstream  24  s
According to the given condition,
32 32
 1
24  s 24  s
 1 1 
 32    1
 24  s 24  s 
 24  s  24  s 
 32   1
 576 - s2 
2
 32  2s = 576  s
 s2  64s  576  0
 (s  72)(s  8)  0
 s  72 or s  8
Sin ce, speed of the stream cannot be negative, the speed of the
stream is 8 km / h.

www.topperlearning.com 15
CBSE X | Mathematics

Board Paper – 2017 All India Set – 3

CBSE
Class X Mathematics
Board Paper – 2017
All India Set – 3
Time: 3 hours Total Marks: 90

General Instructions:
(i) All questions are compulsory.
(ii) The question paper consists of 31 questions divided into four sections – A, B, C
and D.
(iii) Section A contains 4 questions of 1 mark each. Section B contains 6 questions
of 2 marks each, Section C contains 10 questions of 3 marks each and Section
D contains 11 questions of 4 marks each.
(iv) Use of calculators is not permitted.

SECTION A

Question numbers 1 to 4 carry 1 mark each.

1. The probability of selecting a rotten apple randomly from a heap of 900 apples
is 0.18. What is the number of rotten apples in the heap?

2. If a tower 30 m high, casts a shadow 10 3 m long on the ground, then what is


the angle of elevation of the sun?

3. If the angle between two tangents drawn from an external point P to a circle of
radius a and centre O, is 60°, then find the length of OP.

4. What is the common difference of an A.P. in which a21 – a7 = 84?

SECTION B

Question numbers 5 to 10 carry 2 marks each.

5. A circle touches all the four sides of a quadrilateral ABCD. Prove that
AB + CD = BC + DA

6. Prove that the tangents drawn at the end points of a chord of a circle make
equal angles with the chord.

7. A line intersects the y-axis and x-axis at the points P and Q respectively. If
(2, -5) is the mid-point of PQ, then find the coordinates of P and Q.

www.topperlearning.com 1
CBSE X | Mathematics

Board Paper – 2017 All India Set – 3

8. If the distances of P(x, y) from A(5, 1) and B(-1, 5) are equal, then prove that
3x = 2y.

9. Find the value of p, for which one root of the quadratic equation px2 – 14x + 8
= 0 is 6 times the other.

10. For what value of n, are the nth terms of two A.Ps 63, 65, 67,…. and 3, 10,
17,….. equal?

SECTION C

Question numbers 11 to 20 carry 3 marks each.

11. On a straight line passing through the foot of a tower, two points C and D are
at distances of 4 m and 16 m from the foot respectively. If the angles of
elevation from C and D of the top of the tower are complementary, then find
the height of the tower.

12. A bag contains 15 white and some black balls. If the probability of drawing a
black ball from the bag is thrice that of drawing a white ball, find the number
of black balls in the bag.

13. Three semicircles each of diameter 3 cm, a circle of diameter 4.5 cm and a
semicircle of radius 4.5 cm are drawn in the given figure. Find the area of the
shaded region.

 24 
14. In what ratio does the point  , y  divides the line segment joining the points
 11 
P(2, -2) and Q(3, 7)? Also find the value of y.

www.topperlearning.com 2
CBSE X | Mathematics

Board Paper – 2017 All India Set – 3

15. Water in a canal, 5.4 m wide and 1.8 m deep, is flowing with a speed of 25
km/hour. How much area can it irrigate in 40 minutes, if 10 cm of standing
water is required for irrigation?

16. In the given figure, two concentric circles with centre O have radii 21 cm and
 22 
42 cm. If ∠AOB = 60°, find the area of the shaded region.  Use  
 7 

17. The dimensions of a solid iron cuboid are 4.4 m × 2.6 m × 1.0 m. It is melted
and recast into a hollow cylindrical pipe of 30 cm inner radius and thickness 5
cm. Find the length of the pipe.

18. A toy is in the form of a cone of radius 3.5 cm mounted on a hemisphere of


same radius on its circular face. The total height of the toy is 15.5 cm. Find the
total surface area of the toy.

19. How many terms of an A.P. 9, 17, 25, …. must be taken to give a sum of 636?

20. If the roots of the equation (a2 + b2) x2 – 2(ac + bd) x + (c2 + d2) = 0 are
a c
equal, prove that  .
b d

SECTION D

Question numbers 21 to 31 carry 4 marks each.

21. If the points A(k + 1, 2k), B(3k, 2k + 3) and C(5k – 1, 5k) are collinear, then
find the value of k.

www.topperlearning.com 3
CBSE X | Mathematics

Board Paper – 2017 All India Set – 3

22. Construct a triangle ABC with side BC = 7 cm, ∠B = 45°, ∠A = 105°. Then
3
construct another triangle whose sides are times the corresponding sides of
4
the  ABC.

23. Two different dice are thrown together. Find the probability that the numbers
obtained have
(i) even sum, and
(ii) even product

24. In the given figure, XY and X’Y’ are two parallel tangents to a circle with centre
O and another tangents AB with point of contact C, is intersecting XY at A and
X’Y’ at B. Prove that ∠AOB = 90°.

25. In a rain–water harvesting system, the rain-water from a roof of 22 m × 20 m


drains into a cylindrical tank having diameter of base 2 m and height 3.5m. If
the tank is full, find the rainfall in cm. Write your views on water conservation.

26. Prove that the lengths of two tangents drawn from an external point to a circle
are equal.

27. If the ratio of the sum of the first n terms of two A.Ps is (7n + 1) : (4n + 27),
then find the ratio of their 9th terms.

28. Solve for x:


x  1 2x  1 1
  2,where x   ,1
2x  1 x  1 2

29. A takes 6 days less than B to do a work. If both A and B working together can
do it in 4 days, how many days will B take to finish it?

30. From the top of a tower, 100 m high, a man observe two cars on the opposite
sides of the tower and in same straight line with its base, with its base, with
angles of depression 30° and 45°. Find the distance between the cars.
 Take 3  1.732
 

www.topperlearning.com 4
CBSE X | Mathematics

Board Paper – 2017 All India Set – 3

31. In the given figure, O is centre of the circle with AC = 24 cm, AB = 7 cm and
∠BOD = 90°. Find the area of the shaded region.

www.topperlearning.com 5
CBSE X | Mathematics

Board Paper – 2017 Solution All India Set – 3

CBSE
Class X Mathematics
Board Paper – 2017 Solution
All India Set – 3
Time: 3 hours Total Marks: 90

SECTION A

1. Let the total number of rotten apples in a heap = n


Total number of apples in a heap = 900
Probability of selecting a rotten apple from a heap = 0.18
Now,
Number of rotten apples
P(selecting a rotten apple) 
Total number of apples
n
 0.18 
900
 n  0.18  900
 n  162
Hence, the number of rotten apples is 162.

2. Let AB be the tower and BC be its shadow.


AB  30 m, BC  10 3 m
In ABC,
AB
tan  
BC
30
 tan  
10 3
3
 tan  
3
tan   3
But, tan 60  3
   60
Thus, the angle of elevation of sun is 60.

www.topperlearning.com 1
CBSE X | Mathematics

Board Paper – 2017 Solution All India Set – 3

3. In the figure, PA and PB are two tangents from an external point P to a circle
with centre O and radius = a
APB  60 (given)
 APO  30 (tangents are equally inclined to the
line joining the point and the centre)
Now, OA  AP
In right-angled triangle OAP,
OA
sin 30 
OP
1 a
 
2 OP
 OP  2a

4. Let a be the first term and d be the common difference of the given A.P.
 a21  a7  84
  a  20d   a  6d  84
 a  20d  a  6d  84
 14d  84
d6
Hence, the common difference is 6.

SECTION B

5. Since tangents drawn from an external point to a circle are equal in length, we
have
AP = AS ….(i)
BP = BQ ….(ii)
CR = CQ ….(iii)
DR = DS ….(iv)
Adding (i), (ii), (iii) and (iv), we get
AP + BP + CR + DR = AS + BQ + CQ + DS
 (AP + BP) + (CR + DR) = (AS + DS) + (BQ + CQ)
 AB + CD = AD + BC
 AB + CD = BC + DA

www.topperlearning.com 2
CBSE X | Mathematics

Board Paper – 2017 Solution All India Set – 3

6. Let AB be a chord of circle with centre O.


Let AP and BP be two tangents at A and B respectively.
Suppose the tangents meet at point P. Join OP.
Suppose OP meets AB at C.
Now, in ΔPCA and ΔPCB,
PA = PB ….(tangents from an external point are
equal)
∠APC = ∠BPC ….(PA and PB are equally inclined to OP)
PC = PC ….(common)
Hence, ΔPAC  ΔPBC ….(by SAS congruence
criterion)
 ∠PAC = ∠PBC

7. Since a line is intersecting y-axis at P and x-axis at Q,


Coordinates of P = (0, y) and coordinates of Q = (x, 0)
Let R be the mid-point of PQ.
0  x y  0
Then, co-ordinates of R   ,  2, 5 
 2 2 
x y
  ,   2, 5
2 2
x y
  2 and  5
2 2
 x  4 and y  10
Hence, co-ordinates of P are 0,  10  and co-ordinates of Q are  4, 0  .

8. Given, P(x, y) is equidistant from A(5, 1) and B(–1, 5)


Now, AP  BP

5  x   1  y    1  x   5  y 
2 2 2 2

 5  x   1  y    1  x   5  y 
2 2 2 2

      
 25  x2  10x  1  y2  2y  1  x2  2x  25  y2  10y 
 x2  y2  10x  2y  26  x2  y2  2x  10y  26
 10x  2x  10y  2y
 12x  8y
 3x  2y ....(Dividin g throughout by  4)

www.topperlearning.com 3
CBSE X | Mathematics

Board Paper – 2017 Solution All India Set – 3

9. Given, px2 – 14x + 8 = 0


Here, a  p, b  14, c  8
Let  and  be the roots of the given quadratic equation.
Then,   6
b
Now, sum of the roots 
a
(14)
  
p
14
  
p
14
   6 
p
14
 7 
p
2
 ....(i)
p
c
Product of the roots 
a
8
  
p
8
   6 
p
8
 6 2 
p
4
 32 
p
2
2 4
 3   ....[From (i)]
p p
 4 4
 3 2  
p  p
p3

www.topperlearning.com 4
CBSE X | Mathematics

Board Paper – 2017 Solution All India Set – 3

10. For A.P. 63, 65, 67, ….., we have


first term = 63 and common difference = 65 – 63 = 2
Hence, nth term = an = 63 + (n – 1)2
 an = 63 + 2n – 2 = 2n + 61
For A.P. 3, 10, 17, ….., we have
first term = 3 and common difference = 10 – 3 = 7
Hence, nthterm  an'  3  n  1 7
 an'  3  7n  7  7n  4
The two A.Ps will have identical nth term, if
an  an'
 2n  61  7n  4
 5n  65
 n  13

www.topperlearning.com 5
CBSE X | Mathematics

Board Paper – 2017 Solution All India Set – 3

SECTION C

11.

Let AB be the tower with height h.


Let x be the angle of elevation from C.
So, the angle of elevation from D is 90  x  .
....(Sin ce the angles of elevation from C and D are complementary)
In CAB,
AB
tan x 
AC
h
 tan x  ......(i)
4
In DAB,
AB
tan(90  x) 
AD
h
 tan(90  x) 
16
h
 cot x  ......(ii)
16
From (i) and (ii),
h h
tan x  cot x  
4 16
h2
1
64
 h2  64
 h  64
h8 m
Hence, the height of the tower is 8 m.

www.topperlearning.com 6
CBSE X | Mathematics

Board Paper – 2017 Solution All India Set – 3

12. Let the number of black balls in the bag be x.


Number of white balls = 15
Hence, total number of balls in the bag = x + 15
Given, P(black ball)  3  P(white ball)
x 15
  3
x  15 x  15
x 45
 
x  15 x  15
 x  45
Thus, the number of black balls in the bag is 45.

13.

3
Radius of semi-circle A  cm  1.5 cm
2
3
Radius of semi-circle B  cm  1.5 cm
2
3
Radius of semi-circle C  cm  1.5 cm
2
4.5
Radius of circle D  cm  2.25 cm
2
Radius of semi-circle E  4.5 cm
Now, area of the shaded region
 Area of semi-circle (E  B)  Area of semi-circle (A  C)  Area of circle D
1  1
  4.5  1.5     1.5   1.5     2.25 
2 2 2 2 2

2   2  
1 1
  20.25  2.25   2.25  2.25   5.0625 
2 2
1 1
   22.50    4.50  5.0625
2 2
 11.25  2.25  5.0625
 3.9375
22
 3.9375 
7
 12.375 cm2

www.topperlearning.com 7
CBSE X | Mathematics

Board Paper – 2017 Solution All India Set – 3

14.
 24 
Suppose the point A  , y  divides the line segment joining points
 11 
P 2, 2  and Q 3,7  in the ratio k : 1.
 3k  2 7k  2 
Then, the coordinates of A are  , 
 k 1 k 1 
 24 
But, the coordinates of A are given as  , y .
 11 
3k  2 24
 
k 1 11
 33k  22  24k  24
2
 9k  2  k 
9
Hence, the ratio is 2 : 9.
2
7 2
7k  2 9
Also, y y
k 1 2
1
9
14  18
9 4 9 4
y   
29 9 11 11
9

15.
We have,
Width of the canal  5.4 m,
Depth of the canal  1.8 m
It is given that the water is flowing with a speed of 25 km / hr.
Therefore,
Length of the water column formed in 40 mins
40 2
that is, hours  hours
60 3
2 50 50  1000 50000
is 25  km  km  m m
3 3 3 3
2
 Volume of the water flowing in hours
3
50000
 Volume of the cuboid of length m, width 5.4 m and depth 1.8 m
3
2
 Volume of the water flowing in hours
3
50000
  5.4  1.8
3
 162000 m3
This volume  volume of cuboid (10 cm of standing water is required for irrigation)
This volume  base area of field  0.1m
162000
base area 
0.1
Hence, the cannal irrigates 1620000 m2 area in 40 mins

www.topperlearning.com 8
CBSE X | Mathematics

Board Paper – 2017 Solution All India Set – 3

16. We have,

Area of the region ABCD


 Area of sector AOB  Area of sector COD
 60 22 60 22 
   42  42    21  21  cm2
 360 7 360 7 
1 1 
   22  6  42   22  3  21  cm2
6 6 
 22  42  11  21 cm2
  924  231 cm2
 693 cm2
 22 22 
Area of circular ring    42  42   21  21  cm2
 7 7 
 22  6  42  22  3  21 cm2
 5544  1386  cm2
 4158 cm2
Hence, Required shaded region  Area of circular ring  Area of region ABCD
  4158  693 cm2
 3465 cm2

www.topperlearning.com 9
CBSE X | Mathematics

Board Paper – 2017 Solution All India Set – 3

17. Let the length of the pipe be h cm.


Then, volume of iron pipe = volume of iron in the block.
Volume of the block   4.4  2.6  1 m3   440  260  100  cm3
r  Internal radius of the pipe  30 cm
R  External radius of the pipe  (30  5) cm  35 cm
 Volume of the iron pipe  External Volume    Internal Volume 
 R 2h  r2h
 (R 2  r2 )h
 (R  r)(R  r)h
 (35  30)(35  30)h
   65  5  h
Now, Volume of iron in the pipe  Volume of iron in the block
   65  5  h  440  260  100
22
  65  5  h  440  260  100
7
440  260  100  7
h  11200 cm
22  65  5
 h  112 m
Thus, the length of the pipe is 112 m.

18. Radius of common base = 3.5 cm


Total height of toy  15.5 cm
Height of cone  15.5  3.5  12 cm
For cone,
l2  r2  h2
 l2  3.5  12 
2 2

 l2  12.25  144
 l2  156.25
 l  156.25  12.5 cm
 Total surface areaof the toy
 Curved surface area of cone  Curved surface area of hemisphere
 rl  2r 2
22 22
  3.5  12.5  2   3.5  3.5
7 7
22
  3.5 12.5  7
7
22
  3.5  19.5
7
 214.5 cm2

www.topperlearning.com 10
CBSE X | Mathematics

Board Paper – 2017 Solution All India Set – 3

19. Let there be n terms of this A.P.

For this A.P., a  9


d  a2  a1  17  9  8
n
Sn  2a  n  1 d
2
n
 636  2  9  n  1 8 
2
 636  n 9  n  1 4 
 636  n 9  4n  4 
 636  n  4n  5 
 4n2  5n  636  0
 4n2  53n  48n  636  0
 n  4n  53  12  4n  53   0
  4n  53 n  12   0
 4n  53  0 or n  12  0
53
n or n  12
4
Sin ce number of terms can neither be negative nor fractional,
we have n  12

20. We have
a2
 
 b2 x2  2  ac  bd x  c2  d2  0 
The discriminant of the given equation is given by

  
D   2  ac  bd   4  a2  b2  c2  d2 
2


 D  4  ac  bd  4 a2c2  a2d2  b2c2  b2d2
2

  
 D  4 a2c2  b2d2  2abcd  4 a2c2  a2d2  b2c2  b2d2 
 D  4  a c  b d  2abcd  a c
2 2 2 2 2 2
 a2d2  b2c2  b2d2 
 D  4 2abcd  a d  b c 
2 2 2 2

 D  4  ad  bc   2  ad bc  


2 2

 
 D  4  ad  bc 
2

The given equation will have equal roots,if D  0


 4(ad  bc)2  0
 (ad  bc)2  0
 ad  bc  0
 ad  bc
a c
 
b d

www.topperlearning.com 11
CBSE X | Mathematics

Board Paper – 2017 Solution All India Set – 3

SECTION D

21. Given points are A(k + 1, 2k), B(3k, 2k + 3) and C(5k – 1, 5k)
These points will be collinear, if area of the triangle formed by them is zero.
We have,

i.e.,
k 1 2k 3 3k 5k 5k 1 2k 3k 2k 5k 1 2k 3 k 1 5k 0

2k 2 5k 3 15k 2 10k 2 2k 6k 2 10k 2 13k 3 5k 2 5k 0

27k 2 3k 3 21k 2 18k 3 0


27k 2 3k 3 21k 2 18k 3 0
2
6k 15k 6 0
2
2k 5k 2 0
2
2k 4k k 2 0
(k 2)(2k 1) 0
k 2 0 or 2k 1 0
1
k 2 or k
2

www.topperlearning.com 12
CBSE X | Mathematics

Board Paper – 2017 Solution All India Set – 3

22. Steps of construction:


1) Draw BC  7 cm
2) At B, construct CBX  45 and at C construct BCY  180  (45  105)  30
3) Let BX and CY intersect at A. ABC so obtained is the given triangle.
4) Construct an acute angle CBZ at B on opposite side of vertex A of ABC.
5) Mark-off four points (greater of 4 and 3 in 3/4) points B1 ,B2 ,B3 ,B4 on BZ
such that BB1  B1B2  B2B3  B3B4
6) Join B4 to C.
7) Draw B3C ' parallel to B4C which meets BC at C'.
8) From C', draw C'A' parallel to CA meeting BA at A'.
3
Thus, A'BC' is the required triangle, each of whose sides is times
4
the corresponding sides of ABC.

www.topperlearning.com 13
CBSE X | Mathematics

Board Paper – 2017 Solution All India Set – 3

23. Elementary events associated to the random experiment of throwing two dice
are:
(1,1), (1,2), (1,3), (1, 4), (1,5), (1,6),
(2,1), (2,2), (2,3), (2, 4), (2,5), (2,6),
(3,1), (3,2), (3,3), (3, 4), (3,5), (3,6),
(4,1), (4,2), (4,3), (4, 4), (4,5), (4,6),
(5,1), (5,2), (5,3), (5, 4), (5,5), (5,6),
(6,1), (6,2), (6,3), (6, 4), (6,5), (6,6)
 Total number of elementary events  6  6  36
(i) Let A be the event of getting an even number as the product.
i.e., 2,4,6,8,10,12,
Elementary events favourable to event A are:
(1,1),(1,3),(1,5),(2,2),(2, 4),(2,6),(3,1),(3,3),(3,5),
(4,2),(4, 4),(4,6),(5,1),(5,3),(5,5),(6,2),(6, 4),(6,6)
 Total number of favourable events  18
18 1
Hence, required probability  
36 2
(ii) Let B be the event of getting an even number as the sum.
i.e., 2,4,6,8,10,12,16,18,20,24,30,36
Elementary events favourable to event B are:
(1,2),(1, 4),(1,6),(2,1),(2,2),(2,3),(2, 4),(2,5),
(2,6),(3,2),(3, 4),(3,6),(4,1),(4,2),(4,3),(4, 4),
(4,5),(4,6),(5,2),(5, 4),(5,6),(6,1),(6,2),(6,3),
(6, 4),(6,5),(6,6)
 Total number of favourable events  27
27 3
Hence, required probability  
36 4

www.topperlearning.com 14
CBSE X | Mathematics

Board Paper – 2017 Solution All India Set – 3

24. Since tangents drawn from an external point to a circle are equal.
Therefore, AP = AC.
Thus, in triangles AOP and AOC, we have
AP = AC
AO = AO [Common side]
OP = OC [Radii of the same circle]
So, by SSS- criterion of congruence,
we have
AOP  AOC
 PAO = CAO
 PAC = 2CAO
Similarly, we can prove that QBO = CBO
 CBQ = 2CBO
Now, PAC +CBQ = 1800
[sum of the interior angle on the same side of transversal is 180° ]

 2CAO +2 CBO = 180° [Using equations (i) and (ii)]


 CAO +CBO = 90°
 180°   AOB = 90° [Since CAO, CBO and AOB are angles of a
Triangle, CAO + CBO + AOB = 180°]
 AOB = 90°

25. We have,
r  Radius of cylindrical vessel  1m
h  Height of cylindrical vessel  3.5 m
22
 Volume of cylindrical vessel  r 2h   12  3.5 m3  11 m3
7
Let the rainfall be x m.
Then, Volume of the water
 Volume of cuboid of base 22 m  20 m and height x metres
 (22  20  x) m3
Since the vessel is just full of the water that drains out of the roof into the vessel,
Volume of the water  Volumeofthecylindricalvessel
 22  20  x  11
11 1 100
x  m cm  2.5 cm
22  20 40 40
Thus, the rainfall is 2.5 cm.

www.topperlearning.com 15
CBSE X | Mathematics

Board Paper – 2017 Solution All India Set – 3

26. Given: AP and AQ are two tangents from a point A to a circle C(O, r)
To prove: AP = AQ
Construction: Join OP, OQ and OA

Proof:

In OPA and OQA,


OPA  OQA  90 ....(T an gent at any point of a circle is perpendicular
to the radius through the point of contact)
OP  OQ ....(Radii of a circle)
OA  OA ....(Common)
Hence, by RHS-criterion of congruence, we have
OPA  OQA
 AP  AQ ....(c.p.c.t)

www.topperlearning.com 16
CBSE X | Mathematics

Board Paper – 2017 Solution All India Set – 3

27. Let a1, a2, be the first terms and d1, d2 the common differences of the two
given A.P’s.
Then, sum of their n terms is given by
n n
Sn  2a1  n  1 d1  and Sn'  2a2  n  1 d2 
2 2
n
2a1  n  1 d1  2a1  n  1 d1
Sn 2
 '  
Sn n
2a2  n  1 d2  2a2  n  1 d2
2
It is given that,
Sn 7n  1
'

Sn 4n  27
2a1  n  1 d1 7n  1
  ....(i)
2a2  n  1 d2 4n  27
In order to find the ratio of the mth terms of the two given A.P's,
we replace n by 2m  1 in equation (i).
Thus, to find the ratio of the 9th terms of the two given A.P's,
we replace n by 17 2  9  1 in equation (i)
2a1  17  1 d1 7  17  1

2a2  17  1 d2 4  17  27
2a1  16d1 120
 
2a2  16d2 95
a1  8d1 24
 
a2  8d2 19
Thus, the ratio of their 9th terms is 24:19.

www.topperlearning.com 17
CBSE X | Mathematics

Board Paper – 2017 Solution All India Set – 3

x 1 2x 1
28. 2
2x 1 x 1
2 2
x 1 2x 1
2
2x 1 x 1
x2 2x 1 4x2 4x 1
2
2x2 2x x 1
5x2 2x 2
2
2x2 x 1
5x2 2x 2 4x2 2x 2
5x2 2x 2 4x2 2x 2 0
x2 4x 4 0
2
x 2x 2x 4 0
x x 2 2 x 2 0
x 2 x 2 0
2
x 2 0
x 2 0
x 2

www.topperlearning.com 18
CBSE X | Mathematics

Board Paper – 2017 Solution All India Set – 3

29. Suppose B alone takes x days to finish the work.


Then, A alone can finish it in (x – 6) days
Now,
1 1
 A ' s one day's work   B ' s one day's work   x  6  x
1
And, (A  B)' s one day's work 
4
1 1 1
  
x6 x 4
xx6 1
 
x(x  6) 4

2x  6 1
 2

x  6x 4
 8x  24  x2  6x
 x2  6x  8x  24  0
 x2  14x  24  0
 x2  12x  2x  24  0
 x  x  12   2  x  12   0
  x  12   x  2   0
 x  12  0 or x  2  0
 x  12 or x  2
But, x cannot be less than 6.
So, x  12
Hence, B alone can finish the work in 12 days.

www.topperlearning.com 19
CBSE X | Mathematics

Board Paper – 2017 Solution All India Set – 3

30.

The man is at the top of the tower AB.


In right angled triangle ABX and ABY,
AB 100
tan 45  1  XB = 100 m
XB XB
AB 1 100
tan30     YB = 100 3
YB 3 YB
XY = XB + YB = 100+100 3  273.20 (approx)
Hence, the distance between X and Y is 273.20 m approximately.

www.topperlearning.com 20
CBSE X | Mathematics

Board Paper – 2017 Solution All India Set – 3

31. AC = 24 cm, AB = 7 cm

Since BC is the diameter of the circle,


so , BAC  90
In right BAC,
BC2  AC2  AB2
 BC2  242  72
 BC2  625
 BC  25 cm
So, the radius of the circle  OC  12.5 cm
Area of the shaded region
 Area of the circle  Area of BAC  Area of sector CD
1 
= r2   AB  AC   r 2
2 360
 22  1   90 22 
=  12.5  12.5     7  24      12.5  12.5 
 7  2   360 7 
....( BOD = 90  COD = 90)
 491.07  84  122.77
 284.3 cm2 (approximately)
Hence, the area of the shaded region is 284.3 cm2 approximately.

www.topperlearning.com 21
CBSE X | Mathematics

Board Paper 2012 - Solution

CBSE Board
Class X Mathematics
Board Paper Solution – 2012
Time: 3 hours Total Marks: 90

Section A

1. Correct answer: B

Let AB be the tower and BC be its shadow. Let  be the angle of elevation of
the sun.

According to the given information,

BC = 3 AB … (1)

In ABC,

AB AB 1
tan     [Using (1)]
BC 3AB 3

1
We know that tan 30 =
3

 = 30

Hence, the angle of elevation of the sun is 30.

www.topperlearning.com 1
CBSE X | Mathematics

Board Paper 2012 - Solution

2. Correct answer: B

Diameters of two circles are given as 10 cm and 24 cm.

Radius of one circle = r1 = 5 cm, Radius of other circle = r2 = 12 cm

According to the given information,

Area of the larger circle = (r1)2  (r2 )2

 (5)2  (12)2
 (25  144)
 169
 (13)2

Radius of larger circle = 13 cm

Hence, the diameter of larger circle = 26 cm

3. Correct answer: C

Let the original radius and the height of the cylinder be r and h respectively.

Volume of the original cylinder = r²h

r
Radius of the new cylinder =
2

Height of the new cylinder = h

2
r  r2h
Volume of the new cylinder =    h =
2 4

r2h
Volume of the new cylinder 1
Required ratio =  42   1 : 4
Volume of the original cylinder r h 4

4. Correct answer: C

When two dice are thrown together, the total number of outcomes is 36.

Favourable outcomes = {(1, 1), (2, 2), (3, 3), (4, 4), (5, 5), (6, 6)}

Number of favourable outcomes 6 1


Required probability =  
Total number of outcomes 36 6

www.topperlearning.com 2
CBSE X | Mathematics

Board Paper 2012 - Solution

5. Correct answer: B

It is given that the point P divides AB in the ratio 2: 1.

Using section formula, the coordinates of the point P are

 1  1  2  4 1  3  2  6   1  8 3  12 
 2 1
,
2 1    3 , 3   3,5
   

Hence the coordinates of the point P are (3, 5).

6. Correct answer: A

Let the coordinates of the other end of the diameter be (x, y).

We know that the centre is the mid-point of the diameter. So, O(-2, 5)
is the mid-point of the diameter AB. The coordinates of the point A and
B are (2, 3) and (x, y) respectively.

Using mid-point formula, we have,

2x
2   4  2  x  x  6
2
3y
5  10  3  y  y  7
2

Hence, the coordinates of the other end of the diameter are (-6, 7).

7. Correct answer: C

The first 20 odd numbers are 1, 3, 5, … … 39

This is an AP with first term 1 and the common difference 2.

Sum of 20 terms = S20

20
S20  2(1)  (20  1)(2)  10 2  38  400
2 

Thus, the sum of first 20 odd natural numbers is 400.

www.topperlearning.com 3
CBSE X | Mathematics

Board Paper 2012 - Solution

8. Correct answer: A

It is given that 1 is a root of the equations ay2 + ay + 3 = 0 and y2 + y + b


= 0.

Therefore, y = 1 will satisfy both the equations.

 a(1)2 + a(1) + 3 = 0

a+a+3=0

 2a + 3 = 0

3
 a
2

Also, (1)2 + (1) + b = 0

 1 + 1 + b = 0  b = 2

3
 ab =  2  3
2

9. Correct answer: B

It is known that the lengths of tangents drawn from a point outside a circle
are equal in length.

Therefore, we have:

AP = AR … (1) (Tangents drawn from point A)

BP = BQ … (2) (Tangents drawn from point B)

CQ = CR … (3) (Tangents drawn from point C)

Using the above equations,

AR = 4 cm (AP = 4 cm, given)

BQ = 3 cm (BP = 3 cm, given)

AC = 11 cm  RC = 11 cm – 4 cm = 7 cm

 CQ = 7 cm

Hence, BC = BQ + CQ = 3 cm + 7 cm = 10 cm

www.topperlearning.com 4
CBSE X | Mathematics

Board Paper 2012 - Solution

10. Correct answer: A

It is known that the tangents from an external point to the circle are equal.

EK = EM, DK = DH and FM = FH … (1)

Perimeter of EDF = ED + DF + FE

= (EK – DK) + (DH + HF) + (EM – FM)

= (EK – DH) + (DH + HF) + (EM – FH) [Using (1)]

= EK + EM

= 2 EK = 2 (9 cm) = 18 cm

Hence, the perimeter of EDF is 18 cm.

SECTION B

11. It is given that the point A (0, 2) is equidistant from the points B(3, p) and
C(p, 5).

So, AB = AC  AB2 = AC2

Using distance formula, we have

 (0  3)2  (2  p)2  (0  p)2  (2  5)2


 9  4  p2  4p  p2  9
 4  4p  0
 4p  4
p 1

Hence, the value of p = 1.

12. Total number of outcomes is 50.

Favourable outcomes = {12, 24, 36, 48}

Number of favourable outcomes 4 2


Required probability =  
Total number of outcomes 50 25

www.topperlearning.com 5
CBSE X | Mathematics

Board Paper 2012 - Solution

13.

1 4851
Given volume of a hemisphere = 2425 cm3  cm3
2 2
Now, let r be the radius of the hemisphere
2
Volume of a hemisphere  r3
3
2 3 4851
 r 
3 2
2 22 3 4851
  r 
3 7 2
3
4851 3 7  21 
 r3      
2 2 22  2 
21
 r  cm
2
So, Curved surface area of the hemisphere  2r2
11 3
22 21 21
 2  x x  693 sq.cm
7 2 2

14. Given: Tangents PA and PB are drawn from an external point P to two
concentric circles with centre O and radii OA = 8 cm, OB = 5 cm
respectively. Also, AP = 15 cm

Construction: We join the points O and P.

Solution: OA ⊥ AP ; OB ⊥ BP

[Using the property that radius is perpendicular to the


tangent at the point of contact of a circle]

In right angled triangle OAP,

OP2 = OA2 + AP2 [Using Pythagoras Theorem]

= (8)2 + (15)2 = 64 + 225 = 289

∴ OP = 17 cm

In right angled triangle OBP,

OP2 = OB2 + BP2

⇒ BP2 = OP2 - OB2 = (17)2 – (5)2 = 289 – 25 = 264

∴ BP = 264  2 66 cm

www.topperlearning.com 6
CBSE X | Mathematics

Board Paper 2012 - Solution

15. Given: ABC is an isosceles triangle, where AB = AC, circumscribing a circle.

To prove: The point of contact P bisects the base BC. i.e. BP = PC

Proof: It can be observed that

BP and BR; CP and CQ; AR and AQ are pairs of tangents drawn to the circle
from the external points B, C and A respectively.

Since the tangents drawn from an external point to a circle, then

BP = BR --- (i)

CP = CQ --- (ii)

AR = AQ --- (iii)

Given that AB = AC

⇒ AR + BR = AQ + CQ

⇒ BR = CQ [from (iii)]

⇒ BP = CP [from (i) and (ii)]

∴ P bisects BC.

OR

Given: The chord AB of the larger of the two concentric circles, with centre O,
touches the smaller circle at C.

To prove: AC = CB

Construction: Let us join OC.

Proof: In the smaller circle, AB is a tangent to the circle at


the point of contact C.

∴ OC ⊥ AB ------ (i)

(Using the property that the radius of a circle is perpendicular to the tangent
at the point of contact)

For the larger circle, AB is a chord and from (i) we have OC ⊥ AB

∴ OC bisects AB

(Using the property that the perpendicular drawn from the centre to a chord
of a circle bisects the chord)

∴ AC = CB

www.topperlearning.com 7
CBSE X | Mathematics

Board Paper 2012 - Solution

16. Given, OABC is a square of side 7 cm

i.e. OA = AB = BC = OC = 7cm

∴ Area of square OABC = (side)2 = 72 = 49 sq.cm

Given, OAPC is a quadrant of a circle with centre O.

∴ Radius of the sector = OA = OC = 7 cm.

Sector angle = 90o

90o
 Area of quadrant OAPC  x r2
o
360
1 22 2 77
 x x 7   sq.cm  38.5 sq.cm
4 7 2
 Area of shaded portion  Area of Square OABC  Area of quadrant OAPC
  49  38.5 sq. cm  10.5 sq.cm

17. First three- digit number that is divisible by 7 = 105

Next number = 105 + 7 = 112

Therefore the series is 105, 112, 119,…

The maximum possible three digit number is 999.

When we divide by 7, the remainder will be 5.

Clearly, 999 – 5 = 994 is the maximum possible three – digit number


divisible by 7.

The series is as follows:

105, 112, 119, …., 994

Here a = 105, d = 7

Let 994 be the nth term of this A.P.

www.topperlearning.com 8
CBSE X | Mathematics

Board Paper 2012 - Solution

an  a  n  1 d
 994  105  n  1 7
 n  1 7  889
 n  1  127
 n  128

So, there are 128 terms in the A.P.

n
 Sum  first term  last term
2
128
 a1  a128 
2
 64 105  994   64  1099   70336

18. Given quadratic equation is 3x2 – 2kx + 12 = 0

Here a = 3, b = -2k and c = 12

The quadratic equation will have equal roots if ∆ = 0

 b2  4ac  0
Putting the values of a,b and c we get

 2k 2  4 3 12   0


 4k2  144  0
 4k2  144
144
 k2   36
4
Considering square root on both sides,
k  36   6

Therefore, the required values of k are 6 and -6.

www.topperlearning.com 9
CBSE X | Mathematics

Board Paper 2012 - Solution

SECTION – C

19.

Let the co-ordinates of point P be (x, y)

Then using the section formula co-ordinates of P are.

4K  3 8K  5
x y
K 1 K 1

Since P lies on x+y=0

4K  3 8K  5
  0
K 1 K 1

2 1
 4K  2  0  k  K
4 2
1
Hence the value of K  .
2

20. The area of a triangle, whose vertices are (x1, y1), (x2, y2)
and (x3, y3) is

1
| x1 (y2 y3 ) x2 (y3 y1) x3 (y1 y2 ) |
2

Substituting the given coordinates

1
Area of |1(p 7) 4(7 3) ( 9)( 3 p) |
2

1
|(p 7) 40 27 9p | 15
2
10p 60 30
10p 30 or 10p 90
p 3. or p 9

www.topperlearning.com 10
CBSE X | Mathematics

Board Paper 2012 - Solution

21. Let ABCD be a parallelogram such that its sides touching a circle with centre O.
We know that the tangents to a circle from an exterior point are equal in
length.

AP = AS [From A] …(i)

BP = BQ [From B] …(ii)

CR = CQ [From C] …(iii)

and, DR = DS [From D] …(iv)

Adding (i), (ii), (iii) and (iv), we get

AP + BP + CR + DR = AS + BQ + CQ + DS

(AP + BP) + (CR + DR) = (AS + DS) + (BQ + CQ)

AB + CD = AD + BC

2AB = 2BC [ ABCD is a parallelogram AB=CD and BC = AD]

AB=BC

Thus, AB = BC = CD = AD

Hence, ABCD is a rhombus.

www.topperlearning.com 11
CBSE X | Mathematics

Board Paper 2012 - Solution

OR

A circle with centre O touches the sides AB, BC, CD, and DA of a
quadrilateral ABCD at the points P, Q, R and S respectively.

TO PROVE AOB COD 180o and, AOD BOC 180o

CONSTRUCTION Join OP, OQ, OR and OS.

PROOF Since the two tangents drawn from a external point to a circle
subtend equal angles at the centre.

1 2, 3 4, 5 6 and 7 8
Now, 1 2 3 4 5 6 7 8 360o

Sum of all the angles


subtended at a point is 360o

2( 2 3 6 7) 360o and 2( 1 8 4 5) 360o


( 2 3) ( 6 7) 180o and ( 1 8) ( 4 5) 180o

2 3 AOB, 6 7 COD
1 8 AOD and 4 5 BOC

AOB COD 180o


and AOD BOC 180o

Hence Proved

www.topperlearning.com 12
CBSE X | Mathematics

Board Paper 2012 - Solution

22. Given: radius of cyl=radius of cone=r=6cm

Height of the cylinder=height of the cone=h=7cm

Slant height of the cone= l

72 62 85 cm

Total surface area of the remaining solid

=curved surface area of the cylinder + area of the base of the cylinder +
curved surface area of the cone

(2 rh r2 rl)
22 22 22
2x 6 7 62 6 85
7 7 7
792 132
264 85
7 7
132
377.1 85 cm2
7

OR

Volume of the conical heap=volume of the sand emptied from the bucket.

Volume of the conical heap=

1 2 1 2
r h r x24 cm2 (height of the coneis 24)----------(1)
3 3
Volume of the sand in the bucket= r 2h
(18)2 32cm2 (2)
Equating 1 and 2
1 2
r x24 (18)2 32
3
(18)2 x32x3
r2
24
r 36cm

www.topperlearning.com 13
CBSE X | Mathematics

Board Paper 2012 - Solution

23. Area of the shaded region= Area of sector POQ - Area of sector AOB

Area of Shaded region ( R2 r2 )


360 360
30 22
x x(72 3.52 )
360 7
77
cm2
8

24.

4x2 4ax (a2 b2 ) 0


(4x2 4ax a2 ) b2 0
(2x)2 2.2x.a a2 b2 0

(2x a)2 b2 0

(2x a) b (2x a) b 0
(2x a) b 0 or (2x a) b 0
a b a b
x ;x
2 2

OR

3x2  2 6 x  2  0
 3x2  6 x  6 x 2 0
 3 x  3 x  2  2  3 x  2  0
   
  3x 2  3x 2 0 
 2
2
 3x  0

 3x 2  0
 3 x 2
2 2  3 6
 x   
 3
3 2 3

www.topperlearning.com 14
CBSE X | Mathematics

Board Paper 2012 - Solution

25. Given: Position of kite is B.

Height of kite above ground= 45 m

Angle of inclination = 60o

Required length of string = AB

In right angled triangle AOB,

OB 45 3 45
sin A   sin 60o   
AB AB 2 AB
45 x 2 90
 AB    30 3 m
3 3
Hence, the length of the string is 30 3 m

26. It is given that A = 105, C = 30.

Using angle sum property of triangle, we get, B = 45

The steps of construction are as follows:

1. Draw a line segment BC = 6 cm.

2. At B, draw a ray making an angle of 45 with BC.

3. At C, draw a ray making an angle of 30 with BC. Let the two rays meet at
point A.

4. Below BC, make an acute ∠CBX. Along BX mark off three points B1, B2, B3,
such that BB1 = B1B2 = B2B3. Join B3C.

7. From B2, draw B2C || B3C.

8. From C, draw CA || CA, meeting BA at the point A.

Then ABC is the required triangle.

www.topperlearning.com 15
CBSE X | Mathematics

Board Paper 2012 - Solution

27. Let a and d respectively be the first term and the common difference of the
AP.

We know that the nth term of an AP is given by an = a + (n – 1)d

According to the given information,

a16 = 1 + 2 a8

 a + (16 – 1)d = 1 + 2[a + (8 – 1)d]

 a + 15d = 1 + 2a + 14d

 a + 15d = 1 + 2a + 14d

 –a + d = 1 … (1)

Also, it is given that, a12 = 47

 a + (12 – 1)d = 47

 a + 11d = 47 … (2)

Adding (1) and (2), we have:

12d = 48

d=4

From (1),

–a + 4 = 1  a = 3

Hence, an = a + (n – 1)d = 3 + (n – 1)(4) = 3 + 4n – 4 = 4n – 1

Hence, the nth term of the AP is 4n – 1.

www.topperlearning.com 16
CBSE X | Mathematics

Board Paper 2012 - Solution

28. Total number of outcomes = 52

(i) Probability of getting a red king

Here the number of favourable outcomes = 2

No. of favourable outcomes 2 1


Probability
Total number of outcomes 52 26

(ii) Probability of getting a face card

Total number of face cards = 12

No. of favourable outcomes 12 3


Probability
Total number of outcomes 52 13

(iii) Probability of queen of diamonds

Number of queens of diamond = 1

No. of favourable outcomes 1


Probability
Total number of outcomes 52

SECTION – D

29. Here, R = 28 cm and r = 21 cm, we need to find h.

Volume of frustum = 28.49 L = 28.49×1000 cm3 = 28490 cm3

Now, Volume of frustum =


3

h 2
R  Rr  r2 

22h
73
 
282  28  21  212 =28490

22
 h  1813  28490
21
28490  21
h  15 cm
22  1813

Hence the height of bucket is 15 cm.

www.topperlearning.com 17
CBSE X | Mathematics

Board Paper 2012 - Solution

30. Let the height of hill is h.

In right triangle ABC,

50 50 1
 tan30    AB  50 3
AB AB 3

In right triangle ABD,

h h
 tan60   3  h  3AB
AB AB

 
 h  3 50 3  150 m

Hence the height of hill is 150 m.

31. Given: AB is a tangent to a circle with centre O.

To prove: OP is perpendicular to AB.

Construction: Take a point Q on AB and join OQ.

Proof: Since Q is a point on the tangent AB,


other than the point of contact P, so Q will
be outside the circle.

Let OQ intersect the circle at R.

Now OQ = OR + RQ

 OQ > OR  OQ > OP [as OR = OP]

 OP < OQ

Thus OP is shorter than any other segment among all and the shortest length
is the perpendicular from O on AB.

 OP  AB. Hence proved.

www.topperlearning.com 18
CBSE X | Mathematics

Board Paper 2012 - Solution

OR

Let ABCD be a quadrilateral, circumscribing a circle.

Since the tangents drawn to the circle from an external


point are equal, we have

AP = AS ... (1)

PB = BQ ... (2)

RC = QC ... (3)

DR = DS ... (4)

Adding, (1), (2), (3) and (4), we get

AP + PB + RC + DR = AS + BQ + QC + DS

(AP + PB) + (DR + RC) = (AS + SD) + (BQ + QC)

AB + CD = AD + BC.

32. Total cost of books = Rs 80

Let the number of books = x

80
So the cost of each book = Rs
x

80
Cost of each book if he buy 4 more book = Rs
x4

As per given in question:

80 80
 1
x x4
80x  320  80x
 1
x(x  4)
320
 2 1
x  4x
 x2  4x  320  0
 (x  20)(x  16)  0
 x  20,16

Since number of books cannot be negative,

So the number of books he brought is 16.


www.topperlearning.com 19
CBSE X | Mathematics

Board Paper 2012 - Solution

OR

Let the first number be x then the second number be 9 – x as the sum of
both numbers is 9.

1
Now the sum of their reciprocal is , therefore
2

1 1 1
 
x 9x 2
9xx 1
 
x(9  x) 2
9 1
 2

9x  x 2
 18  9x  x2
 x2  9x  18  0
 (x  6)(x  3)  0
 x  6,3

If x = 6 then other number is 3.

And, if x = 3 then other number is 6.

Hence numbers are 3 and 6.

33. Given: S20  240 and a  7

Consider, S20  240

20  n 
 2  7  19d  240  Sn  2 2a  (n  1)d
2  
 10(14  19d)  240
 14  19d  24
 19d  38
 d  2
Now, a24  a  23d  7  23  2  39
Hence, a24  39

www.topperlearning.com 20
CBSE X | Mathematics

Board Paper 2012 - Solution

34. Radius of hemi-sphere = 7 cm

Radius of cone = 7 cm

Height of cone = diameter = 14 cm

Volume of solid = Volume of cone + Volume of Hemi-sphere

1 2 2
 r h  r3
3 3
1 2
 r h  2r 
3
1 22
   49 14  14 
3 7
1 22
   49  28
3 7
22  7  28 4312
  cm3
3 3

Radius of cylinder = Radius of cone = r = 6 cm

Height of the cylinder = Height of the cone = h = 8 cm

Slant height of the cone= l 82 62 100 10 cm

Total surface area of the remaining solid

= Curved Surface Area of the Cylinder + Area of the Base of the Cylinder +
Curved Surface Area of the Cone

(2 rh r2 rl)
22 22 2 22
2x x6x7 x6 x6x 85
7 7 7
792 132
264 85
7 7
132
377.1 85cm2
7

www.topperlearning.com 21
CBSE X | Mathematics

Board Paper 2013 - Solution

CBSE Board
Class X Mathematics
Board Paper - 2013
Time: 3 hours Total Marks: 90

Section A

1. Correct answer: C

Let AB be the tower of height 75 m and C be the position of the car.

In ABC,

AC
cot 30o
AB
AC AB cot 30o
AC 75m 3
AC 75 3m

Thus, the distance of the car from the base of the tower is 75 3 m.

2. Correct answer: A

S = {1, 2, 3, 4, 5, 6}

Let event E be defined as 'getting an even number'.

n(E) = {1, 4, 6}

Number of favourable outcomes 3 1


P E
Number of possible outcomes 6 2

www.topperlearning.com 1
CBSE X | Mathematics

Board Paper 2013 - Solution

3. Correct answer: C

S = {1, 2, 3,..90}

n(S) = 90

The prime number less than 23 are 2, 3, 5, 7, 11, 13, 17, and 19.

Let event E be defined as 'getting a prime number less than 23'.

n(E) = 8

Number of favourable outcomes 8 4


P E
Number of possible outcomes 90 45

4. Correct answer: A

Given: AB, BC, CD and AD are tangents to the circle


with centre O at Q, P, S and R respectively.

AB = 29 cm, AD = 23, DS = 5 cm and B = 90o

Construction: Join PQ.

We know that, the lengths of the tangents drawn from an external point to a
circle are equal.

DS = DR = 5 cm

AR = AD - DR = 23 cm - 5 cm = 18 cm

AQ = AR = 18 cm

QB = AB - AQ = 29 cm - 18 cm = 11 cm

QB = BP = 11 cm

In PQB,

PQ2 = QB2 + BP2 = (11 cm)2 + (11 cm)2 = 2 (11 cm)2

PQ = 11 2 cm ... (1)

In OPQ,

PQ2 = OQ2 + OP2 = r2 + r2 = 2r2

(11 2 )2 = 2r2

121 = r2

www.topperlearning.com 2
CBSE X | Mathematics

Board Paper 2013 - Solution

r = 11

Thus, the radius of the circle is 11 cm.

5. Correct answer: B

AP PB (Given)

CA AP, CB BP (Since radius is perpendicular to tangent)

AC = CB = radius of the circle

Therefore, APBC is a square having side equal to 4 cm.

Therefore, length of each tangent is 4 cm.

6. Correct answer: C

From the figure, the coordinates of A, B, and C are (1, 3), (-1, 0) and (4, 0)

respectively.

Area of  ABC

1
 | 1(0  0)  (1)(0  3)  4(3  0) |
2
1
 | 0  3  12 |
2
1
 | 15 |
2
 7.5 sq units

www.topperlearning.com 3
CBSE X | Mathematics

Board Paper 2013 - Solution

7. Correct answer: B

Let r be the radius of the circle.

From the given information, we have

2 r - r = 37 cm

r 2 1 37 cm
22
r 2 1 37 cm
7
37
r 37 cm
7
r 7 cm

22
Circumference of the circle = 2 r 2 7 cm 44 cm
7

8. Correct answer: C

1 6q 1 1 6q 1 6q
Common difference = 2
3q 3q 3q 3q

9. Given: ABCD be a parallelogram circumscribing a circle with centre O.

To prove: ABCD is a rhombus.

We know that the tangents drawn to a circle from an


exterior point are equal in length.

Therefore, AP = AS, BP = BQ, CR = CQ and DR = DS.

Adding the above equations,

AP + BP + CR + DR = AS + BQ + CQ + DS

(AP + BP) + (CR + DR) = (AS + DS) + (BQ + CQ)

AB + CD = AD + BC

2AB = 2BC

(Since, ABCD is a parallelogram so AB = DC and AD = BC)

AB = BC

Therefore, AB = BC = DC = AD.

Hence, ABCD is a rhombus.

www.topperlearning.com 4
CBSE X | Mathematics

Board Paper 2013 - Solution

10. Dimension of the rectangular card board = 14 cm 7 cm

Since, two circular pieces of equal radii and maximum area touching each
other are cut from the rectangular card board, therefore, the diameter of each
14
of each circular piece is = 7 cm.
2

7
Radius of each circular piece = cm.
2

2
7 22 49
Sum of area of two circular pieces = 2 2 77cm2
2 7 4

Area of the remaining card board

= Area of the card board - Area of two circular pieces

= 14 cm 7 cm - 77 cm2

= 98 cm2 - 77 cm2

= 21 cm2

11. Given: AB = 12 cm, BC = 8 cm and AC = 10 cm.

Let, AD = AF = x cm, BD = BE = y cm and CE = CF = z cm

(Tangents drawn from an external point to the circle are equal in length)

2(x + y + z) = AB + BC + AC = AD + DB + BE + EC + AF + FC = 30 cm

x + y + z = 15 cm

AB = AD + DB = x + y = 12 cm

 z = CF = 15 - 12 = 3 cm

AC = AF + FC = x + z = 10 cm

 y = BE = 15 - 10 = 5 cm

 x = AD = x + y + z - z - y = 15 - 3 - 5 = 7 cm

www.topperlearning.com 5
CBSE X | Mathematics

Board Paper 2013 - Solution

12. Three digit numbers divisible by 7 are

105, 112, 119, … 994

This is an AP with first term (a) = 105 and common difference (d) = 7

Let an be the last term.

an = a + (n - 1)d

994 = 105 + (n - 1)(7)

7(n - 1) = 889

n - 1 = 127

n = 128

Thus, there are 128 three-digit natural numbers that are divisible by 7.

13.

4 3x2 5x 2 3 0
2
4 3x 8x 3x 2 3 0
4x 3x 2 3 3x 2 0

4x 3 3x 2 0

3 2
x or x
4 3

14. Let E be the event that the drawn card is neither a king nor a queen.

Total number of possible outcomes = 52

Total number of kings and queens = 4 + 4 = 8

Therefore, there are 52 - 8 = 44 cards that are neither king nor queen.

Total number of favourable outcomes = 44

Favourable outcomes 44 11
Required probability = P(E) =
Total number of outcomes 52 13

www.topperlearning.com 6
CBSE X | Mathematics

Board Paper 2013 - Solution

15. Let the radius and height of cylinder be r cm and h cm respectively.

Diameter of the hemispherical bowl = 14 cm

Radius of the hemispherical bowl = Radius of the cylinder

14
=r cm 7cm
2

Total height of the vessel = 13 cm

Height of the cylinder, h = 13 cm - 7 cm = 6 cm

Total surface area of the vessel = 2(curved surface area of the cylinder +
curved surface area of the hemisphere)

(Since, the vessel is hollow)

22
2 2 rh 2 r2 4 r h r 4 7 6 7 cm2
7

= 1144 cm2

16. Height of the cylinder, h = 10 cm

Radius of the cylinder = Radius of each hemisphere = r = 3.5 cm

Volume of wood in the toy = Volume of the cylinder - 2


Volume of each hemisphere

2 3
r2h 2 r
3
4
r2 h r
3
22 4
(3.5)2 10 3.5
7 3
38.5 10 4.67
38.5 5.33
205.205 cm3

Radius = 21 cm

www.topperlearning.com 7
CBSE X | Mathematics

Board Paper 2013 - Solution

17. The arc subtends an angle of 60o at the centre.

60o 22
(i) l = o
2 r o
2 21 cm 22 cm
360 360 7

60o 22
(ii) Area of the sector o
r2 o
21 21 cm2 = 231 cm2
360 360 7

18. AB and CD are the diameters of a circle with centre O.

OA = OB = OC = OD = 7 cm (Radius of the circle)

Area of the shaded region

= Area of the circle with diameter OB + (Area of the semi-circle ACDA - Area
of ACD)

2
7 1 2 1
7 CD OA
2 2 2
22 49 1 22 1
49 14 7
7 4 2 7 2

77
77 49
2
66.5 cm2

19. Let the y-axis divide the line segment joining the points (-4, -6) and (10, 12)
in the ratio k: 1 and the point of the intersection be (0, y).

Using section formula, we have

10k 4 12k 6
, 0, y
k 1 k 1
10k 4 4 2
0 10k 4 0 k
k 1 10 5
Thus, the y-axis divides the line segment joining the
given points in the ratio 2:5.
2 24 30
12k 6 12 6
5 5 6
y
k 1 2 2 5 7
1
5 5

6
Thus, the coordinates of the point of division are 0, .
7

www.topperlearning.com 8
CBSE X | Mathematics

Board Paper 2013 - Solution

20.

Let AB and CD be the two poles, where CD (the second pole) = 24 m.

BD = 15 m

Let the height of pole AB be h m.

AL = BD = 15 m and AB = LD = h

So, CL = CD - LD = 24 – h

In ACL,

CL
tan30o
AL
24 h
tan30o
15
1 24 h
3 15
15
24 h 5 3
3
h 24 5 3
h 24 5 1.732 Taking 3 1.732
h 15.34

Thus, height of the first pole is 15.34 m.

www.topperlearning.com 9
CBSE X | Mathematics

Board Paper 2013 - Solution

21. k + 4)x2 + (k + 1)x + 1 = 0

a = k + 4, b = k + 1, c = 1

For equal roots, discriminant, D = 0

b2 - 4ac = 0

(k + 1)2 - 4(k + 4) 1=0

k2 + 2k + 1 - 4k - 16 = 0

k2 - 2k - 15 = 0

k2 - 5k + 3k - 15 = 0

k(k - 5) + 3(k - 5) = 0

(k - 5) (k + 3) = 0

k = 5 or k = -3

Thus, for k = 5 or k = -3, the given quadratic equation has equal roots.

22. Sn = 3n2 + 4n

First term (a1) = S1 = 3(1)2 + 4(1) = 7

S2 = a1 + a2 = 3(2)2 + 4(2) = 20

a2 = 20 - a1 = 20 - 7 = 13

So, common difference (d) = a2 - a1 = 13 - 7 = 6

Now, an = a + (n - 1)d

 a25 = 7 + (25 - 1) 6 = 7 + 24 6 = 7 + 144 = 151

www.topperlearning.com 10
CBSE X | Mathematics

Board Paper 2013 - Solution

23.

Steps of construction:

1. Draw two concentric circle with centre O and radii 4 cm and 6 cm. Take a
point P on the outer circle and then join OP.

2. Draw the perpendicular bisector of OP. Let the bisector intersects OP at M.

3. With M as the centre and OM as the radius, draw a circle. Let it intersect the
inner circle at A and B.

4. Join PA and PB.

Therefore, PA and PB are the required tangents.

www.topperlearning.com 11
CBSE X | Mathematics

Board Paper 2013 - Solution

24. The given points are A(-2,3) B(8,3) and C(6,7).

Using distance formula, we have:

2 2
AB2 = 8 2 3 3

AB2 = 102 + 0

AB2 = 100

2 2
BC2 = 6 8 7 3

BC2 = (-2)2 + 42

BC2 = 4 + 16

BC2 = 20

2 2
CA2 2 6 3 7

CA = (-8)2 + (-4)2

CA2 = 64 + 16

CA2 = 80

It can be observed that:

BC2 + CA2 = 20 + 80 = 100 = AB2

So, by the converse of Pythagoras Theorem,

ABC is a right triangle right angled at C.

25. Diameter of circular end of pipe = 2 cm

2
Radius r1 of circular end of pipe = m 0.01 m
200

2
Area of cross-section = r12 0.01 0.0001 m2

Speed of water = 0.4 m/s = 0.4 60 = 24 metre/min

Volume of water that flows in 1 minute from pipe =


24 0.0001 m3 0.0024 m3

Volume of water that flows in 30 minutes from pipe =

www.topperlearning.com 12
CBSE X | Mathematics

Board Paper 2013 - Solution

30 0.0024 m3 0.072 m3

Radius (r2) of base of cylindrical tank = 40 cm = 0.4 m

Let the cylindrical tank be filled up to h m in 30 minutes.

Volume of water filled in tank in 30 minutes is equal to the volume of water


flowed out in 30 minutes from the pipe.

r22 h 0.072
2
0.4 h 0.072
0.16 h 0.072
0.072
h
0.16
h 0.45 m 45 cm

Therefore, the rise in level of water in the tank in half an hour is 45 cm.

26. The group consists of 12 persons.

Total number of possible outcomes = 12

Let A denote event of selecting persons who are extremely patient

Number of outcomes favourable to A is 3.

Let B denote event of selecting persons who are extremely kind or honest.

Number of persons who are extremely honest is 6.

Number of persons who are extremely kind is 12 - (6 + 3) = 3

Number of outcomes favourable to B = 6 + 3 = 9.

Number of outcomes favrouableto A 3 1


(i) P A
Total number of possible outcomes 12 4

Number of outcomes favorableto B 9 3


(ii) P B
Total number of possible outcomes 12 4

Each of the three values, patience, honesty and kindness is important in


one's life.

www.topperlearning.com 13
CBSE X | Mathematics

Board Paper 2013 - Solution

27. Diameter of upper end of bucket = 30 cm

Radius (r1) of upper end of bucket = 15 cm

Diameter of lower end of bucket = 10 cm

Radius (r2) of lower end of bucket = 5 cm

Slant height (l) of frustum

2
= r1 r2 h2

2 2 2 2
15 5 24 10 24 100 576
676 26cm

Area of metal sheet used to make the bucket

r1 r2 l r22
2
15 5 26 5

520 25 545 cm2

Cost of 100 cm2 metal sheet = Rs 10

Cost of 545 cm2 metal sheet

545 3.14 10
= Rs. Rs.171.13
100

Therefore, cost of metal sheet used to make the bucket is Rs 171.13.

www.topperlearning.com 14
CBSE X | Mathematics

Board Paper 2013 - Solution

28. Given: l and m are two parallel tangents to the circle with centre O touching
the circle at A and B respectively. DE is a tangent at the point C, which
intersects l at D and m at E.

To prove: DOE 90o

Construction: Join OC.

Proof:

In ODA and ODC,

OA = OC (Radii of the same circle)

AD = DC (Length of tangents drawn from an external point to a circle are


equal)

DO = OD (Common side)

ODA ODC (SSS congruence criterion)

DOA COD ... (1)

Similarly, OEB OEC

EOB COE ... (2)

Now, AOB is a diameter of the circle. Hence, it is a straight line.

DOA COD COE EOB 180o

From (1) and (2), we have:

2 COD + 2 COE = 180o

COD COE 90o


DOE 90o

Hence, proved.

www.topperlearning.com 15
CBSE X | Mathematics

Board Paper 2013 - Solution

29. Let the sides of the two squares be x cm and y cm where x > y.

Then, their areas are x2 and y2 and their perimeters are 4x and 4y.

By the given condition:

x2 + y2 = 400 … (1)

and 4x - 4y = 16

4(x - y) = 16 x-y=4

x=y+4 ... (2)

Substituting the value of x from (2) in (1), we get:

(y + 4)2 + y2 = 400

y2 + 16 + 8y + y2 = 400

2y2 + 16 + 8y = 400

y2 + 4y - 192 = 0

y2 + 16y - 12y - 192 = 0

y(y + 16) - 12 (y + 16) = 0

(y + 16) (y - 12) = 0

y = -16 or y = 12

Since, y cannot be negative, y = 12.

So, x = y + 4 = 12 + 4 = 16

Thus, the sides of the two squares are 16 cm and 12 cm.

www.topperlearning.com 16
CBSE X | Mathematics

Board Paper 2013 - Solution

30.

1 1 1 1
2a b
2x 2a b 2x
1 1 1 1
2a b 2x 2x 2a b
2x 2a b 2x b 2a
2x 2a b 2x 2ab
2a b b 2a
2x 2a b 2x 2ab

1 1
x 2a b 2x ab
2x2 2ax bx ab 0
2x x a b x a 0
x a 2x b 0
x a 0 or 2x b 0
b
x a, or x
2

31. Given: A circle with centre O and a tangent XY to the circle at a point P

To Prove: OP is perpendicular to XY.

Construction:
Take a point Q on XY other than P and join
OQ.

Proof:
Here the point Q must lie outside the circle as
if it lies inside the tangent XY will become secant to the circle.

Therefore, OQ is longer than the radius OP of the circle, That is, OQ > OP.
This happens for every point on the line XY except the point P.

So OP is the shortest of all the distances of the point O to the points on XY.

And hence OP is perpendicular to XY.

Hence, proved.

www.topperlearning.com 17
CBSE X | Mathematics

Board Paper 2013 - Solution

32. Given A.P. is -12, -9, -6, ..., 21

First term, a = -12

Common difference, d = 3

Let 21 be the nth term of the A.P.

21 = a + (n - 1)d

21 = -12 + (n - 1) 3

33 = (n - 1) 3

n = 12

Sum of the terms of the A.P. = S12

n 12
= 2a n 1 d 24 11 3 54
2 2

If 1 is added to each term of the AP, the sum of all the terms of the new AP
will increase by n, i.e., 12.

 Sum of all the terms of the new AP = 54 + 12 = 66

33. Let AC and BD be the two poles of the same height h m.

Given AB = 80 m

Let AP = x m, therefore, PB = (80 - x) m

In  APC,

AC
tan30o
AP
1 h
... (1)
3 x

In  BPD,

BD
tan60o
AB
h
3 ... (2)
80 x

www.topperlearning.com 18
CBSE X | Mathematics

Board Paper 2013 - Solution

Dividing (1) by (2),


1 h
3 x
3 h
80 x
1 80 x
3 x
x 240 3x
4x 240
x 60
From (1),
1 h
3 x
60
h 20 3m
3

Thus, the height of both the poles is 20 3m and the distances of the point
from the poles are 60 m and 20 m.

34. The given vertices are A(x, y), B(1, 2) and C(2, 1).

It is know that the area of a triangle whose vertices are (x1, y1), (x2, y2) and
(x3, y3) is given by

1
| x1 y2 y3 x2 y3 y1 x3 y1 y2 |
2

 Area of ABC

1
|x 2 1 1 1 y 2 y 2 |
2
1
| x 1 y 2y 4 |
2
1
| x y 3|
2

The area of ABC is given as 6 sq units.

1
x y 3 6 x y 3 12
2
x y 15

www.topperlearning.com 19
CBSE X | MATHEMATICS

Board Paper ˗ 2014

CBSE Board
Class X Summative Assessment – II
Mathematics
Board Question Paper 2014
Time: 3 hrs Max. Marks: 90

Solution
Section A

1. Correct answer: C

The multiples of 4 between 1 and 15 are 4, 8 and 12. The probability of getting a
3 1
multiple of 4 = 
15 5

2. Correct answer: A

Let AB be the tower and BC be distance between tower and car. Let  be the angle of
depression of the car.

According to the given information,

In ABC,
BC 1
tan   [Using (1)] and tan 30 =
AB 3

150 150 3
 BC =   50 3
3 3

Hence, distance between the tower and car is 50 3 .

www.topperlearning.com 1
CBSE X | MATHEMATICS

Board Paper ˗ 2014

3. Correct answer: D

TA  TP  TAP  TPA
TB  TP  TBP  TPB
TAP  TBP=TPA +TPB=APB
TAP  TBP  APB  180  sum of.....180
APB  APB  180
2APB  180
APB  90

4. Correct answer: B

k, 2k - 1, 2k + 1 are in Arithmetic Progression

2  (2k - 1) = k + 2k +1

4k - 2 = 3k +1

k=3

www.topperlearning.com 2
CBSE X | MATHEMATICS

Board Paper ˗ 2014

5. Correct answer: B

Given  AOB is given as 90 

ΔAOB is an isosceles triangle since OA = OB

Therefore  OAB =  OBA = 45 

Thus  AOP = 45  and  BOP = 45 

Hence ∆AOP and ∆BOP also are isosceles triangles

Thus let AP = PB = OP = x

Using Pythagoras theorem

x 2 + x 2 = 102

Thus 2 x 2 = 100

x =5 2

Hence length of chord AB = 2 x = 10 2

www.topperlearning.com 3
CBSE X | MATHEMATICS

Board Paper ˗ 2014

6. Correct answer: A

We see that AB = 4 units and BC = 3 units

Using Pythagoras theorem

AC2  AB2  BC2

= 42  32

AC2  25

Thus AC = 5 units
Hence length of the diagonal of the rectangle is 5 units

www.topperlearning.com 4
CBSE X | MATHEMATICS

Board Paper ˗ 2014

7. Correct answer: C

It is given that AB = 5 and BC = 12

Using Pythagoras theorem

AC2  AB2  BC2

= 52  122

= 169

Thus AC = 13

We know that two tangents drawn to a circle from the same point that is exterior to
the circle are of equal lengths.
Thus AM = AQ = a

Similarly MB = BP = b and PC = CQ = c

We know AB = a + b = 5

BC = b + c = 12 and AC = a + c = 13

Solving simultaneously we get a=3, b=2 and c=10

We also know that the tangent is perpendicular to the radius

Thus OMBP is a square with side b

Hence the length of the radius of the circle inscribed in the right angled triangle is 2
cm.

www.topperlearning.com 5
CBSE X | MATHEMATICS

Board Paper ˗ 2014

8. Correct answer: A

There are in all 23 = 8 combinations or outcomes for the gender of the 3 children

The eight combinations are as follows

BBB, BBG, BGB, BGG, GBB, GBG, GGB, GGG

7
Thus the probability of having at least one boy in a family is
8

SECTION B

9. Solution:

Given: AB and CD are common tangents to both the circles.

To prove: AB = CD

Proof:

We know that two tangents drawn to a circle for the same exterior point are equal.

Thus we get AE = EC

Similarly ED = EB

AB = AE + EB and CD = ED + EC

Since AE = EC we can write AB = EC + EB

And since ED = EB we get CD = EB + EC

Therefore AB = CD

Hence proved.

www.topperlearning.com 6
CBSE X | MATHEMATICS

Board Paper ˗ 2014

10.

Given:  ABC is an isosceles triangle with a circle inscribed in the triangle.

To prove: BD = DC

Proof:

AF and AE are tangents drawn to the circle from point A.

Since two tangents drawn to a circle from the same exterior point are equal,
AF = AE = a

Similarly BF = BD = b and CD = CE = c

We also know that  ABC is an isosceles triangle

Thus AB = AC,

a+b=a+c

Thus b = c

Therefore, BD = DC

Hence proved.

www.topperlearning.com 7
CBSE X | MATHEMATICS

Board Paper ˗ 2014

11. Solution:

The total number of outcomes when two dice are tossed together is 36.

The sample space is as follows

i. Favourable outcomes = { (2,2) (2,4) (2,6) (4,2) (4,4) (4,6) (6,2) (6,4) (6,6) }

Probability that the number on each dice is even

Number of favourable outcomes 9 1


=  
Total number of outcomes 36 4
ii. Favouable outcomes = { (1,4) (2,3) (3,2) (4,1) }

Probability that the sum of the numbers appearing on the two dice is 5

Number of favourable outcomes 4 1


=  
Total number of outcomes 36 9

12. Solution:

Given total surface area of hemisphere = 462 cm2


2r2  462
r  8.574cm
2
Volume of a hemisphere  r3
3
2 3 2 22
 r    8.5743  1320.54
3 3 7
3
4851 3 7  21 
 r3    
2 2 22  2 

www.topperlearning.com 8
CBSE X | MATHEMATICS

Board Paper ˗ 2014

13. Solution:

Numbers which are divisible by both 2 and 5 are the numbers which are divisible by
10.

Thus we need to find the number of natural numbers between 101 and 999 which
are divisible by 10.

The first number between 101 and 999 which is divisible by 10 is 110

And the last number between 101 and 999 which is divisible by 10 is 990

Using the formula for arithmetic progression where first term ( a ) = 110, last term
( Tn ) = 990 and difference (d) =10

Tn  a  (n  1)d
990  110  (n  1)10
880  (n  1)10
88  n  1
n  89

Hence there are 89 natural numbers between 101 and 999 which are divisible by
both 2 and 5.

14. Solution:

Given: Quadratic equation 9x2  3kx  k  0 has equal roots

Let β be the equal roots of the equation

3k k
Thus 2β =  (Sum of the roots is equal to –b/a)
9 3
k
We get  
6

k
And we also know that  2 = (Product of the roots is equal to c/a)
9

k2 k

36 9
k 1
For k  0, 
36 9

Thus k = 4

www.topperlearning.com 9
CBSE X | MATHEMATICS

Board Paper ˗ 2014

SECTION – C

15. Solution:

Let P and Q be the two positions of the plane and A be the point of observation. Let
ABC be the horizontal line through A.

It is given that angles of elevation of the plane in two positions P and Q from a point
A are 60° and 30° respectively.

∴ ∠PAB = 60°, ∠QAB = 30°. It is also given that PB = 3000√3 m meters

In ΔABP, we have

Tan 60 = BP/AB

Root 3 = 3000√3/ AB

AB = 3000 m

In ΔACQ, we have

tan30 = CQ/AC

1/√3 = 3000√3/ AC

AC = 9000 m

∴ Distance = BC = AC – AB = 9000m – 3000m = 6000m

Thus, the plane travels 6km in 30 seconds

Hence speed of plane = 6000/30 = 200 m/sec = 720km/h

www.topperlearning.com 10
CBSE X | MATHEMATICS

Board Paper ˗ 2014

16. Solution:

Diameter of sphere curved out = side of cube = 7cm ∴ r = 3.5 cm

Volume of cube = a3

= 73

= 343 cm3

Volume of sphere curved out = 4/3  r3

= 4/3 × 22/7 × 7/2 × 7/2 × 7/2

= 179.66 cm3

Volume of of wood left = 343 - 179.66 = 163.34 cm3

17. Solution:

Speed = 4km/h = 200/3 m/min

Volume of water irrigate in 10 min = 10 × 6 × 1.5 × 200/3 = 6000m3

Volume of water irrigated = base area (of irrigated land) x height = base area x 8cm
= base area x 0.08m

6000 = base area x 0.08

Base area = 6000/0.08 =75000 m2 = 7.5 hectare

www.topperlearning.com 11
CBSE X | MATHEMATICS

Board Paper ˗ 2014

18.

Solution:

Area of trapezium = 24.5 cm2

½ [AD + BC] × AB = 24.5 cm2

½ [10+4] × AB = 24.5

AB = 3.5 cm

r = 3.5 cm

Area of quadrant = ¼ × pi × r2 = 0.25 × 22/7 × 3.5 × 3.5 = 9.625 cm2

The area of shaded region = 24.5 - 9.625 = 14.875 cm2

www.topperlearning.com 12
CBSE X | MATHEMATICS

Board Paper ˗ 2014

19. Solution:

Point p lies on x axis so it’s ordinate is 0 (Using section formula)

Let the ratio be k: 1 Let the coordinate of the point be P(x , 0)

As given A(3,-3) and B(-2,7)

Py = (my2 + ny1) /(m + n)

0 = (k × 7 +1 × -3) / (k+1)

0(k + 1) = 7k -3

0 = 7k - 3

3 = 7k

k = 3 /7

k:1=3:7

x = (mx2 + nx1) /(m + n) = [(3/7 × -2 )+(1 × 3)]/(3/7 + 1) = 1.5

www.topperlearning.com 13
CBSE X | MATHEMATICS

Board Paper ˗ 2014

20. Solution:

The area of shaded region = Area of ring – Area of ABCD

= [(R2 – r2) ] - [ (R2 – r2)]×(θ/360)

= [ (R2 – r2) ] [1 –(θ/360)]

= [22/7 (422 – 212) ] [1 –(60/360)] = 3465 cm2

21. Solution:
(16/x)-1 = 15/(x+1)

(16 - x )/x = 15/(x+1)

15 x = 16x + 16 – x2 – x

16 = x2

x=4

22. Solution:
The sum of 2nd and the 7th terms of an AP is 30

a + d + a + 6 d = 30

2a + 7d = 30

15th term is 1 less than twice the 8th term

a + 14d = 2(a + 7d) - 1

a + 14d = 2a + 14d - 1

a=1

Now, 2 × 1 + 7d = 30
d=4

AP : 1,5,9 ………

www.topperlearning.com 14
CBSE X | MATHEMATICS

Board Paper ˗ 2014

23. Solution:

Find midpoint of AB draw the circle

24. Solution:

AC2 = (5-2)2 + (6 +1)2 = 9 + 49 =58 sq. unit

BD2 = (5-2)2 + (-1 - 6)2 =9 + 49 =58 sq. unit

Diagonals of parallelogram are equal so rectangle

SECTION D

25. Given : A circle C (0, r) and a tangent l at point A.

To prove : OA ⊥ l

www.topperlearning.com 15
CBSE X | MATHEMATICS

Board Paper ˗ 2014

Construction: Take a point B, other than A, on the tangent l. Join OB. Suppose OB
meets the circle in C.

Proof: We know that, among all line segment joining the point O to a point on l, the
perpendicular is shortest to l.

OA = OC (Radius of the same circle)

Now, OB = OC + BC.

∴ OB > OC

⇒ OB > OA

⇒ OA < OB

26. Solution:

Volume of 150 spherical marbles, each of diameters 1.4 cm = volume of cylindrical


vessel of diameter 7 cm

150 × 4/3 ×  × 1.4/2 ×1.4/2 × 1.4/2 =  × 7/2 × 7/2 × h

h = 5.6 cm

27. Solution:

Volume of container = 1/3  × h (R2 + r2 + Rr)

= 1/3 × 22/7 × 24[20 × 20 + 8 × 8 + 20 × 8 ]

= 15689.14 cm3

= 15.69 litre
The cost of milk which can completely fill the container at the rate of Rs.21 per liter
= Rs. (21 × 15.69) = Rs. 329.49

www.topperlearning.com 16
CBSE X | MATHEMATICS

Board Paper ˗ 2014

28. Solution:

Let AB is the tower of height h meter and AC is flagstaff of height x meter.

APB = 45˚ and BPC = 60˚

Tan 60 = (x + h)/120

√3 = (x + h)/120

(x + h) = 120√3

x = 120√3 –h

Tan 45˚ = h/120

1 = h/120

h = 120

Therefore height of the flagstaff = 120√3 – 120

= 120(√3 - 1) m

= 87.6 m

www.topperlearning.com 17
CBSE X | MATHEMATICS

Board Paper ˗ 2014

29. Solution:

Let speed of stream = x km/h

Speed f boat in steel water = 18 km/h

Speed f boat in upstream = (18 – x )km/h

Speed f boat in downstream = (18 + x ) km/h

Distance = 24 km

As per question,

24 km /(18 – x ) = 24 km/(18 + x ) + 1

x2 + 48x – 324 = 0
x = 6 or – 54

Hence, the speed of stream = 6 km/h

30. Solution:

Class 1 plant trees = 2 × class 1 x 2 section = 2 × 1 × 2 = 4 × class

= 4 × 1 = 4 trees

Class 2 plant trees =4 × class = 4 × 2 = 8 trees

a=4

d=8

n = 12
S12 = 12/2[2 × 4 + 11 × 4] = 312 trees

31. Solution:

(x-3) / (x-4) +(x -5) /(x-6) = 10/3

[(x -3 )(x - 6) + (x- 4) (x-5) ] /[(x- 4)x (x- 6) ] = 10/3

2[x2 - 9x + 19]/[ x2 - 10x + 24] = 10/3

2x2 - 23x + 63 = 0
x = 7 and 9/2

www.topperlearning.com 18
CBSE X | MATHEMATICS

Board Paper ˗ 2014

32. Solution:
(i) face card are removed from a pack of 52 playing card = 6

Total favorable outcomes = 52 – 6 = 46

Number of all possible outcomes = 26 - 6= 20

P[E] = 20/46 = 0.43

(ii) Number of all possible outcomes a queen = 2

P[E] = 2/46= 1/23

(iii) Number of all possible outcomes an ace = 2

P[E] = 2/46= 1/23

(iv) Number of all possible outcomes = 6

P[E] = 6/46= 3/23

33. Solution:
Let co – ordinate of D (x, y) and D is midpoint of BC

x = (3 + 5)/2 = 4; y = (2 - 2)/2 = 0

Now Area of triangle ABD = ½ {4(-2 – 0) + 3[(0 – (-6)] + 4 [(-6) – (-2)]}

= 0.5 × [ -8 + 18 - 16] = 3 sq unit

and Area of triangle ACD = ½ [5(-6 - 0) + 4(0 - 2) + 4(2 + 6)] = 3 sq unit

Hence, the median AD divides triangle ABC into two triangle of equal area.

www.topperlearning.com 19
CBSE X | MATHEMATICS

Board Paper ˗ 2014

34. Solution:

Let ABCD be a quadrilateral circumscribing a circle centered at O such that it


touches the circle at point P, Q, R, S. Let us

Join the vertices of the quadrilateral ABCD to the center of the circle.

Consider ΔOAP and ΔOAS,

AP = AS (Tangents from the same point)

OP = OS (Radii of the same circle)

OA = OA (Common side)

ΔOAP ≅ ΔOAS (SSS congruence criterion)

Therefore, A ↔ A, P ↔ S, O ↔ O

And thus, ∠POA = ∠AOS

∠1 = ∠8

Similarly,

∠2 = ∠3

∠4 = ∠5

∠6 = ∠7

∠1 + ∠2 + ∠3 + ∠4 + ∠5 + ∠6 + ∠7 + ∠8 = 360º

(∠1 + ∠8) + (∠2 + ∠3) + (∠4 + ∠5) + (∠6 + ∠7) = 360º


2∠1 + 2∠2 + 2∠5 + 2∠6 = 360º

2(∠1 + ∠2) + 2(∠5 + ∠6) = 360º

(∠1 + ∠2) + (∠5 + ∠6) = 180º

www.topperlearning.com 20
CBSE X | MATHEMATICS

Board Paper ˗ 2014

∠AOB + ∠COD = 180º

Similarly, we can prove that ∠BOC + ∠DOA = 180º

Hence, opposite sides of a quadrilateral circumscribing a circle subtend


supplementary angles at the centre of the circle.

www.topperlearning.com 21
CBSE X | Mathematics

Board Paper – 2015 Solution All India Set – 1

CBSE
Class X Mathematics
Board Paper – 2015 Solution
All India Set – 1

Time : 3 hours Total Marks : 90

SECTION A

1.
Given quadratic equation is,
px 2  2 5px  15  0
Here,a  p, b  2 5p, c  15
For real equal roots, discriminant  0
 b2  4ac  0

 
2
 2 5p  4p 15  0
20p2  60p  0
20p  p  3  0
 p  3 or p  0
But, p  0 is not possible.
p  3

2.
Let AB be the tower and BC be its shadow.
AB  20, BC  20 3
In ABC,
AB
tan  
BC
20
tan  
20 3
1
tan  
3
1
but, tan 30 
3
  30
 The Sun is at an altitude of 30 .

www.topperlearning.com 1
CBSE X | Mathematics

Board Paper – 2015 Solution All India Set – 1

3.
Two dice are tossed
S = {(1,1),(1,2),(1,3),(1,4),(1,5),(1,6),
(2,1),(2,2),(2,3),(2,4),(2,5),(2,6),
(3,1),(3,2),(3,3),(3,4),(3,5),(3,6),
(4,1),(4,2),(4,3),(4,4),(4,5),(4,6),
(5,1),(5,2),(5,3),(5,4),(5,5),(5,6),
(6,1),(6,2),(6,3),(6,4),(6,5),(6,6)}
Total number of outcomes when two dice are tossed = 6 x 6= 36
Favourable events of getting product as 6 are:
(1  6  6), (6  1  6),(2  3  6),(3  2  6)
i.e.(1,6), (6,1), (2,3), (3,2)
Favourable events of getting product as 6 = 4
4 1
P(getting product as 6) = 
36 9

4.

mOPT  90  radius is perpendicular to the tangent 


So, OPQ = OPT  QPT
= 90  60
= 30
mPOQ = 2QPT  2  60  120
reflex mPOQ = 360  120  240
1
mPRQ = reflex POQ
2
1
= 240
2
 mPRQ  120

www.topperlearning.com 2
CBSE X | Mathematics

Board Paper – 2015 Solution All India Set – 1

SECTION B

5.

Given that mPRQ = 120


We know that the line joining the centre and
the external point is the angle bisector between
the tangents.
120
Thus, mPRO = mQRO =  60
2
Also we know that lengths of tangents from an external point
are equal.
Thus, PR = RQ.
Join OP and OQ.
Since OP and OQ are the radii from the centre O,
OP  PR and OQ  RQ.
Thus, OPR and OQR are right angled congruent triangles.
Hence, POR=90  PRO=90  60  30
QOR=90  QRO = 90  60  30
1
sin QRO = sin30 
2
PR 1
sin30  
OR 2
 OR  2PR
 OR  PR  PR
 OR  PR  QR

www.topperlearning.com 3
CBSE X | Mathematics

Board Paper – 2015 Solution All India Set – 1

6.

Let the given circle touch the sides AB and AC of the triangle at points F and E
respectively and let the length of line segment AF be x.
Now, it can be observed that:
BF = BD = 6 cm (tangents from point B)
CE = CD = 9 cm (tangents from point C)
AE = AF = x (tangents from point A)

AB = AF + FB = x + 6
BC = BD + DC = 6 + 9 = 15
CA = CE + EA = 9 + x
2s = AB + BC + CA = x + 6 + 15 + 9 + x = 30 + 2x
s = 15 + x
s – a = 15 + x – 15 = x
s – b = 15 + x – (x + 9) = 6
s – c = 15 + x – (6 + x) = 9
Area of ABC = s  s  a  s  b  s  c 

54  15  x  x 6  9

54  3 6 15x  x 2 

18  6 15x  x 2 
324  6 15x  x  2

54  15x  x2
x2  15x  54  0
x2  18x  3x  54  0
x(x  18)  3(x  18)
 x  18  x  3  0
x  18 and x  3
As distance cannot be negative, x = 3
AC = 3 + 9 = 12
AB = AF + FB = 6 + x = 6 + 3 = 9

www.topperlearning.com 4
CBSE X | Mathematics

Board Paper – 2015 Solution All India Set – 1

7.
4x 2  4bx  (a 2  b2 )  0
 a2  b2 
 x 2  bx   0
 4 
b a2  b2
 x  2  x 
2

2 4
2 2
 b  b a b  b
2 2
 x  2  x    
2
 
2 2 4 2
2
 b  a2
x   
 2 4
b a
x 
2 2
b a
x 
2 2
b  a b  a
x ,
2 2
ab ab
Hence, the roots are    and  .
 2   2 

8.
S5  S7  167 and S10  235
n
Now, Sn 
2
2a   n  1 d
S5  S7  167
5 7
 2a  4d  2a  6d  167
2 2
 5a  10d  7a  21d  167
 12a  31d  167 ....(1)
Also, S10  235
10
 2a  9d  235
2
 10a  45d  235
 2a  9d  47 ....(2)

www.topperlearning.com 5
CBSE X | Mathematics

Board Paper – 2015 Solution All India Set – 1

Multiplying equation (2) by 6, we get


12a  54d  282 .....(3)
Subtracting (1) from (3), we get
12a  54d  282
   12a  31d  167
  
23d  115
d  5
Substituting value of d in (2), we have
2a  9(5)  47
 2a  45  47
 2a  2
a 1
Thus, the given A.P. is 1, 6, 11, 16,..........

9.
ABC is right angled at B.
 AC2  AB2  BC2 ....(1)
Also, A   4,7  , B   p,3 and C  7,3

Now, AC2  7  4   3  7   3   4   9  16  25


2 2 2 2

AB2   p  4   3  7   p2  8p  16   4 
2 2 2

 p2  8p  16  16
 p2  8p  32
BC2  7  p   3  3  49  14p  p2  0
2 2

 p2  14p  49
From (1), we have
  
25  p2  8p  32  p2  14p  49 
 25  2p2  22p  81
 2p2  22p  56  0
 p2  11p  28  0
 p2  7p  4p  28  0
 p p  7  4p  7  0
  p  7  p  4   0
 p  7 and p  4

www.topperlearning.com 6
CBSE X | Mathematics

Board Paper – 2015 Solution All India Set – 1

10.
Given, the point s A(x,y), B( 5,7) and C( 4,5) are collinear.
So, the area formed by these vertices is 0.
1
  x 7  5   55  y    4  y  7    0
2
1
 2x  25  5y  4y  28  0
2
1
 2x  y  3  0
2
 2x  y  3  0
 y  2x  3

SECTION C

11.
Here it is given that,
T14 = 2(T8)
⇒ a + (14 – 1)d = 2[a + (8 – 1)d]
⇒ a + 13d = 2[a + 7d]
⇒ a + 13d = 2a + 14d
⇒ 13d – 14d = 2a – a
⇒ – d = a …. (1)

Now, it is given that its 6th term is –8.


T6 = –8
⇒ a + (6 – 1)d = – 8
⇒ a + 5d = –8
⇒ –d + 5d = –8 [∵ Using (1)]
⇒ 4d = –8

⇒ d = –2
Subs. this in eq. (1), we get a = 2
Now, the sum of 20 terms,
n
Sn  2a  (n  1)d 
2
20
S20  2a  (20  1)d 
2
= 102(2)  19( 2)
= 10[4  38]
=  340

www.topperlearning.com 7
CBSE X | Mathematics

Board Paper – 2015 Solution All India Set – 1

12.
For the given equation, 3x 2  2 2x  2 3  0
Comparing thisequation withax 2  bx  c  0, we obtain
a  3,b  2 2,c  2 3
Now, D  b2  4ac
 ( 2 2)2  4( 3)( 2 3)
 8  24  32  4 2
Using quadraticformula, we obtain
b  b2  4ac
x
2a
( 2 2)  4 2
x
2 3
2 24 2 2 2 4 2
x or x 
2 3 2 3
2 2 2 2 2 2
x or x =
3 3
3 2  2
x or x 
3 3
 2
 x  3 2 or x 
3
 2
 x  6 or x 
3

13.
Let BC be the height at which the aeroplane is observed from point A.
Then, BC = 1500 3
In 15 seconds, the aeroplane moves from point A to D.
A and D are the points where the angles of elevation 60 and 30
are formed respectively.
Let BA = x metres and AD  y metres
BC = x + y

www.topperlearning.com 8
CBSE X | Mathematics

Board Paper – 2015 Solution All India Set – 1

In CBA,
BC
tan60°=
BA
1500 3
3
x
 x  1500 m ....(1)

In CBD,
BC
tan30°=
BD
1 1500 3

3 xy
 x  y  1500(3)  4500
1500  y  4500
 y  3000 m ....(2)
We know that, the aeroplane moves from point A to D in 15 seconds and the
distance covered is 3000 metres. (by 2)
dis tance
Speed 
time
3000
Speed 
15
Speed  200m / s

18
Converting it to km/hr = 200   720km / hr
5

www.topperlearning.com 9
CBSE X | Mathematics

Board Paper – 2015 Solution All India Set – 1

14.
Here, P(x,y) divides line segment AB, such that
3
AP  AB
7
AP 3

AB 7
AB 7

AP 3
AB 7
1  1
AP 3
AB  AP 7 3

AP 3
BP 4

AP 3
AP 3

BP 4
P divides AB in the ratio 3: 4
3  2  4  2 3   4   4  2 
x ; y
3 4 3 4
68 12  8
x ; y
7 7
2 20
x ; y
7 7
 2 20 
 The coordinates of P are  , 
 7 7 

15.
Here the jar contains red, blue and orange balls.
Let the number of red balls be x.
Let the number of blue balls be y.
Number of orange balls = 10
 Total number of balls = x + y + 10
Now, let P be the probability of drawing a ball from the jar
x
P(a red ball) =
x + y + 10
1 x
 
4 x + y + 10
 4x  x + y + 10
 3x  y  10 ----(i)

www.topperlearning.com 10
CBSE X | Mathematics

Board Paper – 2015 Solution All India Set – 1

Next,
y
P(a blue ball) =
x + y + 10
1 y
 
3 x + y + 10
 3y  x + y + 10
 2y  x  10 -----(ii)
Multiplying eq. (i) by 2 and adding to eq. (ii), we get
6x  2y  20
x + 2y  10
5x = 30
 x=6
Subs. x = 6 in eq. (i), we get y = 8
 Total number of balls = x + y + 10 = 6 + 8 + 10 = 24
Hence, total number of balls in the jar is 24.

16.
Radius of the circle =14 cm

Central Angle, 𝜽 = 60,


Area of the minor segment
 1
  r2  r2 sin 
360 2
60 1
   142   142  sin60
360 2
1 22 1 3
   14  14   14  14 
6 7 2 2
22  14
  49 3
3
22  14 147 3
 
3 3
308  147 3 2
 cm
3
308  147 3 2
Area of the minor segment  cm
3

www.topperlearning.com 11
CBSE X | Mathematics

Board Paper – 2015 Solution All India Set – 1

308  147 3 2
Area of major segment  r2  cm
3
22 308  147 3 2
  14  14  cm
7 3
308  147 3 2
 616  cm
3

17.
Diameter of the tent = 4.2 m
Radius of the tent, r = 2.1 m
Height of the cylindrical part of tent, hcylinder = 4 m
Height of the conical part, hcone = 2.8 m

 hcone2  r2

 2.82  2.12
 2.82  2.12
 12.25  3.5 m
Curved surface area of the cylinder = 2𝜋r hcylinder
22
=2× × 2.1 × 4
7
= 22 × 0.3 × 8 = 52.8 m2
22
Curved surface area of the conical tent = 𝜋rl = × 2.1 × 3.5 = 23.1 m2
7
Total area of cloth required for building one tent
= Curved surface area of the cylinder + Curved surface area of the conical tent
= 52.8 + 23.1
= 75.9 m2
Cost of building one tent = 75.9 × 100 = Rs 7590
Total cost of 100 tents = 7590 × 100 = Rs 7,59,000
759000
Cost to be borne by the associations = = Rs 3,79,500
2
It shows the helping nature, unity and cooperativeness of the associations.

www.topperlearning.com 12
CBSE X | Mathematics

Board Paper – 2015 Solution All India Set – 1

18.
Internal diameter of the bowl = 36 cm
Internal radius of the bowl, r = 18 cm
2 2
Volume of the liquid, V = 𝜋r3 = × 𝜋 × 183
3 3
Let the height of the small bottle be ‘h’.
Diameter of a small cylindrical bottle = 6 cm
Radius of a small bottle, R = 3 cm
Volume of a single bottle = 𝜋R2h = 𝜋 × 32 × h
No. of small bottles, n = 72
10 2
Volume wasted in the transfer = × × 𝜋 × 183
100 3
Volume of liquid to be transferred in the bottles
2 10 2
= × 𝜋 × 183 − × × 𝜋 × 183
3 100 3
2  10 
= × 𝜋 × 183  1 
3  100 
2 90
= × 𝜋 × 183 ×
3 100
Volume of the liquid to be transferred
Number of small cylindrical bottles =
Volume of a single bottle
2 90
  183 
 72  3 2
100
 3  h
2 9
 183 
 72  3 2 10
3 h
2 9
 18  18  18 
h 3 10
32  72
 h  5.4 cm

Height of the small cylindrical bottle = 5.4 cm

www.topperlearning.com 13
CBSE X | Mathematics

Board Paper – 2015 Solution All India Set – 1

19. Side of the cubical block, a = 10 cm


Longest diagonal of the cubical block = a√3 = 10√3 cm
Since the cube is surmounted by a hemisphere, therefore the side of the cube should
be equal to the diameter of the hemisphere.
Diameter of the sphere = 10 cm
Radius of the sphere, r = 5 cm
Total surface area of the solid = Total surface area of the cube – Inner cross-section
area of the hemisphere + Curved surface area of the hemisphere
= 6a2 – 𝜋r2 + 2𝜋r2
= 6a2 + 𝜋r2
= 6 × (10)2 + 3.14 52
 600  78.5  678.5 cm2
Total surface area of the solid = 678.5 cm2
Cost of painting per sq. cm = Rs. 5
Cost of painting the total surface area of the solid = 678.5 × 5 = Rs. 3392.50

20. No. of cones = 504


Diameter of a cone = 3.5 cm
Radius of the cone, r = 1.75 cm
Height of the cone, h = 3 cm
Volume of a cone
1
 r2h
3
2
1  3.5 
     3
3  2 
1 3.5 3.5
     3cm3
3 2 2
Volume of 504 cones
1 3.5 3.5
 504      3cm3
3 2 2
Let the radius of the new sphere be ‘R’.
4
Volume of the sphere  R3
3
Volume of 504 cones = Volume of the sphere
1 3.5 3.5 4
504      3  R3
3 2 2 3
504  1   3.5  3.5  3  3
  R3
3 2 2 4  
504  3  49
 R3 
64

www.topperlearning.com 14
CBSE X | Mathematics

Board Paper – 2015 Solution All India Set – 1

7  8  9  3  72
3
R 
64
8  27  73
 R3 
64
2 3 7
R 
4
21
R   10.5 cm
2
Radius of the new sphere = 10.5 cm
Surface area of the new sphere = 4R 2
22 21 21
 4  
7 2 2
 2772 cm2

SECTION D

21. Let l be the length of the longer side and b be the length of the shorter side.
Given that the length of the diagonal of the rectangular field is 16 metres more than
the shorter side.
Thus, diagonal = 16 + b
Since longer side is 14 metres more than shorter side, we have,
 l = 14 + b
Diagonal is the hypotenuse of the triangle.
Consider the following figure of the rectangular field.

www.topperlearning.com 15
CBSE X | Mathematics

Board Paper – 2015 Solution All India Set – 1

By applying Pythagoras Theorem in ABD, we have,


Diagonal2  Length2  Breadth2
 (16  b)2  b2
 256  b2  32b  196  b2  28b  b2
 256  32b  196  28b  b2
 60  32b  28b  b2
 b2  4b  60  0
 b2  10b  6b  60  0
 b(b  10)  6(b  10)  0
 (b  6)(b  10)  0
 (b  6)  0 or (b  10)  0
 b  6 or b=10
As breadth cannot be negative, breadth = 10 m
Thus, length of the rectangular field = 14+10 = 24m

22. Consider the given A.P. 8, 10, 12, …


Here the initial term is 8 and the common difference is 10 - 8 = 2 and 12 - 10 = 2
General term of an A.P. is tn and formula to find out tn is
t n  a  (n  1)d
 t 60  8  (60  1)  2
 t 60  8  59  2
 t 60  8  118
 t 60  126
We need to find the sum of the last 10 terms.
Thus,
Sum of last 10 terms = Sum of first 60 terms - Sum of first 50 terms
n
Sn  2a  (n  1)d 
2
60
 S60  2  8  (60  1)  2
2
 S60  3016  59  2
 S60  30[134]
 S60  4020

www.topperlearning.com 16
CBSE X | Mathematics

Board Paper – 2015 Solution All India Set – 1

Similarly,
50
 S50  2  8  (50  1)  2
2
 S50  2516  49  2
 S50  25114
 S50  2850
Thus the sum of last 10 terms = S60  S50  4020  2850  1170
Therefore,
Sum of last 10 terms = Sum of first 60 terms - Sum of first 50 terms

23.

Let x be the first speed of the train


Distance
We know that  time
Speed
Thus, we have,
54 63
  3 hours
x x 6
54(x  6)63x
 3
x(x  6)
 54(x+6)+63x=3x(x+6)
 54x+324+63x=3x 2  18x
 117x  324  3x 2  18x
 3x 2  117x  324  18x  0
 3x 2  99x  324  0
 x 2  33x  108  0
 x 2  36x  3x  108  0
 x(x  36)  3(x  36)  0
 (x  3)(x  36)  0
 (x  3)  0 or (x-36)=0
 x=-3 or x =36
Speed cannot be negative and hence intial speed of the train is 36km/hour

www.topperlearning.com 17
CBSE X | Mathematics

Board Paper – 2015 Solution All India Set – 1

24. Consider the following diagram.

Let P be an external point and PA and PB be tangents to the circle.


We need to prove that PA = PB
Now consider the triangles OAP and OBP
mA = mB = 90
OP = OP [common]
OA = OB = radii of the circle
Thus, by Right Angle-Hypotenuse-Side criterion of congruence we have,
OAP  OBP
The corresponding parts of the congruent triangles are congruent.
Thus,
PA = PB

25.
In the figure, C is the midpoint of the minor arc PQ, O is the centre of the circle and
AB is tangent to the circle through point C.
We have to show the tangent drawn at the midpoint of the arc PQ of a circle is parallel
to the chord joining the end points of the arc PQ.
We will show PQ AB.
It is given that C is the midpoint point of the arc PQ.
So, arc PC = arc CQ.
PC = CQ

www.topperlearning.com 18
CBSE X | Mathematics

Board Paper – 2015 Solution All India Set – 1

This shows that PQC is an isosceles triangle.


Thus, the perpendicular bisector of the side PQ of PQC passes through vertex C.
The perpendicular bisector of a chord passes through the centre of the circle.
So the perpendicular bisector of PQ passes through the centre O of the circle.
Thus perpendicular bisector of PQ passes through the points O and C.
PQ  OC
AB is the tangent to the circle through the point C on the circle.
AB  OC
The chord PQ and the tangent PQ of the circle are perpendicular to the same line OC.
PQ  AB.

26.

1)Construct the ABC as per given measurements.


2)In the half plane of AB which does not contain C, draw AX
such that BAX is an acute angle.
3) With some approprriate radius and centre A, Draw an arc to
intersect AX at B1 . Similarly, with center B1 and the same radius,
draw an arc to intersect BX at B2 such that B1B2 = B3B4 = B4B5 = B5B6
 B6B7 =B7B8
4) Draw B6B.
5) Through B8 draw a ray parallel to B6B. to intersect AY at B'.
6) Through B' draw a ray parallel to BC to intersect AZ at C'.
Thus, AB'C' is the required triangle.

www.topperlearning.com 19
CBSE X | Mathematics

Board Paper – 2015 Solution All India Set – 1

27.

Let PB be the surface of the lake and A be the point of observation such that
AP = 20 metres. Let C be the position of the cloud and C’ be its reflection in the lake.
Then CB = C’B. Let AM be perpendicular from A on CB.
Then mCAM  30 and mC'AM  60
Let CM = h. Then CB = h + 20 and C’B = h + 20. (CB=CB’ since refection about PB)

www.topperlearning.com 20
CBSE X | Mathematics

Board Paper – 2015 Solution All India Set – 1

In CMA we have,
CM
tan30 
AM
1 h
 
3 AM
 AM  3h...................(i)
In AMC' we have,
C'M
tan60 
AM
C'B  BM
 3
AM
h  20  20
 3
AM
h  20  20
 AM  ...................(ii)
3
From equation (i) and (ii), we get
h  20  20
3h 
3
 3h  h  40
 2h  40
 h  20 m
 AM  20 3
Now, to find AC using pythagoras theorem
AC2  AM2  MC2
 1600
AC  40
Hence, the height of the cloud from the
surface of the lake is 40 metres.

28.
Let S be the sample space of drawing a card from a well-shuffled deck.
n  S   52 C1  52

(i)There are 13 spade cards and 4 ace's in a deck


As ace of spade is included in 13 spade cards,
so there are 13 spade cards and 3 ace's

a card of spade or an ace can be drawn in 13


C1  3 C1  13  3  16
16 4
Probability of drawing a card of spade or an ace = 
52 13
www.topperlearning.com 21
CBSE X | Mathematics

Board Paper – 2015 Solution All India Set – 1

(ii)There are 2 black King cards in a deck


a card of black King can be drawn in 2 C1  2
2 1
Probability of drawing a black king = 
52 26

(iii)There are 4 Jack and 4 King cards in a deck.


So there are 52  8 = 44 cards which are neither Jacks nor Kings.
a card which is neither a Jack nor a King can be drawn in 44
C1  44
44 11
Probability of drawing a card which is neither a Jack nor a King = 
52 13

(iv)There are 4 King and 4 Queen cards in a deck.


So there are 4  4 = 8 cards which are either King or Queen.
a card which is either a King or a Queen can be drawn in 8 C1  8
8 2
Probability of drawing a card which is either a King or a Queen = 
52 13

29.
Take  x1 ,y 1   1, 1  ,  4,2k  and  k, 5
It is given that the area of the triangle is 24 sq. unit

Area of the triangle having vertices  x 1 ,y 1  ,  x 2 ,y 2  and  x3 ,y 3 


is given by
1
=  x1  y 2  y 3   x 2  y 3  y 1   x3  y 1  y 2  
2
1
24  1  2k   5    4    5   1     k    1   2k  
2

48   2k  5  16  k  2k 2  
2k 2  3k  27  0
 2k  9  k  3  0
9
 k   or k  3
2

9
 The values of k are  and3.
2

www.topperlearning.com 22
CBSE X | Mathematics

Board Paper – 2015 Solution All India Set – 1

30.
PQRS is a square.
So each side is equal and angle between the adjacent sides is a right angle.
Also the diagonals perpendicularly bisect each other.
In PQR using pythagoras theorem,
PR 2 =PQ2 +QR 2
2 2
PR 2   42   42
PR= 2  42
1 42
OR  PR=  OQ
2 2
From the figure we can see that the radius of flower bed ORQ is OR.
1
Area of sector ORQ  r2
4
2
1  42 
 
4  2 
1
Area of the ROQ =  RO  OQ
2
1 42 42
  
2 2 2
2
 42 
 
 2 
Area of the flower bed ORQ
=Area of sector ORQ  Area of the ROQ
2 2
1  42   42 
=  
4  2   2 
2
 42    
     1
 2  2 
  441  0.57
 251.37cm2
Area of the flower bed ORQ = Area of the flower bed OPS
= 251.37cm2
Total area of the two flower beds
= Area of the flower bed ORQ + Area of the flower bed OPS
=251.37  251.37
 502.74 cm2

www.topperlearning.com 23
CBSE X | Mathematics

Board Paper – 2015 Solution All India Set – 1

31.
Height of the cylinder (h) = 10 cm
Radius of the base of the cylinder = 4.2 cm
Volume of original cylinder = r2h
22 2
   4.2  10
7
 554.4cm3
2 3
Volume of hemisphere = r
3
2 22 3
    4.2
3 7
 155.232 cm3

Volume of the remaining cylinder after scooping out hemisphere from each end
 Volume of original cylinder  2  Volume of hemisphere
 554.4  2  155.232
 243.936 cm3

The remaining cylinder is melted and converted to


a new cylindrical wire of 1.4 cm thickness.
So they have same volume and radius of new cylindrical wire is 0.7 cm.
Volume of the remaining cylinder = Volume of the new cylindrical wire
243.936  r2h
22 2
243.936   0.7  h
7
h  158.4 cm

 The length of the new cylindrical wire of 1.4 cm thickness is 158.4 cm.

www.topperlearning.com 24
CBSE X | Mathematics

Board Paper – 2015 Solution All India Set – 2

CBSE
Class X Mathematics
Board Paper – 2015 Solution
All India Set – 2

Time: 3 hours Total Marks: 90

1.

mOPT  90  radius is perpendicular to the tangent 


So, OPQ = OPT  QPT
= 90  60
= 30
mPOQ = 2QPT  2  60  120
reflex mPOQ = 360  120  240
1
PRQ = reflex POQ
2
1
=  240
2
 120
 mPRQ  120

www.topperlearning.com 1
CBSE X | Mathematics

Board Paper – 2015 Solution All India Set – 2

2.
The given quadratic equation is,
px 2  2 5px  15  0
Here,a  p, b  2 5p, c  15
For real equal roots, discriminant  0
 b2  4ac  0

 
2
 2 5p  4p 15  0

20p2  60p  0
20p  p  3  0
 p  3 or p  0
But, p  0 is not possible.
p  3

3.
Let AB be the tower and BC be its shadow.
AB  20, BC  20 3
In ABC,
AB
tan  
BC
20
tan  
20 3
1
tan  
3
1
but, tan 30 
3
  30
 The Sun is at an altitude of 30 .

www.topperlearning.com 2
CBSE X | Mathematics

Board Paper – 2015 Solution All India Set – 2

4.
Two dice are tossed
S = [(1,1),(1,2),(1,3),(1,4),(1,5),(1,6),
(2,1),(2,2),(2,3),(2,4),(2,5),(2,6),
(3,1),(3,2),(3,3),(3,4),(3,5),(3,6),
(4,1),(4,2),(4,3),(4,4),(4,5),(4,6),
(5,1),(5,2),(5,3),(5,4),(5,5),(5,6),
(6,1),(6,2),(6,3),(6,4),(6,5),(6,6)]
Total number of outcomes when two dice are tossed = 6 x 6= 36
Favourable events of getting the product as 6 are:
(1  6  6), (6  1  6),(2  3  6),(3  2  6)
i.e.(1,6), (6,1), (2,3), (3,2)
Favourable events of getting product as 6 = 4
4 1
P(getting product as 6) = 
36 9

SECTION B

5.
Given that the point s A(x,y), B( 5,7) and C( 4,5) are collinear.
So, the area formed by these vertices is 0.
1
  x 7  5   55  y    4  y  7    0
2
1
 2x  25  5y  4y  28  0
2
1
 2x  y  3  0
2
 2x  y  3  0
 y  2x  3

www.topperlearning.com 3
CBSE X | Mathematics

Board Paper – 2015 Solution All India Set – 2

6.
S5  S7  167 and S10  235
n
Now, Sn 
2
2a   n  1 d
S5  S7  167
5 7
 2a  4d  2a  6d  167
2 2
 5a  10d  7a  21d  167
 12a  31d  167 ....(1)
Also, S10  235
10
 2a  9d  235
2
 10a  45d  235
 2a  9d  47 ....(2)
Multiplying equation (2) by 6, we get
12a  54d  282 .....(3)
Subtracting (1) from (3), we get
12a  54d  282
   12a  31d  167
  
23d  115
d  5
Substituting value of d in (2), we have
2a  9(5)  47
 2a  45  47
 2a  2
a 1
Thus, the given A.P. is 1, 6, 11, 16,..........

www.topperlearning.com 4
CBSE X | Mathematics

Board Paper – 2015 Solution All India Set – 2

7.

Given that m PRQ = 120


We know that the line joining the centre and
the external point is the angle bisector between
the tangents.
120
Thus, m PRO = m QRO = 60
2
Also we know that lengths of tangents from an external point
are equal.
Thus, PR = RQ.
Join OP and OQ.
Since OP and OQ are the radii from the centre O,
OP PR and OQ RQ.
Thus, OPR and OQR are right angled congruent triangles.
Hence, m POR = 90 m PRO = 90 60 30
m QOR = 90 m QRO = 90 60 30
1
sin QRO = sin30
2
PR
But sin30
OR
PR 1
Thus,
OR 2
OR 2PR
OR PR PR
OR PR QR

www.topperlearning.com 5
CBSE X | Mathematics

Board Paper – 2015 Solution All India Set – 2

8.

Let the given circle touch the sides AB and AC of the triangle at points F and E
respectively and let the length of the line segment AF be x.
Now, it can be observed that:
BF = BD = 6 cm (tangents from point B)
CE = CD = 9 cm (tangents from point C)
AE = AF = x (tangents from point A)

AB = AF + FB = x + 6
BC = BD + DC = 6 + 9 = 15
CA = CE + EA = 9 + x
2s = AB + BC + CA = x + 6 + 15 + 9 + x = 30 + 2x
s = 15 + x
s – a = 15 + x – 15 = x
s – b = 15 + x – (x + 9) = 6
s – c = 15 + x – (6 + x) = 9
Area of ABC = s  s  a  s  b  s  c 

54  15  x  x 6  9

54  3 6 15x  x 2 

18  6 15x  x2 
324  6 15x  x  2

54  15x  x2
x2  15x  54  0
x2  18x  3x  54  0
x(x  18)  3(x  18)
 x  18  x  3  0
x  18 and x  3
As distance cannot be negative, x = 3
AC = 3 + 9 = 12
AB = AF + FB = 6 + x = 6 + 3 = 9

www.topperlearning.com 6
CBSE X | Mathematics

Board Paper – 2015 Solution All India Set – 2

9.
4x 2  4bx  (a 2  b2 )  0
 a2  b2 
 x 2  bx   0
 4 
b a2  b2
 x  2  x 
2

2 4
2 2
 b  b a b  b
2 2
 x  2  x    
2
 
2 2 4 2
2
 b  a2
x   
 2 4
b a
x 
2 2
b a
x 
2 2
b  a b  a
x ,
2 2
ab ab
Hence, the roots are    and  .
 2   2 

www.topperlearning.com 7
CBSE X | Mathematics

Board Paper – 2015 Solution All India Set – 2

10. Given that A(4, 3), B(-1, y) and C(3, 4) are the vertices of the ABC.
∆ABC is a right triangle at A.
Hence by applying the Pythagoras Theorem, we have,
AB2  AC2  BC2 ....(1)
Let us find the distances, AB, BC and CA using the
distance formula.

 1  4    y  3
2 2
AB =

3  1    4  y 
2 2
BC 

3  4    4  3
2 2
CA   2
Squaring both the sides, we have
AB2  25  y 2  9  6y
BC2  4  16  y 2  8y
AC2  2
Therefore, from equation (1), we have,
25  y 2  9  6y  2  4  16  y 2  8y
 36  y 2  6y  20  y 2  8y
 16  6y  8y
 16  8y  6y
 2y  16
16
 y 
2
 y  8

www.topperlearning.com 8
CBSE X | Mathematics

Board Paper – 2015 Solution All India Set – 2

SECTION C

11. Diameter of the tent = 4.2 m


Radius of the tent, r = 2.1 m
Height of the cylindrical part of tent, hcylinder = 4 m
Height of the conical part, hcone = 2.8 m
Slant height of the conical part, 

hcone2 r2

2.82 2.12
2.82 2.12
12.25 3.5 m
Curved surface area of the cylinder = 2𝜋r hcylinder
22
=2× × 2.1 × 4
7
= 22 × 0.3 × 8 = 52.8 m2
22
Curved surface area of the conical tent = 𝜋rl = × 2.1 × 3.5 = 23.1 m2
7
Total area of cloth required for building one tent
= Curved surface area of the cylinder + Curved surface area of the conical tent
= 52.8 + 23.1
= 75.9 m2
Cost of building one tent = 75.9 × 100 = Rs. 7590
Total cost of 100 tents = 7590 × 100 = Rs. 7,59,000
759000
Cost to be borne by the associations = = Rs. 3,79,500
2
It shows the helping nature, unity and cooperativeness of the associations.

www.topperlearning.com 9
CBSE X | Mathematics

Board Paper – 2015 Solution All India Set – 2

12.
Let BC be the height at which the aeroplane is observed from point A.
Then, BC = 1500 3
In 15 seconds, the aeroplane moves from point A to D.
A and D are the points where the angles of elevation 60 and 30
are formed respectively.
Let BA = x metres and AD  y metres
BC = x + y

In CBA,
BC
tan60°=
BA
1500 3
3
x
 x  1500 m ....(1)

In CBD,
BC
tan30°=
BD
1 1500 3

3 xy
 x  y  1500(3)  4500
1500  y  4500
 y  3000 m ....(2)
We know that the aeroplane moves from point A to D in 15 seconds and the distance
covered is 3000 metres. (by 2)

www.topperlearning.com 10
CBSE X | Mathematics

Board Paper – 2015 Solution All India Set – 2

dis tance
Speed 
time
3000
Speed 
15
Speed  200m/s

18
Converting it to km/hr = 200   720 km/hr
5

13. Internal diameter of the bowl = 36 cm


Internal radius of the bowl, r = 18 cm
2 2
Volume of the liquid, V = 𝜋r3 = × 𝜋 × 183
3 3
Let the height of the small bottle be ‘h’.
Diameter of a small cylindrical bottle = 6 cm
Radius of the small bottle, R = 3 cm
Volume of a single bottle = 𝜋R2h = 𝜋 × 32 × h
No. of small bottles, n = 72
10 2
Volume wasted in the transfer = × × 𝜋 × 183
100 3
Volume of liquid to be transferred in the bottles
2 10 2
= × 𝜋 × 183 − × × 𝜋 × 183
3 100 3
2 10
= × 𝜋 × 183 1
3 100
2 90
= × 𝜋 × 183 ×
3 100
Volume of the liquid to be transferred
Number of the small cylindrical bottles =
Volume of a single bottle
2 90
183
72 3 100
32 h
2 9
183
72 3 2 10
3 h
2 9
18 18 18
h 3 2
10
3 72
h 5.4 cm

Height of the small cylindrical bottle = 5.4 cm

www.topperlearning.com 11
CBSE X | Mathematics

Board Paper – 2015 Solution All India Set – 2

14.
Here the jar contains red, blue and orange balls.
Let the number of red balls be x.
Let the number of blue balls be y.
Number of orange balls = 10
 Total number of balls = x + y + 10
Now, let P be the probability of drawing a ball from the jar
x
P(a red ball) =
x + y + 10
1 x
 
4 x + y + 10
 4x  x + y + 10
 3x  y  10 ----(i)
Next,
y
P(a blue ball) =
x + y + 10
1 y
 
3 x + y + 10
 3y  x + y + 10
 2y  x  10 -----(ii)
Multiplying eq. (i) by 2 and adding to eq. (ii), we get
6x  2y  20
x + 2y  10
5x = 30
 x=6
Subs. x = 6 in eq. (i), we get y = 8
 Total number of balls = x + y + 10 = 6 + 8 + 10 = 24
Hence, total number of balls in the jar is 24.

www.topperlearning.com 12
CBSE X | Mathematics

Board Paper – 2015 Solution All India Set – 2

15. Side of the cubical block, a = 10 cm


Longest diagonal of the cubical block = a√3 = 10√3 cm
Since the cube is surmounted by a hemisphere, therefore the side of the cube should
be equal to the diameter of the hemisphere.
Diameter of the sphere = 10 cm
Radius of the sphere, r = 5 cm
Total surface area of the solid = Total surface area of the cube – Inner cross-section
area of the hemisphere + Curved surface area of the hemisphere
= 6a2 – 𝜋r2 + 2𝜋r2
= 6a2 + 𝜋r2
= 6 × (10)2 + 3.14 52
600 78.5 678.5 cm2
Total surface area of the solid = 678.5 cm2
Cost of painting per sq. cm = Rs. 5
Cost of painting the total surface area of the solid = 678.5 × 5 = Rs. 3392.50

16.
Here, P(x,y) divides line segment AB, such that
3
AP  AB
7
AP 3

AB 7
AB 7

AP 3
AB 7
1  1
AP 3
AB  AP 7 3

AP 3
BP 4

AP 3
AP 3

BP 4
P divides AB in the ratio 3: 4
3  2  4  2 3   4   4  2 
x ; y
3 4 3 4
68 12  8
x ; y
7 7
2 20
x ; y
7 7
 2 20 
 The co  ordinates of P are  , 
 7 7 

www.topperlearning.com 13
CBSE X | Mathematics

Board Paper – 2015 Solution All India Set – 2

17. No. of cones = 504


Diameter of a cone = 3.5 cm
Radius of the cone, r = 1.75 cm
Height of the cone, h = 3 cm
Volume of a cone
1 2
rh
3
2
1 3.5
3
3 2
1 3.5 3.5
3 cm3
3 2 2
Volume of 504 cones
1 3.5 3.5
504 3 cm3
3 2 2
Let the radius of the new sphere be ‘R’.
4 3
Volume of the sphere R
3
Volume of 504 cones = Volume of the sphere
1 3.5 3.5 4 3
504 3 R
3 2 2 3
504 1 3.5 3.5 3 3
R3
3 2 2 4
504 3 49
R3
64
7 8 9 3 72
R3
64
8 27 73
R3
64
2 3 7
R
4
21
R 10.5 cm
2
Radius of the new sphere = 10.5 cm

Surface area of the new sphere=4R 2


22 21 21
 4  
7 2 2
 2772 cm2

www.topperlearning.com 14
CBSE X | Mathematics

Board Paper – 2015 Solution All India Set – 2

18. Given that the area of the circle is 1256 cm2.


r2  1256
 3.14  r2  1256
1256
 r2 
3.14
 r  400
2

 r  20 cm
If all the vertices of a rhombus lie on a circle, then
the rhombus is square.
Consider the following figure.

Here A, B, C and D are four points on the circle.


Thus, OA = OB = OC = OD = radius of the circle.
AC and BD are the diameters of the circle.
Consider the  ADC.
By Pythagoras theorem, we have,
AD2  CD2  AC2
 2AD2   2  20  ....[AD  CD, side of the square]
2

 2AD2   40
2

 2AD2  1600
1600
 AD2 
2
 AD  800cm2
2

If AD is the side of the square, then AD2 is the area of the square.
Thus area of the square is 800cm2

www.topperlearning.com 15
CBSE X | Mathematics

Board Paper – 2015 Solution All India Set – 2

19. Consider the given equation:


2x2  6 3  60  0
 x2  3 3  30  0
Let us the quadratic formula to find x.
b  b2  4ac
x=
2a
Here, a = 1, b = 3 3 and c =  30
Thus,

3 3 
2
3 3   4  1   30 
x=
2 1
3 3  27  120
 x=
2
3 3  147
 x=
2

20. Given that 16th term of an A.P. is five times its third term.
We know that
t n  a   n  1 d
Thus,
t 16  a  16  1  d
t 3  a  3  1  d
Since t 16  5t 3 , we have,
a  16  1  d  5 a  3  1  d 
 a  15d  5a  2d 
 a  15d  5a  10d
 5d  4a
 4a  5d  0...(1)
Also given that t 10  41
 t 10  a  10  1  d
 41  a  9d
 a  9d  41...(2)
Multiplying equation (2) by 4, we have,
4a + 36d = 164...(3)

www.topperlearning.com 16
CBSE X | Mathematics

Board Paper – 2015 Solution All India Set – 2

Subtracting equation (1) from equation (3), we have,


36   5  d  164
 41d  164
164
d
41
d4
Substituting d = 4 in equation (1) 4a  5d  0,we have,
4a  5  4  0
 4a  20  0
 4a  20
20
a
4
a 5
We need to find S15
We know that
n
Sn  2a   n  1  d 
2
15
 S15  2  5  15  1   4 
2
 a  5,n  15,d  4 

15
 S15  10  14  4
2
15
 S15   66
2
 S15  495

www.topperlearning.com 17
CBSE X | Mathematics

Board Paper – 2015 Solution All India Set – 2

SECTION D

21. In the figure, C is the midpoint of the minor arc PQ, O is the centre of the circle and
AB is tangent to the circle through point C.
We have to show the tangent drawn at the mid-point of the arc PQ of a circle is
parallel to the chord joining the end points of the arc PQ.
We will show PQ AB.
It is given that C is the midpoint point of the arc PQ.
So, arc PC = arc CQ.
PC = CQ

This shows that PQC is an isosceles triangle.


Thus, the perpendicular bisector of the side PQ of PQC passes through vertex C.
The perpendicular bisector of a chord passes through the centre of the circle.
So the perpendicular bisector of PQ passes through the centre O of the circle.
Thus the perpendicular bisector of PQ passes through the points O and C.
PQ  OC
AB is the tangent to the circle through the point C on the circle.
AB  OC
The chord PQ and the tangent PQ of the circle are perpendicular to the same line OC.
PQ  AB.

www.topperlearning.com 18
CBSE X | Mathematics

Board Paper – 2015 Solution All India Set – 2

22.

Let PB be the surface of the lake and A be the point of observation such that
AP = 20 metres. Let C be the position of the cloud and C’ be its reflection in the lake.
Then CB = C’B. Let AM be perpendicular from A on CB.
Then mCAM  30 and mC'AM  60
Let CM = h. Then CB = h + 20 and C’B = h + 20. (CB=CB’ since refection about PB)
In CMA we have,
CM
tan30 
AM
1 h
 
3 AM
 AM  3h...................(i)

In AMC' we have,
C'M
tan 60 
AM
C'B  BM
 3
AM
h  20  20
 3
AM
h  20  20
 AM  ...................(ii)
3
From equation (i) and (ii), we get
h  20  20
3h 
3
 3h  h  40
 2h  40
 h  20 m
 AM  20 3
Now , to find AC u sin g pythagoras theorem
AC2  AM2  MC2
 1600
AC  40
Hence, the height of the cloud from the
surface of the lake is 40 metres.

www.topperlearning.com 19
CBSE X | Mathematics

Board Paper – 2015 Solution All India Set – 2

23.
Let S be the sample space of drawing a card from a well-shuffled deck.
n  S   52 C1  52

(i)There are 13 spade cards and 4 ace's in a deck


As ace of spade is included in 13 spade cards,
so there are 13 spade cards and 3 ace's

A card of spade or an ace can be drawn in 13


C1  3 C1  13  3  16
16 4
Probability of drawing a card of spade or an ace = 
52 13

(ii)There are 2 black king cards in a deck


A card of black king can be drawn in 2 C1  2
2 1
Probability of drawing a black king = 
52 26

(iii)There are 4 jack and 4 king cards in a deck.


So there are 52  8 = 44 cards which are neither jack nor king.
a card which is neither a jack nor a king can be drawn in 44
C1  44
44 11
Probability of drawing a card which is neither a jack nor a king = 
52 13

(iv)There are 4 king and 4 queen cards in a deck.


So there are 4  4 =8 cards which are either king or queen.
a card which is either a king or a queen can be drawn in 8 C1  8
8 2
Probability of drawing a card which is either a king or a queen = 
52 13

www.topperlearning.com 20
CBSE X | Mathematics

Board Paper – 2015 Solution All India Set – 2

24. PQRS is a square.

So each side is equal and angle between the adjacent sides is a right angle.
Also the diagonals perpendicularly bisect each other.
In PQR using pythagoras theorem,
PR 2 =PQ2 +QR 2
2 2
PR 2   42   42
PR= 2  42
1 42
OR  PR=  OQ
2 2
From the figure we can see that the radius of the flower bed ORQ is OR.
1
Area of sector ORQ  r 2
4
2
1  42 
 
4  2 
1
Area of the ROQ =  RO  OQ
2
1 42 42
  
2 2 2
2
 42 
 
 2 

Area of the flower bed ORQ


= Area of sector ORQ  Area of the ROQ
2 2
1  42   42 
=  
4  2   2 
2
 42    
     1
 2  2 
  441 0.57
 251.37cm2

Area of the flower bed ORQ =Area of the flower bed OPS
=251.37cm2

Total area of the two flower beds


= Area of the flower bed ORQ+Area of the flower bed OPS
=251.37  251.37
 502.74cm2

www.topperlearning.com 21
CBSE X | Mathematics

Board Paper – 2015 Solution All India Set – 2

25. Height of the cylinder (h) = 10 cm


Radius of the base of the cylinder = 4.2 cm
Volume of original cylinder =r2h
22 2
   4.2  10
7
 554.4cm3
2
Volume of hemisphere = r3
3
2 22 3
    4.2
3 7
 155.232cm3

Volume of the remaining cylinder after scooping out the hemisphere from each end
 Volume of original cylinder  2  Volume of hemisphere
 554.4  2  155.232
 243.936cm3

The remaining cylinder is melted and converted to


a new cylindrical wire of 1.4 cm thickness.
So they have the same volume and radius of the new cylindrical wire, i.e. 0.7 cm.
Volume of the remaining cylinder = Volume of the new cylindrical wire
243.936  r2h
22 2
243.936   0.7  h
7
h  158.4cm

 The length of the new cylindrical wire of 1.4 cm thickness is 158.4 cm.

www.topperlearning.com 22
CBSE X | Mathematics

Board Paper – 2015 Solution All India Set – 2

26. Let  be the length of the longer side and b be the length of the shorter side.
Given that the length of the diagonal of the rectangular field is 16 metres more than
the shorter side.
Thus, diagonal = 16 + b
Since longer side is 14 metres more than shorter side, we have,
=14 + b
Diagonal is the hypotenuse of the triangle.
Consider the following figure of the rectangular field.

By applying Pythagoras Theorem in ABD, we have,


Diagonal2 Length2 Breadth2
(16 b)2 (14 b)2 b2
256 b2 32b 196 b2 28b b2
256 32b 196 28b b2
60 32b 28b b2
b2 4b 60 0
b2 10b 6b 60 0
b(b 10) 6(b 10) 0
(b 6)(b 10) 0
(b 6) 0 or (b 10) 0
b 6 or b 10
As breadth cannot be negative, breadth =10 m
Thus, length of the rectangular field=14+10=24 m

www.topperlearning.com 23
CBSE X | Mathematics

Board Paper – 2015 Solution All India Set – 2

27. Consider the given A.P. 8, 10, 12, …


Here the initial term is 8 and the common difference is 10 - 8 = 2 and 12 - 10 = 2
General term of an A.P. is tn and formula to tn is
tn a n 1 d
t 60 8 60 1 2
t 60 8 59 2
t 60 8 118
t 60 126
We need to find the sum of last 10 terms.
Thus,
Sum of last 10 terms = Sum of first 60 terms - Sum of first 50 terms
n
Sn 2a n 1 d
2
60
S60 2 8 60 1 2
2
S60 30 16 59 2
S60 30 134
S60 4020

Similarly,
50
S50 2 8 50 1 2
2
S50 25 16 49 2
S50 25 114
S50 2850
Therefore,
Sum of last 10 terms = Sum of first 60 terms Sum of first 50 terms
Thus the sum of last 10 terms =S60 S50 4020 2850 1170

www.topperlearning.com 24
CBSE X | Mathematics

Board Paper – 2015 Solution All India Set – 2

28. Let x be the initial speed of the bus.


Distance
We know that time
Speed
Thus,we have,
75 90
3 hours
x x 10
75 x+10 90x
3
x x+10
75 x+10 90x 3x x+10
75x 750 90x 3x 2 30x
2
165x 750 3x 30x
2
3x 165x 750 30x 0
2
3x 135x 750 0
x2 45x 250 0
x 2 50x 5x 250 0
x(x 50) 5(x 50) 0
(x 50) 0 or (x 5) 0
x 5 or x 50
Speed cannot be negative and hence first speed of the train is 50 km/hour.

www.topperlearning.com 25
CBSE X | Mathematics

Board Paper – 2015 Solution All India Set – 2

29.

Given: Line l is tangent to the circle  O, r  at point A.


To prove: OA  l
Proof: Let P  l , P  A.
If P is in the interior of circle  O, r  , then the line l will be a secant of the circle and not a tangent.
But l is a tangent of the circle, so P is not in the interior of the circle.
Also P  A.
P is the point in the exterior of the circle.

OP > OA. OA is the radius of the circle 
Therefore each point P l except A satisfies the inequality OP > OA.
Therefore OA is the shortest distance of line l from O.
OA  l .

www.topperlearning.com 26
CBSE X | Mathematics

Board Paper – 2015 Solution All India Set – 2

30. Steps of construction:


1) Construct the triangle as per given measurements.
2) Take any arbitrary radius and draw two arcs of circle from point B on AC,
intersecting AC at X and Y.
3) Taking X and Y as centres, draw two arcs of circles to intersect each other at point
E. Join B and E. BE is the perpendicular from B on AC.
4) BDC is a right angled. Hence, BC the hypotenuse will form the diameter of the
circle passing through the vertices of BDC.
5) BC = 8 cm OC = 4 cm. draw a circle of radius equal 4 cm, passing through B, D
and C.
6) Join O and A. Obtain the mid-point P of segment OA by drawing perpendicular
bisector to OA.
7) Draw a circle with centre P and radius AP.
8) Let B and F be the points of intersection of these two circles.
Hence, AB and AF are the required tangents.

www.topperlearning.com 27
CBSE X | Mathematics

Board Paper – 2015 Solution All India Set – 2

31. Let A(k + 1, 1), B(4, -3) and C(7, - k) are the vertices of the triangle.
Given that the area of the triangle is 6 sq. units.
Area of the triangle is given by
1
 
A=  x1  y 2  y 3   x 2  y 3  y 1   x3 y 1  y 2 
2 
1
 
 6   x1  y 2  y 3   x 2  y 3  y 1   x3 y 1  y 2 
2 
 12   k  1 3  k   4  k  1   7 1  3
 12  3k  k 2  3  k  4k  4  28
 12  k 2  6k  21
 k 2  6k  21  12  0
 k 2  6k  9  0
  k  3  0
2

 k  3,3

www.topperlearning.com 28
CBSE X | Mathematics

Board Paper – 2015 Solution All India Set – 3

CBSE
Class X Mathematics
Board Paper – 2015 Solution
All India Set – 3
Time: 3 hours Total Marks: 90

1.
Let AB be the tower and BC be its shadow.
AB  20, BC  20 3
In ABC,
AB
tan  
BC
20
tan  
20 3
1
tan  
3
1
but, tan30 
3
  30
 The Sun is at an altitude of 30 .

2.
Two dice are tossed
S = [(1,1),(1,2),(1,3),(1,4),(1,5),(1,6),
(2,1),(2,2),(2,3),(2,4),(2,5),(2,6),
(3,1),(3,2),(3,3),(3,4),(3,5),(3,6),
(4,1),(4,2),(4,3),(4,4),(4,5),(4,6),
(5,1),(5,2),(5,3),(5,4),(5,5),(5,6),
(6,1),(6,2),(6,3),(6,4),(6,5),(6,6)]
Total number of outcomes when two dice are tossed = 6 x 6= 36
Favourable events of getting the product as 6 are:
(1  6  6), (6  1  6),(2  3  6),(3  2  6)
i.e.(1,6), (6,1), (2,3), (3,2)
Favourable events of getting product as 6 = 4
4 1
P(getting product as 6) = 
36 9

www.topperlearning.com 1
CBSE X | Mathematics

Board Paper – 2015 Solution All India Set – 3

3.
Given quadratic equation is,
px 2  2 5px  15  0
Here,a  p, b  2 5p, c  15
For real equal roots, discriminant  0
 b2  4ac  0

 
2
 2 5p  4p 15  0
20p2  60p  0
20p  p  3  0
 p  3 or p  0
But, p  0 is not possible.
p  3

www.topperlearning.com 2
CBSE X | Mathematics

Board Paper – 2015 Solution All India Set – 3

4.

mOPT  90  radius is perpendicular to the tangent 


So, OPQ = OPT  QPT
= 90  60
= 30
mPOQ = 2mQPT  2  60  120
reflex mPOQ = 360  120  240
1
PRQ = reflex POQ
2
1
=  240
2
 120
 mPRQ  120

www.topperlearning.com 3
CBSE X | Mathematics

Board Paper – 2015 Solution All India Set – 3

SECTION B

5.
S5  S7  167 and S10  235
n
Now, Sn 
2
2a   n  1 d
S5  S7  167
5 7
 2a  4d  2a  6d  167
2 2
 5a  10d  7a  21d  167
 12a  31d  167 ....(1)
Also, S10  235
10
 2a  9d  235
2
 10a  45d  235
 2a  9d  47 ....(2)
Multiplying equation (2) by 6, we get
12a  54d  282 .....(3)
Subtracting (1) from (3), we get
12a  54d  282
   12a  31d  167
  
23d  115
d  5
Substituting value of d in (2), we have
2a  9(5)  47
 2a  45  47
 2a  2
a 1
Thus, the given A.P. is 1, 6, 11, 16,..........

www.topperlearning.com 4
CBSE X | Mathematics

Board Paper – 2015 Solution All India Set – 3

6.
ABC is right triangle at B.
 AC2  AB2  BC2 ....(1)
Also, A   4,7  , B   p,3 and C  7,3

Now, AC2  7  4   3  7   3   4   9  16  25


2 2 2 2

AB2   p  4   3  7   p2  8p  16   4 
2 2 2

 p2  8p  16  16
 p2  8p  32
BC2  7  p   3  3  49  14p  p2  0
2 2

 p2  14p  49
From (1), we have
  
25  p2  8p  32  p2  14p  49 
 25  2p2  22p  81
 2p2  22p  56  0
 p2  11p  28  0
 p2  7p  4p  28  0
 p p  7  4p  7  0
  p  7  p  4   0
 p  7 and p  4

www.topperlearning.com 5
CBSE X | Mathematics

Board Paper – 2015 Solution All India Set – 3

7.
Given that m PRQ = 120
We know that the line joining the centre and
the external point is the angle bisector between
the tangents.
120
Thus, m PRO = m QRO = 60
2
Also we know that lengths of tangents from an external point
are equal.
Thus, PR = RQ.
Join OP and OQ.
Since OP and OQ are the radii from the centre O,
OP PR and OQ RQ.
Thus, OPR and OQR are right angled congruent triangles.
Hence, POR=90 PRO=90 60 30
QOR=90 QRO=90 60 30
1
sin QRO = sin30
2
PR
But sin30
OR
PR 1
Thus,
OR 2
OR 2PR
OR PR PR
OR PR QR

www.topperlearning.com 6
CBSE X | Mathematics

Board Paper – 2015 Solution All India Set – 3

8. Let the given circle touch the sides AB and AC of the triangle at points F and E
respectively and let the length of line segment AF be x.
Now, it can be observed that:
BF = BD = 6 cm (tangents from point B)
CE = CD = 9 cm (tangents from point C)
AE = AF = x (tangents from point A)

AB = AF + FB = x + 6
BC = BD + DC = 6 + 9 = 15
CA = CE + EA = 9 + x
2s = AB + BC + CA = x + 6 + 15 + 9 + x = 30 + 2x
s = 15 + x
s – a = 15 + x – 15 = x
s – b = 15 + x – (x + 9) = 6
s – c = 15 + x – (6 + x) = 9
Area of ABC = s  s  a  s  b  s  c 

54  15  x  x 6  9

54  3 6 15x  x 2 

18  6 15x  x 2 
324  6 15x  x  2

54  15x  x2
x2  15x  54  0
x2  18x  3x  54  0
x(x  18)  3(x  18)
 x  18  x  3  0
x  18 and x  3
As distance cannot be negative, x = 3
AC = 3 + 9 = 12
AB = AF + FB = 6 + x = 6 + 3 = 9

www.topperlearning.com 7
CBSE X | Mathematics

Board Paper – 2015 Solution All India Set – 3

9.
Given, the point s A(x,y), B( 5,7) and C( 4,5) are collinear.
So, the area formed by these vertices is 0.
1
  x 7  5   55  y    4  y  7    0
2
1
 2x  25  5y  4y  28  0
2
1
 2x  y  3  0
2
 2x  y  3  0
 y  2x  3

10. x2 – 2ax – (4b2 – a2) = 0


⇒ x2 + (2b – a)x – (2b + a)x – (4b2 – a2) = 0
⇒ x(x + 2b – a) – (2b + a)(x + 2b – a) = 0
⇒ (x + 2b – a)(x – 2b – a) = 0
⇒ (x + 2b – a) = 0, (x – 2b – a) = 0
 x = a − 2b, a + 2b

www.topperlearning.com 8
CBSE X | Mathematics

Board Paper – 2015 Solution All India Set – 3

SECTION C

11. Diameter of the tent = 4.2 m


Radius of the tent, r = 2.1 m
Height of the cylindrical part of tent, hcylinder = 4 m
Height of the conical part, hcone = 2.8 m
Slant height of the conical part, 

hcone2 r2

2.82 2.12
2.82 2.12
12.25 3.5 m
Curved surface area of the cylinder = 2𝜋r hcylinder
22
=2× × 2.1 × 4
7
= 22 × 0.3 × 8 = 52.8 m2
22
Curved surface area of the conical tent = 𝜋rl = × 2.1 × 3.5 = 23.1 m2
7
Total area of cloth required for building one tent
= Curved surface area of the cylinder + Curved surface area of the conical tent
= 52.8 + 23.1
= 75.9 m2
Cost of building one tent = 75.9 × 100 = Rs. 7590
Total cost of 100 tents = 7590 × 100 = Rs. 7,59,000
759000
Cost to be borne by the associations = = Rs. 3,79,500
2
It shows the helping nature, unity and cooperativeness of the associations.

www.topperlearning.com 9
CBSE X | Mathematics

Board Paper – 2015 Solution All India Set – 3

12. Internal diameter of the bowl = 36 cm


Internal radius of the bowl, r = 18 cm
2 2
Volume of the liquid, V = 𝜋r3 = × 𝜋 × 183
3 3
Let the height of the small bottle be ‘h’.
Diameter of a small cylindrical bottle = 6 cm
Radius of a small bottle, R = 3 cm
Volume of a single bottle = 𝜋R2h = 𝜋 × 32 × h
No. of small bottles, n = 72
10 2
Volume wasted in the transfer = × × 𝜋 × 183
100 3
Volume of liquid to be transferred in the bottles
2 10 2
= × 𝜋 × 183 − × × 𝜋 × 183
3 100 3
2 10
= × 𝜋 × 183 1
3 100
2 90
= × 𝜋 × 183 ×
3 100
Volume of the liquid to be transferred
Number of the small cylindrical bottles =
Volume of a single bottle
2 90
183
72 3 100
32 h
2 9
183
72 3 2 10
3 h
2 9
18 18 18
h 3 10
32 72
h 5.4 cm

Height of the small cylindrical bottle = 5.4 cm

www.topperlearning.com 10
CBSE X | Mathematics

Board Paper – 2015 Solution All India Set – 3

13. Side of the cubical block, a = 10 cm


Longest diagonal of the cubical block = a√3 = 10√3 cm
Since the cube is surmounted by a hemisphere, therefore the side of the cube should
be equal to the diameter of the hemisphere.
Diameter of the sphere = 10 cm
Radius of the sphere, r = 5 cm
Total surface area of the solid = Total surface area of the cube – Inner cross-section
area of the hemisphere + Curved surface area of the hemisphere
= 6a2 – 𝜋r2 + 2𝜋r2
= 6a2 + 𝜋r2
= 6 × (10)2 + 3.14 52
600 78.5 678.5 cm2
Total surface area of the solid = 678.5 cm2
Cost of painting 100 cm2 = Rs. 5
5
Cost of painting 1 cm2 = Rs.
100
5
Cost of painting the total surface area of the solid = × 678.5 = Rs. 33.925  Rs. 34.
100

www.topperlearning.com 11
CBSE X | Mathematics

Board Paper – 2015 Solution All India Set – 3

14. No. of cones = 504


Diameter of a cone = 3.5 cm
Radius of the cone, r = 1.75 cm
Height of the cone, h = 3 cm
Volume of a cone
1 2
rh
3
2
1 3.5
3
3 2
1 3.5 3.5
3 cm3
3 2 2
Volume of 504 cones
1 3.5 3.5
504 3cm3
3 2 2
Let the radius of the new sphere be ‘R’.
4 3
Volume of the sphere R
3
Volume of 504 cones = Volume of the sphere
1 3.5 3.5 4 3
504 3 R
3 2 2 3
504 1 3.5 3.5 3 3
R3
3 2 2 4
504 3 49
R3
64
7 8 9 3 72
R3
64
8 27 73
R3
64
2 3 7
R
4
21
R 10.5 cm
2
Radius of the new sphere = 10.5 cm
Surface area of the new sphere=4R 2
22 21 21
 4  
7 2 2
 2772 cm2

www.topperlearning.com 12
CBSE X | Mathematics

Board Paper – 2015 Solution All India Set – 3

15. √3 x2 – 2√2x – 2√3 = 0


Here a = √3, b = − 2√2, c = − 2√3
( 2 2)  ( 2 2)2  4  3  ( 2 3)
x 
2 3
2 2  8  24

2 3
2 2  32

2 3
2 24 2

2 3
2 2 4 2 2 2 4 2
 ,
2 3 2 3
3 2 2 2
 ,
3 2 3
2
x  6 , 
3

www.topperlearning.com 13
CBSE X | Mathematics

Board Paper – 2015 Solution All India Set – 3

16.
Let BC be the height at which the aeroplane is observed from point A.
Then, BC = 1500 3
In 15 seconds, the aeroplane moves from point A to D.
A and D are the points where the angles of elevation 60 and 30
are formed respectively.
Let BA = x metres and AD  y metres
BC = x + y

In CBA,
BC
tan60°=
BA
1500 3
3
x
 x  1500 m ....(1)

In CBD,
BC
tan30°=
BD
1 1500 3

3 xy
 x  y  1500(3)  4500
1500  y  4500
 y  3000 m ....(2)
We know that the aeroplane moves from point A to D in 15 seconds and the distance
covered is 3000 metres. (by 2)

www.topperlearning.com 14
CBSE X | Mathematics

Board Paper – 2015 Solution All India Set – 3

dis tance
Speed 
time
3000
Speed 
15
Speed  200m/s

18
Converting it to km/hr = 200   720 km/hr
5

17. Radius of the circle = 14 cm


Central Angle, 𝜽 = 60,
Area of the minor segment
1 2
r2 r sin
360 2
60 1
142 142 sin60
360 2
1 22 1 3
14 14 14 14
6 7 2 2
22 14
49 3
3
22 14 147 3
3 3
308 147 3 2
cm
3
308 147 3 2
Area of the minor segment cm
3

www.topperlearning.com 15
CBSE X | Mathematics

Board Paper – 2015 Solution All India Set – 3

18. Let the first term be ‘a’ and the common difference be ‘d’ of the A.P.
t13 = 4t3
⇒ a + 12d = 4(a + 2d)
⇒ a + 12d = 4a + 8d
⇒ 4d = 3a
4d
⇒a=
3
t5 = 16
⇒ a + 4d = 16
4d
⇒ + 4d = 16
3
4d  12d
⇒ = 16
3
16d
⇒ = 16
3
d=3
n
Sn = [2a + (n – 1)d]
2
10
S10 = [2 × 10 + (10 – 1) × 3]
2
= 5 [20 + 27]
= 5 × 47
= 235
Sum of the first 10 terms = 235.

19.
2
AP  AB
5
AP 2

AB 5
AP 2

PB 3
P divides AB in the ratio of 2 : 3 internally
 6  2  13 7  2  23 
 23 , 23 
 
 15 20 
 , 
 5 5 
 3,4 
Co-ordinates of P(3, 4)

www.topperlearning.com 16
CBSE X | Mathematics

Board Paper – 2015 Solution All India Set – 3

20. n(total balls in the bag) = x


2 20 number of black balls
P(getting a black ball) =  
5 50 total number of balls
Number of black balls in the bag =20
Total number of balls = 50
3 15 number of white balls
P(getting a white ball) =  
10 50 total number of balls
Number of white balls in the bag = 15
Number of red ball
= Total number of ball – (Number of black ball + Number of white ball)
= 50 – (20 + 15)
= 50 – 35
= 15
Total number of balls in the bag = 15.

www.topperlearning.com 17
CBSE X | Mathematics

Board Paper – 2015 Solution All India Set – 3

SECTION D
21.

Let PB be the surface of the lake and A be the point of observation such that
AP = 20 metres. Let C be the position of the cloud and C’ be its reflection in the lake.
Then CB = C’B. Let AM be perpendicular from A on CB.
Then mCAM  30 and mC'AM  60
Let CM = h. Then CB = h + 20 and C’B = h + 20. (CB=CB’ since refection about PB)
In CMA we have,
CM
tan30 
AM
1 h
 
3 AM
 AM  3h...................(i)

In AMC' we have,
C'M
tan 60 
AM
C'B  BM
 3
AM
h  20  20
 3
AM
h  20  20
 AM  ...................(ii)
3
From equation (i) and (ii), we get
h  20  20
3h 
3
 3h  h  40
 2h  40
 h  20 m
 AM  20 3
Now , to find AC u sin g pythagoras theorem
AC2  AM2  MC2
 1600
AC  40
Hence, the height of the cloud from the
surface of the lake is 40 metres.

www.topperlearning.com 18
CBSE X | Mathematics

Board Paper – 2015 Solution All India Set – 3

22.
Let S be the sample space of drawing a card from a well-shuffled deck.
n  S   52 C1  52

(i)There are 13 spade cards and 4 ace's in a deck


As ace of spade is included in 13 spade cards,
so there are 13 spade cards and 3 ace's

A card of spade or an ace can be drawn in 13


C1  3 C1  13  3  16
16 4
Probability of drawing a card of spade or an ace = 
52 13

(ii)There are 2 black king cards in a deck


A card of black king can be drawn in 2 C1  2
2 1
Probability of drawing a black king = 
52 26

(iii)There are 4 jack and 4 king cards in a deck.


So there are 52  8 = 44 cards which are neither jacks nor kings.
A card which is neither a jack nor a king can be drawn in 44
C1  44
44 11
Probability of drawing a card which is neither a jack nor a king = 
52 13

(iv)There are 4 king and 4 queen cards in a deck.


So there are 4  4 =8 cards which are either king or queen.
a card which is either a king or a queen can be drawn in 8 C1  8
8 2
Probability of drawing a card which is either a king or a queen = 
52 13

www.topperlearning.com 19
CBSE X | Mathematics

Board Paper – 2015 Solution All India Set – 3

23. PQRS is a square.


So each side is equal and angle between the adjacent sides is a right angle.
Also the diagonals perpendicularly bisect each other.
In PQR using pythagoras theorem,
PR 2 = PQ2 + QR 2
2 2
PR 2   42   42
PR = 2  42
1 42
OR  PR=  OQ
2 2
From the figure we can see that the radius of the flower bed ORQ is OR.
1
Area of sector ORQ  r 2
4
2
1  42 
 
4  2 
1
Area of the ROQ =  RO  OQ
2
1 42 42
  
2 2 2
2
 42 
 
 2 
Area of the flower bed ORQ
=Area of sector ORQ  Area of the ROQ
2 2
1  42   42 
=  
4  2   2 
2
 42    
     1
 2  2 
  441  0.57
 251.37cm2

Area of the flower bed ORQ = Area of the flower bed OPS
= 251.37cm2

Total area of the two flower beds


= Area of the flower bed ORQ + Area of the flower bed OPS
= 251.37  251.37
 502.74cm2

www.topperlearning.com 20
CBSE X | Mathematics

Board Paper – 2015 Solution All India Set – 3

24. Height of the cylinder (h) = 10 cm


Radius of the base of the cylinder = 4.2 cm
Volume of original cylinder = r2h
22 2
   4.2  10
7
 554.4 cm3
2
Volume of hemisphere = r3
3
2 22 3
    4.2
3 7
 155.232 cm3

Volume of the remaining cylinder after scooping out the hemisphere from each end
 Volume of original cylinder  2  Volume of hemisphere
 554.4  2  155.232
 243.936cm3

The remaining cylinder is melted and converted to


a new cylindrical wire of 1.4 cm thickness.
So they have same volume and radius of new cylindrical wire is 0.7 cm.
Volume of the remaining cylinder = Volume of the new cylindrical wire
243.936  r2h
22 2
243.936   0.7  h
7
h  158.4cm

 The length of the new cylindrical wire of 1.4 cm thickness is 158.4 cm.

www.topperlearning.com 21
CBSE X | Mathematics

Board Paper – 2015 Solution All India Set – 3

25. Let  be the length of the longer side and b be the length of the shorter side.
Given that the length of the diagonal of the rectangular field is 16 metres more than
the shorter side.
Thus, diagonal = 16 + b
Since longer side is 14 metres more than shorter side, we have,
= 14 + b
Diagonal is the hypotenuse of the triangle.
Consider the following figure of the rectangular field.

By applying Pythagoras Theorem in ABD, we have,


Diagonal2 Length2 Breadth2
(16 b)2 (14 b)2 b2
256 b2 32b 196 b2 28b b2
256 32b 196 28b b2
60 32b 28b b2
b2 4b 60 0
2
b 10b 6b 60 0
b(b 10) 6(b 10) 0
(b 6)(b 10) 0
(b 6) 0 or (b 10) 0
b 6 or b 10
As breadth cannot be negative, breadth = 10 m
Thus, length of the rectangular field = 14 + 10 = 24 m

www.topperlearning.com 22
CBSE X | Mathematics

Board Paper – 2015 Solution All India Set – 3

26. Consider the following diagram.

Let P be an external point and PA and PB are tangents to the circle.


We need to prove that PA = PB
Now consider the triangles OAP and OBP
mA = mB = 90
OP = OP [common]
OA = OB = radii of the circle
Thus, by Right Angle-Hypotenuse-Side criterion of congruence we have,
OAP  OBP
The corresponding parts of the congruent triangles are congruent.
Thus,
PA = PB

www.topperlearning.com 23
CBSE X | Mathematics

Board Paper – 2015 Solution All India Set – 3

27. In the figure, C is the midpoint of the minor arc PQ, O is the centre of the circle and
AB is tangent to the circle through point C.
We have to show that the tangent drawn at the midpoint of the arc PQ of a circle is
parallel to the chord joining the end points of the arc PQ.
We will show that PQ AB.
It is given that C is the midpoint point of the arc PQ.
So, arc PC = arc CQ.
PC = CQ

This shows that PQC is an isosceles triangle.


Thus, the perpendicular bisector of the side PQ of PQC passes through vertex C.
The perpendicular bisector of a chord passes through the centre of the circle.
So the perpendicular bisector of PQ passes through the centre O of the circle.
Thus perpendicular bisector of PQ passes through the points O and C.
PQ  OC
AB is the tangent to the circle through the point C on the circle.
AB  OC
The chord PQ and the tangent PQ of the circle are perpendicular to the same line OC.
PQ  AB.

www.topperlearning.com 24
CBSE X | Mathematics

Board Paper – 2015 Solution All India Set – 3

28. Let the original speed of the truck = s km/hr


New speed of the truck = (s + 20) km/hr
Time taken for 150 km + Time taken for 200 km = 5
150 200
 5
s (s  20)
150s  3000  200s
 5
s(s  20)
350s  3000
 5
s(s  20)
50(7s  60)
 5
s(s  20)
 10(7s  60)  s(s  20)
 70s  600  s2  20s
 s2  50s  600  0
 s2  60s  10s  600  0
 s(s  60)  10(s  60)  0
 (s  10)(s  60)  0
 s  10,60
s  10[Not possible]
 First speed of the truck = 60 km/hr

29. 5, 12, 19, …………50 terms


Common difference, d = 7
First term, a = 5
Last term, t50 = a + (50 – 1)d = 5 + (50 – 1) × 7 = 5 + 49 × 7 = 5 + 343 = 348
Sum of last 15 terms = S50 – S35
50 35
[2  5  (50  1)  7]  [2  5  (35  1)  7]
2 2
35
 25[10  343]  [10  34  7]
2
35
 25  353   248
2
 8825  4340
 4485
The sum of last 15 terms = 4485.

www.topperlearning.com 25
CBSE X | Mathematics

Board Paper – 2015 Solution All India Set – 3

30.
Steps for construction:
1. Draw a line segment AB = 5 cm.
2. At B construct mABC = 60
3. Take a measure of 6 cm, and draw an arc from B on BC.
4. Join AC to obtain ABC.
5. Below AB, make an acute angle BAX.
6. Since 7 > 5, mark off 7 points A1, A2, A3, A4, A5, A6 and A7 such that
AA1 = A1A2 = A2A3 = A3A4 = A4A5 = A5A6 = A6A7.
7. Join A7B.
5
8. Since we have to construct a triangle each of whose sides is of the
7
corresponding sides of ABC. So take five parts out of seven equal parts on AX.
i.e. from point A5, draw A5B' || A7B, meeting AB at B'.
9. From B', draw B'C' || BC, meeting AC at C'
10. AB'C' is the required, each of the sides is five-seventh of the corresponding
sides of ABC.

www.topperlearning.com 26
CBSE X | Mathematics

Board Paper – 2015 Solution All India Set – 3

31. A(k + 1, 2k) , B(3k, 2k + 3) and (5k – 1, 5k)


If 3 points are collinear then area of triangle formed by them = 0
1
(k  1)(2k  3  5k)  2k(3k  5k  1)  1(15k 2  10k 2  2k  15k  3)  0
2
1
 3k 2  3k  3k  3  4k 2  2k  15k 2  10k 2  2k  15k  3  0
2
1
6k 2  11k   0
2
6k 2  11k  0
11
k  0, 
6

www.topperlearning.com 27
CBSE X | Mathematics

Board Paper – 2016 Solution All India Set – 1

CBSE
Class X Mathematics
Board Paper – 2016 Solution
All India Set – 1
Time: 3 hours Total Marks: 90

SECTION A

1. In the given figure,


In ΔACO,
OA = OC …(Radii of the same circle)
 ΔACO is an isosceles triangle.
∠CAB = 30° …(Given)
 ∠CAO = ∠ACO = 30°
…(angles opposite to equal sides of an isosceles
triangle are equal)
∠PCO = 90° …(radius drawn at the point of contact is
perpendicular to the tangent)
Now ∠PCA = ∠PCO – ∠CAO
 ∠PCA = 90° – 30° = 60°

2. If k + 9, 2k – 1 and 2k + 7 are the consecutive terms of A.P., then the


common difference will be the same.
 (2k – 1) – (k + 9) = (2k + 7) – (2k – 1)
 k – 10 = 8
 k = 18

3. Let AB be the ladder and CA be the wall.


The ladder makes an angle of 60° with the horizontal.
 ΔABC is a 30°-60°-90°, right triangle.
Given: BC = 2.5 m, ∠ABC = 60°
 ∠BAC = 30°
 AB = 5 cm

4. There are 26 red cards including 2 red queens.


Two more queens along with 26 red cards will be 26 + 2 = 28
28
 P(getting a red card or a queen) 
52
28 24 6
 P(getting neither a red card nor a queen)  1   
52 52 13

www.topperlearning.com 1
CBSE X | Mathematics

Board Paper – 2016 Solution All India Set – 1

SECTION B

5. Given –5 is a root of the quadratic equation 2x2 + px – 15 = 0.


 5 satisfies the given equation.
 2  5  p  5   15  0
2

 50  5p  15  0
 35  5p  0
 5p  35  p  7
 
Substituting p  7 in p x2  x  k  0, we get

 2
7 x x k  0 
 7x2  7x  k  0
The roots of the equation are equal.
 Discriminant  b2  4ac  0
Here, a  7,b  7, c  k
b2  4ac  0
  7   4  7  k   0
2

 49  28k  0
 28k  49
49 7
k  
28 4

6. Since P and Q are the points of trisection of AB, AP = PQ = QB


Thus, P divides AB internally in the ratio 1 : 2
and Q divides AB internally in the ratio 2 : 1.
 By section formula,
 1  7   2 2  1  4   2  2    7  4 4  4   3 
P
 1  2
,
1  2
  
3
,
3    3 ,0    1,0 
     
 2  7   1 2  2  4   1  2    14  2 8  2   12 6 
Q ,    ,  ,   4,2 

 2 1 2 1   3 3   3 3 

7. Since tangents drawn from an exterior point to a circle are equal in length,
AP = AS ….(1)
BP = BQ ….(2)
CR = CQ ….(3)
DR = DS ….(4)
Adding equations (1), (2), (3) and (4), we get
AP + BP + CR + DS = AS + BQ + CQ + DS
 (AP + BP) + (CR + DR) = (AS + DS) + (BQ + CQ)
 AB + CD = AD + BC
 AB + CD = BC + DA …..(proved)

www.topperlearning.com 2
CBSE X | Mathematics

Board Paper – 2016 Solution All India Set – 1

8. Let A(3, 0), B(6, 4) and C(–1, 3) be the given points.


Now,

6  3   4  0   32  42  9  16  25
2 2
AB 

 1  6   3  4    7    1 
2 2 2 2
BC  49  1  50

 1  3  3  0    4 
2 2 2
AC   32  16  9  25
 AB  AC

   25
2
AB2  25

BC2  50   50

AC2  25   25

 AB2  AC2  BC2


Thus, ABC is a right-angled isosceles triangle.

9. 4th term of an A.P.= a4 = 0


 a + (4 – 1)d = 0
 a + 3d = 0
 a = –3d ….(1)
25 term of an A.P. = a25
th

= a + (25 – 1)d
= –3d + 24d ….[From (1)]
= 21d
3 times 11 term of an A.P. = 3a11
th

= 3[a + (11 – 1)d]


= 3[a + 10d]
= 3[–3d + 10d]
= 3 × 7d
= 21d
 a25 = 3a11
i.e., the 25th term of the A.P. is three times its 11th term.

www.topperlearning.com 3
CBSE X | Mathematics

Board Paper – 2016 Solution All India Set – 1

10. In the given figure,


OP = 2r … (Given)
∠OTP = 90° … (radius drawn at the point of contact is perpendicular to the
tangent)
In OTP,
OT 1
sinOPT =   sin30
OP 2
 OPT = 30
 ∠TOP = 60°
 ΔOTP is a 30°-60°-90°, right triangle.
In ΔOTS,
OT = OS … (Radii of the same circle)
 ΔOTS is an isosceles triangle.
 ∠OTS = ∠OST … (Angles opposite to equal sides of an isosceles triangle are
equal)

In ΔOTQ and ΔOSQ


OS = OT … (Radii of the same circle)
OQ = OQ ...(side common to both triangles)
∠OTQ = ∠OSQ … (angles opposite to equal sides of an isosceles triangle are
equal)
 ΔOTQ ≅ ΔOSQ … (By S.A.S)

 ∠TOQ = ∠SOQ = 60° … (C.A.C.T)

 ∠TOS = 120° … (∠TOS = ∠TOQ + ∠SOQ = 60° + 60° = 120°)


 ∠OTS + ∠OST = 180° – 120° = 60°
 ∠OTS = ∠OST = 60° ÷ 2 = 30°

www.topperlearning.com 4
CBSE X | Mathematics

Board Paper – 2016 Solution All India Set – 1

SECTION C

11. Diameter, AB = 13 cm
13
 Radius of the circle, r   6.5 cm
2
ACB is the angle in the semi-circle.
 ACB  90
Now, in ACB, using Pythagoras theorem, we have
AB2  AC2  BC2
 13  12   BC 
2 2 2

 BC   13  12   169  144  25


2 2 2

 BC  25  5 cm
Now, area of shaded region  Area of semi-circle  Area of ACB
1 1
 r2   BC  AC
2 2
1 1
  3.14   6.5   5  12
2

2 2
 66.33  30
 36.33 cm2
Thus, the area of the shaded region is 36.33 cm2.

12. For conical portion, we have


r = 1.5 m and l = 2.8 m
 S1 = Curved surface area of conical portion
 S1 = πrl
= π × 1.5 × 2.8
= 4.2π m2

For cylindrical portion, we have


r = 1.5 m and h = 2.1 m
 S2 = Curved surface area of cylindrical portion
 S2 = 2πrh
= 2 × π × 1.5 × 2.1
= 6.3π m2

Area of canvas used for making the tent  S1  S2


 4.2  6.3
 10.5
22
 10.5 
7
 33 m2
Total cost of the canvas at the rate of Rs. 500 per m2  Rs. 500  33  Rs. 16500

www.topperlearning.com 5
CBSE X | Mathematics

Board Paper – 2016 Solution All India Set – 1

13. P(x, y) is equidistant from the points A(a + b, b – a) and B(a – b, a + b).
 AP = BP
 x   a  b    y  b  a   x   a  b    y   a  b  
2 2 2 2

 x   a  b    y  b  a   x   a  b    y   a  b  


2 2 2 2

 x2  2x  a  b    a  b   y2  2y b  a  b  a
2 2

 x2  2x  a  b    a  b   y2  2y  a  b    a  b 
2 2

 2x  a  b   2y b  a  2x  a  b   2y  a  b 
 ax  bx  by  ay  ax  bx  ay  by
 2bx  2ay
 bx  ay ....(proved)

14. Area of the region ABDC = Area of sector AOC – Area of sector BOD
40 22 40 22
   14  14   77
360 7 360 7
1 1
  22  14  2   22  7  1
9 9
22
  28  7 
9
22
  21
9
154

3
 51.33 cm2
22 22
Area of circular ring   14  14  77
7 7
 22  14  2  22  7  1
 22  28  7 
 22  21
 462 cm2
 Re quired shaded region  Area of circular ring  Area of region ABDC
 462  51.33
 410.67 cm2
Thus, the area of shaded region is 410.67 cm2.

www.topperlearning.com 6
CBSE X | Mathematics

Board Paper – 2016 Solution All India Set – 1

15. Let a1, a2 be the first terms and d1, d2 the common differences of the two
given A.P’s.
n n
Then, we have Sn  2a1  n  1 d1  and Sn'  2a2  n  1 d2 
2 2
n
2a1  n  1 d1  2a  n  1 d
Sn
 '  2  1   1
2a2  n  1 d2  2a2  n  1 d2
Sn n
2
S 7n  1
It is given that n' 
Sn 4n  27
2a1  n  1 d1 7n  1
  ....(1)
2a2  n  1 d2 4n  27
To find the ratio of the mth terms of the two given A.P's,
replace n by (2m  1) in equation (1).
2a1  2m  1  1 d1 7 2m  1  1
 
2a2  2m  1  1 d2 4 2m  1  27
2a1  2m  2  d1 14m  7  1
 
2a2  2m  2  d2 8m  4  27
a1  m  1 d1 14m  6
 
a2  m  1 d2 8m  23
Hence, the ratio of the mth terms of the two A.P's is 14m  6 : 8m  23.

www.topperlearning.com 7
CBSE X | Mathematics

Board Paper – 2016 Solution All India Set – 1

1 1 2
16.  
 x  1 x  2  x  2 x  3 3


 x  3    x  1  2
 x  1 x  2  x  3 3
x 3  x 1 2
 
x 2

 2x  x  2  x  3  3
2x  4 2
 
x 2

 3x  2  x  3  3
2x  4 2
 3 2 2

x  3x  3x  9x  2x  6 3
2x  4 2
 3 2

x  6x  11x  6 3
 6x  12  2x3  12x2  22x  12
 2x3  12x2  16x  0

 2x x2  6x  8  0 

 x2  6x  8  0 
2
 x  4x  2x  8  0
 x  x  4  2  x  4  0
  x  4  x  2   0
 x  4  0 or x  2  0
x  4 or x  2

17. Let the radius of the conical vessel = r1 = 5 cm


Height of the conical vessel = h1 = 24 cm
Radius of the cylindrical vessel = r2
Let the water rise upto the height of h2 cm in the cylindrical vessel.
Now, volume of water in conical vessel = volume of water in cylindrical vessel
1
 r12h1  r22h2
3
 r12h1  3r22h2
 5  5  24  3  10  10  h2
5  5  24
 h2   2 cm
3  10  10
Thus, the water will rise upto the height of 2 cm in the cylindrical vessel.

www.topperlearning.com 8
CBSE X | Mathematics

Board Paper – 2016 Solution All India Set – 1

18. Radius of sphere = r = 6 cm


4 4
Volume of sphere  r 3    6   288 cm3
3

3 3
Let R be the radius of cylindrical vessel.
5 32
Reise in the water level of cylindrical vessel  h  3 cm  cm
9 9
32 32
Increase in volume of cylindrical vessel  R 2h  R 2   R 2
9 9
Now, volume of water displaced by the sphere is equal to volume of sphere.
32
 R 2  288
9
288  9
 R2   81
32
 R  9 cm
 Diameter of the cylindrical vessel  2  R  2  9  18 cm

19. Let CD be the hill and suppose the man is standing on the
deck of a ship at point A.
The angle of depression of the base C of the hill CD
observed from A is 30° and the angle of elevation of the top
D of the hill CD observed from A is 60°.
 ∠EAD = 60° and ∠BCA = 30°
In ΔAED,
DE
tan60 
EA
h
 3 
x
 h  3x ....(1)
In ABC,
AB
tan30 
BC
1 10
 
3 x
 x  10 3 ....(2)
Substituting x  10 3 in equation (1), we get
h  3  10 3  10  3  30
 DE  30 m
 CD  CE  ED  10  30  40 m
Thus, the distance of the hill from the ship is 10 3 m and
the height of the hill is 40 m.

www.topperlearning.com 9
CBSE X | Mathematics

Board Paper – 2016 Solution All India Set – 1

20. When three coins are tossed together, the possible outcomes are
HHH, HTH, HHT, THH, THT, TTH, HTT, TTT
 Total number of possible outcomes = 8
(i) Favourable outcomes of exactly two heads are HTH, HHT, THH
 Total number of favourable outcomes  3
3
 P(exactly two heads) 
8
(ii) Favourable outcomes of at least two heads are HHH, HTH, HHT, THH
 Total number of favourable outcomes  4
4 1
 P(at least two heads)  
8 2
(iii)Favourable outcomes of at least two tails are THT, TTH, HTT, TTT
 Total number of favourable outcomes  4
4 1
 P(at least two tails)  
8 2

SECTION D

21. Height of conical upper part = 3.5 m, and radius = 2.8 m


(Slant height of cone)2 2.12 2.82 4.41 7.84
Slant height of cone 12.25 3.5 m
The canvas used for each tent
curved surface area of cylindrical base curved surface area of conical upper part
2 rh rl
r(2h l)
22
2.8(7 3.5)
7
22
2.8 10.5
7
92.4 m2
So, the canvas used for one tent is 92.4 m2.
Thus, the canvas used for 1500 tents (92.4 1500) m2.
Canvas used to make the tents cos t Rs. 120 per sq. m
So, canvas used to make 1500 tents will
cos t Rs. 92.4 1500 120.
The amount shared by each school to set up the tents
92.4 1500 120
Rs.332640
50
The amount shared by each school to set up the tents is Rs.332640.
The value to help others in times of troubles is generated from the problem.

www.topperlearning.com 10
CBSE X | Mathematics

Board Paper – 2016 Solution All India Set – 1

22. Consider a circle centered at O.


Let PR and QR are tangents drawn from an external point R to the circle
touching at points P and Q respectively.
Join OR.

Proof:
In OPR and OQR,
OP = OQ ... (Radii of the same circle)
∠OPR = ∠OQR …. (Since PR and QR are
tangents to the circle)
OR = OR ... (Common side)
 OPR ≅ OQ R ….(By R.H.S)
 PR = QR ….(c.p.c.t)
Thus, tangents drawn from an external point to a circle are equal.

23. Steps of construction:


(i) Take a point O on the plane of the paper and draw a circle of radius
OA = 4 cm.
(ii) Produce OA to B such that OA = AB = 4 cm.
(iii) Draw a circle with centre at A and radius AB.
(iv) Suppose it cuts the circle drawn in step (i) at P and Q.
(v) Join BP and BQ to get the desired tangents.

Justification:
In OAP, OA = OP = 4 cm ...(radii of the same circle)
Also, AP = 4 cm ….(Radius of the circle with centre A)
 OAP is equilateral.
 ∠PAO = 60
 ∠BAP = 120
In BAP, we have BA = AP and ∠ BAP = 120
 ∠ABP = ∠APB = 30
Similarly we can get ∠ABQ = 30
 ∠PBQ = 60

www.topperlearning.com 11
CBSE X | Mathematics

Board Paper – 2016 Solution All India Set – 1

24. AO’ = O’X = XO = OC …..(Since the two circles are equal.)


So, OA = AO’ + O’X + XO …..(A-O’-X-O)
 OA = 3O’A
In AO'D and AOC,
DAO' = CAO ....(Common angle)
ADO' = ACO ....(both measure 90)
 ADO' ~ ACO ....(By AA test of similarity)
DO' O'A O'A 1
   
CO OA 3O'A 3

1 2 4
25.
x 1 x 2 x 4
L.C.M. of all the deno min ators is (x  1)(x  2)(x  4)
Multiply throughout by the L.C.M., we get
(x  2)(x  4)  2(x  1)(x  4)  4(x  1)(x  2)
 (x  4)(x  2  2x  2)  4(x2  3x  2)
 (x  4)(3x  4)  4x2  12x  8
 3x2  16x  16  4x2  12x  8
 x2  4x  8  0
Now, a  1,b  4, c  8
b  b2  4ac 4  16  32 4  48 4  4 3
x   
2a 2 2 2
x  2 2 3

26. MP = YX = 40 m
 QM  h  40
In right angled QMY,
QM h - 40
tan 45  1 ....(MY = PX)
MY PX
 PX  h  40 ....(1)
In right angled QPX,
QP QP
tan60   3 
PX PX
h
 PX  ....(2)
3
h
From (1) and (2), h  40 
3
 3h  40 3  h
 3h  h  40 3
 1.73h  h  40(1.73)  h  94.79 m
Thus, PQ is 94.79 m.

www.topperlearning.com 12
CBSE X | Mathematics

Board Paper – 2016 Solution All India Set – 1

27. Let there be a value of x such that the sum of the numbers of the houses
preceding the house numbered x is equal to the sum of the numbers of the
houses following it.
That is, 1  2  3  . . . .  (x - 1)  (x  1)  (x  2)  . . . . .  49
 1  2  3  . . . .  (x  1)
 [1  2  ......  x  (x  1)  ....  49]  (1  2  3  . . . .  x)
x 1 49 x
 [1  x  1]  [1  49]  [1  x]
2 2 2
 x(x  1)  49  50  x(1  x)
 x(x  1)  x(1  x)  49  50
 x2  x  x  x2  49  50
 x2  49  25
 x  7  5  35
Since x is not a fraction, the value of x satisfying the given condition exists
and is equal to 35.

AD AE 1
28.
AB AC 3
AB AC
  3
AD AE
AD  DB AE  EC
  3
AD AE
DB EC
1  1 3
AD AE
DB EC
  2
AD AE
AD AE 1
  
DB EC 2
 AD : DB  AE : EC  1 : 2
So, D and E divide AB and AC respectively in the ratio 1 : 2.
So the coordinates of D and E are
 1  8 5  12   17   7  8 2  12   14 
 1  2 , 1  2    3, 3  and  1  2 , 1  2    5, 3  respectively.
       

www.topperlearning.com 13
CBSE X | Mathematics

Board Paper – 2016 Solution All India Set – 1

Area of ADE
1  17 14   17 14 
=  4  3  3  3  5  6   3  6  5  3  4  3 
2    
1  68   85 56 
=   14  30   18  
2 3   3 3 
1  68  42  90   54  85  56 
= 
2  3   3 

1  200   195 
= 
2  3   3 
1 5
= 
2 3
5
 sq. units
6

4 1 7 4

6 5 2 6

Area of ABC
1
=  4  5  1  2  7  6   1  6  7  5  4  2 
2
1
= 20  2  42    6  35  8 
2
1
= 64  49
2
15
= sq. units
2
5
Area of ADE 1
  6 
Area of ABC 15 9
2

29. x is selected from 1,2,3 and 4


1,2,3, 4
y is selected from 1, 4,9 and 16
Let A  {1, 4,9,16,2,8,18,32,3,12,27, 48,36, 64} which consists
of elements that are product of x and y
Number of outcomes less than 16
P(product of x and y is less than 16) 
Total number of outcomes
7

14
1

2

www.topperlearning.com 14
CBSE X | Mathematics

Board Paper – 2016 Solution All India Set – 1

30. Perimeter of shaded region  AB  PB  arc length AP...(1)


 r
Arc length AP   2r  ....(2)
360 180
In right angled OAB,
AB
tan =  AB  r tan ....(3)
r
OB
sec  =  OB  r sec 
r
OB  OP  PB
 r sec   r  PB
 PB  r sec   r.....(4)
Substitute (2), (3) and (4) in (1), we get
Perimeter of shaded region  AB  PB  arc length AP
r
 r tan   r sec   r 
180
  
 r tan   sec   1
 180 

31. Let the speed of the stream be s km/h.

Speed of the motor boat  24 km / h


Speed of the motor boat upstream  24  s
Speed of the motor boat downstream  24  s
According to the given condition,
32 32
 1
24  s 24  s
 1 1 
 32    1
 24  s 24  s 
 24  s  24  s 
 32   1
 576 - s2 
2
 32  2s = 576  s
 s2  64s  576  0
 (s  72)(s  8)  0
 s  72 or s  8
Sin ce, speed of the stream cannot be negative, the speed of the
stream is 8 km / h.

www.topperlearning.com 15
CBSE X | Mathematics

Board Paper – 2017 All India Set – 3

CBSE
Class X Mathematics
Board Paper – 2017
All India Set – 3
Time: 3 hours Total Marks: 90

General Instructions:
(i) All questions are compulsory.
(ii) The question paper consists of 31 questions divided into four sections – A, B, C
and D.
(iii) Section A contains 4 questions of 1 mark each. Section B contains 6 questions
of 2 marks each, Section C contains 10 questions of 3 marks each and Section
D contains 11 questions of 4 marks each.
(iv) Use of calculators is not permitted.

SECTION A

Question numbers 1 to 4 carry 1 mark each.

1. The probability of selecting a rotten apple randomly from a heap of 900 apples
is 0.18. What is the number of rotten apples in the heap?

2. If a tower 30 m high, casts a shadow 10 3 m long on the ground, then what is


the angle of elevation of the sun?

3. If the angle between two tangents drawn from an external point P to a circle of
radius a and centre O, is 60°, then find the length of OP.

4. What is the common difference of an A.P. in which a21 – a7 = 84?

SECTION B

Question numbers 5 to 10 carry 2 marks each.

5. A circle touches all the four sides of a quadrilateral ABCD. Prove that
AB + CD = BC + DA

6. Prove that the tangents drawn at the end points of a chord of a circle make
equal angles with the chord.

7. A line intersects the y-axis and x-axis at the points P and Q respectively. If
(2, -5) is the mid-point of PQ, then find the coordinates of P and Q.

www.topperlearning.com 1
CBSE X | Mathematics

Board Paper – 2017 All India Set – 3

8. If the distances of P(x, y) from A(5, 1) and B(-1, 5) are equal, then prove that
3x = 2y.

9. Find the value of p, for which one root of the quadratic equation px2 – 14x + 8
= 0 is 6 times the other.

10. For what value of n, are the nth terms of two A.Ps 63, 65, 67,…. and 3, 10,
17,….. equal?

SECTION C

Question numbers 11 to 20 carry 3 marks each.

11. On a straight line passing through the foot of a tower, two points C and D are
at distances of 4 m and 16 m from the foot respectively. If the angles of
elevation from C and D of the top of the tower are complementary, then find
the height of the tower.

12. A bag contains 15 white and some black balls. If the probability of drawing a
black ball from the bag is thrice that of drawing a white ball, find the number
of black balls in the bag.

13. Three semicircles each of diameter 3 cm, a circle of diameter 4.5 cm and a
semicircle of radius 4.5 cm are drawn in the given figure. Find the area of the
shaded region.

 24 
14. In what ratio does the point  , y  divides the line segment joining the points
 11 
P(2, -2) and Q(3, 7)? Also find the value of y.

www.topperlearning.com 2
CBSE X | Mathematics

Board Paper – 2017 All India Set – 3

15. Water in a canal, 5.4 m wide and 1.8 m deep, is flowing with a speed of 25
km/hour. How much area can it irrigate in 40 minutes, if 10 cm of standing
water is required for irrigation?

16. In the given figure, two concentric circles with centre O have radii 21 cm and
 22 
42 cm. If ∠AOB = 60°, find the area of the shaded region.  Use  
 7 

17. The dimensions of a solid iron cuboid are 4.4 m × 2.6 m × 1.0 m. It is melted
and recast into a hollow cylindrical pipe of 30 cm inner radius and thickness 5
cm. Find the length of the pipe.

18. A toy is in the form of a cone of radius 3.5 cm mounted on a hemisphere of


same radius on its circular face. The total height of the toy is 15.5 cm. Find the
total surface area of the toy.

19. How many terms of an A.P. 9, 17, 25, …. must be taken to give a sum of 636?

20. If the roots of the equation (a2 + b2) x2 – 2(ac + bd) x + (c2 + d2) = 0 are
a c
equal, prove that  .
b d

SECTION D

Question numbers 21 to 31 carry 4 marks each.

21. If the points A(k + 1, 2k), B(3k, 2k + 3) and C(5k – 1, 5k) are collinear, then
find the value of k.

www.topperlearning.com 3
CBSE X | Mathematics

Board Paper – 2017 All India Set – 3

22. Construct a triangle ABC with side BC = 7 cm, ∠B = 45°, ∠A = 105°. Then
3
construct another triangle whose sides are times the corresponding sides of
4
the  ABC.

23. Two different dice are thrown together. Find the probability that the numbers
obtained have
(i) even sum, and
(ii) even product

24. In the given figure, XY and X’Y’ are two parallel tangents to a circle with centre
O and another tangents AB with point of contact C, is intersecting XY at A and
X’Y’ at B. Prove that ∠AOB = 90°.

25. In a rain–water harvesting system, the rain-water from a roof of 22 m × 20 m


drains into a cylindrical tank having diameter of base 2 m and height 3.5m. If
the tank is full, find the rainfall in cm. Write your views on water conservation.

26. Prove that the lengths of two tangents drawn from an external point to a circle
are equal.

27. If the ratio of the sum of the first n terms of two A.Ps is (7n + 1) : (4n + 27),
then find the ratio of their 9th terms.

28. Solve for x:


x  1 2x  1 1
  2,where x   ,1
2x  1 x  1 2

29. A takes 6 days less than B to do a work. If both A and B working together can
do it in 4 days, how many days will B take to finish it?

30. From the top of a tower, 100 m high, a man observe two cars on the opposite
sides of the tower and in same straight line with its base, with its base, with
angles of depression 30° and 45°. Find the distance between the cars.
 Take 3  1.732
 

www.topperlearning.com 4
CBSE X | Mathematics

Board Paper – 2017 All India Set – 3

31. In the given figure, O is centre of the circle with AC = 24 cm, AB = 7 cm and
∠BOD = 90°. Find the area of the shaded region.

www.topperlearning.com 5
CBSE X | Mathematics

Board Paper – 2017 Solution All India Set – 3

CBSE
Class X Mathematics
Board Paper – 2017 Solution
All India Set – 3
Time: 3 hours Total Marks: 90

SECTION A

1. Let the total number of rotten apples in a heap = n


Total number of apples in a heap = 900
Probability of selecting a rotten apple from a heap = 0.18
Now,
Number of rotten apples
P(selecting a rotten apple) 
Total number of apples
n
 0.18 
900
 n  0.18  900
 n  162
Hence, the number of rotten apples is 162.

2. Let AB be the tower and BC be its shadow.


AB  30 m, BC  10 3 m
In ABC,
AB
tan  
BC
30
 tan  
10 3
3
 tan  
3
tan   3
But, tan 60  3
   60
Thus, the angle of elevation of sun is 60.

www.topperlearning.com 1
CBSE X | Mathematics

Board Paper – 2017 Solution All India Set – 3

3. In the figure, PA and PB are two tangents from an external point P to a circle
with centre O and radius = a
APB  60 (given)
 APO  30 (tangents are equally inclined to the
line joining the point and the centre)
Now, OA  AP
In right-angled triangle OAP,
OA
sin 30 
OP
1 a
 
2 OP
 OP  2a

4. Let a be the first term and d be the common difference of the given A.P.
 a21  a7  84
  a  20d   a  6d  84
 a  20d  a  6d  84
 14d  84
d6
Hence, the common difference is 6.

SECTION B

5. Since tangents drawn from an external point to a circle are equal in length, we
have
AP = AS ….(i)
BP = BQ ….(ii)
CR = CQ ….(iii)
DR = DS ….(iv)
Adding (i), (ii), (iii) and (iv), we get
AP + BP + CR + DR = AS + BQ + CQ + DS
 (AP + BP) + (CR + DR) = (AS + DS) + (BQ + CQ)
 AB + CD = AD + BC
 AB + CD = BC + DA

www.topperlearning.com 2
CBSE X | Mathematics

Board Paper – 2017 Solution All India Set – 3

6. Let AB be a chord of circle with centre O.


Let AP and BP be two tangents at A and B respectively.
Suppose the tangents meet at point P. Join OP.
Suppose OP meets AB at C.
Now, in ΔPCA and ΔPCB,
PA = PB ….(tangents from an external point are
equal)
∠APC = ∠BPC ….(PA and PB are equally inclined to OP)
PC = PC ….(common)
Hence, ΔPAC  ΔPBC ….(by SAS congruence
criterion)
 ∠PAC = ∠PBC

7. Since a line is intersecting y-axis at P and x-axis at Q,


Coordinates of P = (0, y) and coordinates of Q = (x, 0)
Let R be the mid-point of PQ.
0  x y  0
Then, co-ordinates of R   ,  2, 5 
 2 2 
x y
  ,   2, 5
2 2
x y
  2 and  5
2 2
 x  4 and y  10
Hence, co-ordinates of P are 0,  10  and co-ordinates of Q are  4, 0  .

8. Given, P(x, y) is equidistant from A(5, 1) and B(–1, 5)


Now, AP  BP

5  x   1  y    1  x   5  y 
2 2 2 2

 5  x   1  y    1  x   5  y 
2 2 2 2

      
 25  x2  10x  1  y2  2y  1  x2  2x  25  y2  10y 
 x2  y2  10x  2y  26  x2  y2  2x  10y  26
 10x  2x  10y  2y
 12x  8y
 3x  2y ....(Dividin g throughout by  4)

www.topperlearning.com 3
CBSE X | Mathematics

Board Paper – 2017 Solution All India Set – 3

9. Given, px2 – 14x + 8 = 0


Here, a  p, b  14, c  8
Let  and  be the roots of the given quadratic equation.
Then,   6
b
Now, sum of the roots 
a
(14)
  
p
14
  
p
14
   6 
p
14
 7 
p
2
 ....(i)
p
c
Product of the roots 
a
8
  
p
8
   6 
p
8
 6 2 
p
4
 32 
p
2
2 4
 3   ....[From (i)]
p p
 4 4
 3 2  
p  p
p3

www.topperlearning.com 4
CBSE X | Mathematics

Board Paper – 2017 Solution All India Set – 3

10. For A.P. 63, 65, 67, ….., we have


first term = 63 and common difference = 65 – 63 = 2
Hence, nth term = an = 63 + (n – 1)2
 an = 63 + 2n – 2 = 2n + 61
For A.P. 3, 10, 17, ….., we have
first term = 3 and common difference = 10 – 3 = 7
Hence, nthterm  an'  3  n  1 7
 an'  3  7n  7  7n  4
The two A.Ps will have identical nth term, if
an  an'
 2n  61  7n  4
 5n  65
 n  13

www.topperlearning.com 5
CBSE X | Mathematics

Board Paper – 2017 Solution All India Set – 3

SECTION C

11.

Let AB be the tower with height h.


Let x be the angle of elevation from C.
So, the angle of elevation from D is 90  x  .
....(Sin ce the angles of elevation from C and D are complementary)
In CAB,
AB
tan x 
AC
h
 tan x  ......(i)
4
In DAB,
AB
tan(90  x) 
AD
h
 tan(90  x) 
16
h
 cot x  ......(ii)
16
From (i) and (ii),
h h
tan x  cot x  
4 16
h2
1
64
 h2  64
 h  64
h8 m
Hence, the height of the tower is 8 m.

www.topperlearning.com 6
CBSE X | Mathematics

Board Paper – 2017 Solution All India Set – 3

12. Let the number of black balls in the bag be x.


Number of white balls = 15
Hence, total number of balls in the bag = x + 15
Given, P(black ball)  3  P(white ball)
x 15
  3
x  15 x  15
x 45
 
x  15 x  15
 x  45
Thus, the number of black balls in the bag is 45.

13.

3
Radius of semi-circle A  cm  1.5 cm
2
3
Radius of semi-circle B  cm  1.5 cm
2
3
Radius of semi-circle C  cm  1.5 cm
2
4.5
Radius of circle D  cm  2.25 cm
2
Radius of semi-circle E  4.5 cm
Now, area of the shaded region
 Area of semi-circle (E  B)  Area of semi-circle (A  C)  Area of circle D
1  1
  4.5  1.5     1.5   1.5     2.25 
2 2 2 2 2

2   2  
1 1
  20.25  2.25   2.25  2.25   5.0625 
2 2
1 1
   22.50    4.50  5.0625
2 2
 11.25  2.25  5.0625
 3.9375
22
 3.9375 
7
 12.375 cm2

www.topperlearning.com 7
CBSE X | Mathematics

Board Paper – 2017 Solution All India Set – 3

14.
 24 
Suppose the point A  , y  divides the line segment joining points
 11 
P 2, 2  and Q 3,7  in the ratio k : 1.
 3k  2 7k  2 
Then, the coordinates of A are  , 
 k 1 k 1 
 24 
But, the coordinates of A are given as  , y .
 11 
3k  2 24
 
k 1 11
 33k  22  24k  24
2
 9k  2  k 
9
Hence, the ratio is 2 : 9.
2
7 2
7k  2 9
Also, y y
k 1 2
1
9
14  18
9 4 9 4
y   
29 9 11 11
9

15.
We have,
Width of the canal  5.4 m,
Depth of the canal  1.8 m
It is given that the water is flowing with a speed of 25 km / hr.
Therefore,
Length of the water column formed in 40 mins
40 2
that is, hours  hours
60 3
2 50 50  1000 50000
is 25  km  km  m m
3 3 3 3
2
 Volume of the water flowing in hours
3
50000
 Volume of the cuboid of length m, width 5.4 m and depth 1.8 m
3
2
 Volume of the water flowing in hours
3
50000
  5.4  1.8
3
 162000 m3
This volume  volume of cuboid (10 cm of standing water is required for irrigation)
This volume  base area of field  0.1m
162000
base area 
0.1
Hence, the cannal irrigates 1620000 m2 area in 40 mins

www.topperlearning.com 8
CBSE X | Mathematics

Board Paper – 2017 Solution All India Set – 3

16. We have,

Area of the region ABCD


 Area of sector AOB  Area of sector COD
 60 22 60 22 
   42  42    21  21  cm2
 360 7 360 7 
1 1 
   22  6  42   22  3  21  cm2
6 6 
 22  42  11  21 cm2
  924  231 cm2
 693 cm2
 22 22 
Area of circular ring    42  42   21  21  cm2
 7 7 
 22  6  42  22  3  21 cm2
 5544  1386  cm2
 4158 cm2
Hence, Required shaded region  Area of circular ring  Area of region ABCD
  4158  693 cm2
 3465 cm2

www.topperlearning.com 9
CBSE X | Mathematics

Board Paper – 2017 Solution All India Set – 3

17. Let the length of the pipe be h cm.


Then, volume of iron pipe = volume of iron in the block.
Volume of the block   4.4  2.6  1 m3   440  260  100  cm3
r  Internal radius of the pipe  30 cm
R  External radius of the pipe  (30  5) cm  35 cm
 Volume of the iron pipe  External Volume    Internal Volume 
 R 2h  r2h
 (R 2  r2 )h
 (R  r)(R  r)h
 (35  30)(35  30)h
   65  5  h
Now, Volume of iron in the pipe  Volume of iron in the block
   65  5  h  440  260  100
22
  65  5  h  440  260  100
7
440  260  100  7
h  11200 cm
22  65  5
 h  112 m
Thus, the length of the pipe is 112 m.

18. Radius of common base = 3.5 cm


Total height of toy  15.5 cm
Height of cone  15.5  3.5  12 cm
For cone,
l2  r2  h2
 l2  3.5  12 
2 2

 l2  12.25  144
 l2  156.25
 l  156.25  12.5 cm
 Total surface areaof the toy
 Curved surface area of cone  Curved surface area of hemisphere
 rl  2r 2
22 22
  3.5  12.5  2   3.5  3.5
7 7
22
  3.5 12.5  7
7
22
  3.5  19.5
7
 214.5 cm2

www.topperlearning.com 10
CBSE X | Mathematics

Board Paper – 2017 Solution All India Set – 3

19. Let there be n terms of this A.P.

For this A.P., a  9


d  a2  a1  17  9  8
n
Sn  2a  n  1 d
2
n
 636  2  9  n  1 8 
2
 636  n 9  n  1 4 
 636  n 9  4n  4 
 636  n  4n  5 
 4n2  5n  636  0
 4n2  53n  48n  636  0
 n  4n  53  12  4n  53   0
  4n  53 n  12   0
 4n  53  0 or n  12  0
53
n or n  12
4
Sin ce number of terms can neither be negative nor fractional,
we have n  12

20. We have
a2
 
 b2 x2  2  ac  bd x  c2  d2  0 
The discriminant of the given equation is given by

  
D   2  ac  bd   4  a2  b2  c2  d2 
2


 D  4  ac  bd  4 a2c2  a2d2  b2c2  b2d2
2

  
 D  4 a2c2  b2d2  2abcd  4 a2c2  a2d2  b2c2  b2d2 
 D  4  a c  b d  2abcd  a c
2 2 2 2 2 2
 a2d2  b2c2  b2d2 
 D  4 2abcd  a d  b c 
2 2 2 2

 D  4  ad  bc   2  ad bc  


2 2

 
 D  4  ad  bc 
2

The given equation will have equal roots,if D  0


 4(ad  bc)2  0
 (ad  bc)2  0
 ad  bc  0
 ad  bc
a c
 
b d

www.topperlearning.com 11
CBSE X | Mathematics

Board Paper – 2017 Solution All India Set – 3

SECTION D

21. Given points are A(k + 1, 2k), B(3k, 2k + 3) and C(5k – 1, 5k)
These points will be collinear, if area of the triangle formed by them is zero.
We have,

i.e.,
k 1 2k 3 3k 5k 5k 1 2k 3k 2k 5k 1 2k 3 k 1 5k 0

2k 2 5k 3 15k 2 10k 2 2k 6k 2 10k 2 13k 3 5k 2 5k 0

27k 2 3k 3 21k 2 18k 3 0


27k 2 3k 3 21k 2 18k 3 0
2
6k 15k 6 0
2
2k 5k 2 0
2
2k 4k k 2 0
(k 2)(2k 1) 0
k 2 0 or 2k 1 0
1
k 2 or k
2

www.topperlearning.com 12
CBSE X | Mathematics

Board Paper – 2017 Solution All India Set – 3

22. Steps of construction:


1) Draw BC  7 cm
2) At B, construct CBX  45 and at C construct BCY  180  (45  105)  30
3) Let BX and CY intersect at A. ABC so obtained is the given triangle.
4) Construct an acute angle CBZ at B on opposite side of vertex A of ABC.
5) Mark-off four points (greater of 4 and 3 in 3/4) points B1 ,B2 ,B3 ,B4 on BZ
such that BB1  B1B2  B2B3  B3B4
6) Join B4 to C.
7) Draw B3C ' parallel to B4C which meets BC at C'.
8) From C', draw C'A' parallel to CA meeting BA at A'.
3
Thus, A'BC' is the required triangle, each of whose sides is times
4
the corresponding sides of ABC.

www.topperlearning.com 13
CBSE X | Mathematics

Board Paper – 2017 Solution All India Set – 3

23. Elementary events associated to the random experiment of throwing two dice
are:
(1,1), (1,2), (1,3), (1, 4), (1,5), (1,6),
(2,1), (2,2), (2,3), (2, 4), (2,5), (2,6),
(3,1), (3,2), (3,3), (3, 4), (3,5), (3,6),
(4,1), (4,2), (4,3), (4, 4), (4,5), (4,6),
(5,1), (5,2), (5,3), (5, 4), (5,5), (5,6),
(6,1), (6,2), (6,3), (6, 4), (6,5), (6,6)
 Total number of elementary events  6  6  36
(i) Let A be the event of getting an even number as the product.
i.e., 2,4,6,8,10,12,
Elementary events favourable to event A are:
(1,1),(1,3),(1,5),(2,2),(2, 4),(2,6),(3,1),(3,3),(3,5),
(4,2),(4, 4),(4,6),(5,1),(5,3),(5,5),(6,2),(6, 4),(6,6)
 Total number of favourable events  18
18 1
Hence, required probability  
36 2
(ii) Let B be the event of getting an even number as the sum.
i.e., 2,4,6,8,10,12,16,18,20,24,30,36
Elementary events favourable to event B are:
(1,2),(1, 4),(1,6),(2,1),(2,2),(2,3),(2, 4),(2,5),
(2,6),(3,2),(3, 4),(3,6),(4,1),(4,2),(4,3),(4, 4),
(4,5),(4,6),(5,2),(5, 4),(5,6),(6,1),(6,2),(6,3),
(6, 4),(6,5),(6,6)
 Total number of favourable events  27
27 3
Hence, required probability  
36 4

www.topperlearning.com 14
CBSE X | Mathematics

Board Paper – 2017 Solution All India Set – 3

24. Since tangents drawn from an external point to a circle are equal.
Therefore, AP = AC.
Thus, in triangles AOP and AOC, we have
AP = AC
AO = AO [Common side]
OP = OC [Radii of the same circle]
So, by SSS- criterion of congruence,
we have
AOP  AOC
 PAO = CAO
 PAC = 2CAO
Similarly, we can prove that QBO = CBO
 CBQ = 2CBO
Now, PAC +CBQ = 1800
[sum of the interior angle on the same side of transversal is 180° ]

 2CAO +2 CBO = 180° [Using equations (i) and (ii)]


 CAO +CBO = 90°
 180°   AOB = 90° [Since CAO, CBO and AOB are angles of a
Triangle, CAO + CBO + AOB = 180°]
 AOB = 90°

25. We have,
r  Radius of cylindrical vessel  1m
h  Height of cylindrical vessel  3.5 m
22
 Volume of cylindrical vessel  r 2h   12  3.5 m3  11 m3
7
Let the rainfall be x m.
Then, Volume of the water
 Volume of cuboid of base 22 m  20 m and height x metres
 (22  20  x) m3
Since the vessel is just full of the water that drains out of the roof into the vessel,
Volume of the water  Volumeofthecylindricalvessel
 22  20  x  11
11 1 100
x  m cm  2.5 cm
22  20 40 40
Thus, the rainfall is 2.5 cm.

www.topperlearning.com 15
CBSE X | Mathematics

Board Paper – 2017 Solution All India Set – 3

26. Given: AP and AQ are two tangents from a point A to a circle C(O, r)
To prove: AP = AQ
Construction: Join OP, OQ and OA

Proof:

In OPA and OQA,


OPA  OQA  90 ....(T an gent at any point of a circle is perpendicular
to the radius through the point of contact)
OP  OQ ....(Radii of a circle)
OA  OA ....(Common)
Hence, by RHS-criterion of congruence, we have
OPA  OQA
 AP  AQ ....(c.p.c.t)

www.topperlearning.com 16
CBSE X | Mathematics

Board Paper – 2017 Solution All India Set – 3

27. Let a1, a2, be the first terms and d1, d2 the common differences of the two
given A.P’s.
Then, sum of their n terms is given by
n n
Sn  2a1  n  1 d1  and Sn'  2a2  n  1 d2 
2 2
n
2a1  n  1 d1  2a1  n  1 d1
Sn 2
 '  
Sn n
2a2  n  1 d2  2a2  n  1 d2
2
It is given that,
Sn 7n  1
'

Sn 4n  27
2a1  n  1 d1 7n  1
  ....(i)
2a2  n  1 d2 4n  27
In order to find the ratio of the mth terms of the two given A.P's,
we replace n by 2m  1 in equation (i).
Thus, to find the ratio of the 9th terms of the two given A.P's,
we replace n by 17 2  9  1 in equation (i)
2a1  17  1 d1 7  17  1

2a2  17  1 d2 4  17  27
2a1  16d1 120
 
2a2  16d2 95
a1  8d1 24
 
a2  8d2 19
Thus, the ratio of their 9th terms is 24:19.

www.topperlearning.com 17
CBSE X | Mathematics

Board Paper – 2017 Solution All India Set – 3

x 1 2x 1
28. 2
2x 1 x 1
2 2
x 1 2x 1
2
2x 1 x 1
x2 2x 1 4x2 4x 1
2
2x2 2x x 1
5x2 2x 2
2
2x2 x 1
5x2 2x 2 4x2 2x 2
5x2 2x 2 4x2 2x 2 0
x2 4x 4 0
2
x 2x 2x 4 0
x x 2 2 x 2 0
x 2 x 2 0
2
x 2 0
x 2 0
x 2

www.topperlearning.com 18
CBSE X | Mathematics

Board Paper – 2017 Solution All India Set – 3

29. Suppose B alone takes x days to finish the work.


Then, A alone can finish it in (x – 6) days
Now,
1 1
 A ' s one day's work   B ' s one day's work   x  6  x
1
And, (A  B)' s one day's work 
4
1 1 1
  
x6 x 4
xx6 1
 
x(x  6) 4

2x  6 1
 2

x  6x 4
 8x  24  x2  6x
 x2  6x  8x  24  0
 x2  14x  24  0
 x2  12x  2x  24  0
 x  x  12   2  x  12   0
  x  12   x  2   0
 x  12  0 or x  2  0
 x  12 or x  2
But, x cannot be less than 6.
So, x  12
Hence, B alone can finish the work in 12 days.

www.topperlearning.com 19
CBSE X | Mathematics

Board Paper – 2017 Solution All India Set – 3

30.

The man is at the top of the tower AB.


In right angled triangle ABX and ABY,
AB 100
tan 45  1  XB = 100 m
XB XB
AB 1 100
tan30     YB = 100 3
YB 3 YB
XY = XB + YB = 100+100 3  273.20 (approx)
Hence, the distance between X and Y is 273.20 m approximately.

www.topperlearning.com 20
CBSE X | Mathematics

Board Paper – 2017 Solution All India Set – 3

31. AC = 24 cm, AB = 7 cm

Since BC is the diameter of the circle,


so , BAC  90
In right BAC,
BC2  AC2  AB2
 BC2  242  72
 BC2  625
 BC  25 cm
So, the radius of the circle  OC  12.5 cm
Area of the shaded region
 Area of the circle  Area of BAC  Area of sector CD
1 
= r2   AB  AC   r 2
2 360
 22  1   90 22 
=  12.5  12.5     7  24      12.5  12.5 
 7  2   360 7 
....( BOD = 90  COD = 90)
 491.07  84  122.77
 284.3 cm2 (approximately)
Hence, the area of the shaded region is 284.3 cm2 approximately.

www.topperlearning.com 21

You might also like